Boards- 3

¡Supera tus tareas y exámenes ahora con Quizwiz!

At which level are the intercostals *fully* innervated

T11

At which levels are abdominal muscles *fully* innervated

T11

DTR abs

T8-12

What is the difference b/w "weak" and "strong" C5 quadriplegia?

Weak: biceps & shoulder mm have an MMT grade of fair or less Strong: those mm have grades of fair+ or better

Shoulder pain is caused by ___ in the shoulder girdle mm, esp if the only mm innervated are those at C__ &/or higher.

Weakness; C6

precautions for posterior hip dislocation

flexion >90 IR ADD past neutral

After stroke: trunk

flexion and lateral flexion toward involved side

at greater than 30 degrees the ITB is a _____

flexor

After stroke: knee

flexor or extensor tone

non-dominant hemisphere strokes lead to

hemi-neglect unawareness of midline visual perceptual disordders visual neglect

treatment of pelvis fracture

includes exercise within the framework of the injury and to patient tolerance, gait training with the most appropriate assistive device.

garden stage classification terminology type 1

incomplete or impacted fracture (may not require surgical fixation)

roles of intensity of therapy

increase motor reorganization 1200 reps

TBI severity based on duration of amnesia

mild- <24 hours moderate- 1-7 days severe- 1-4 weeks very severe >4 weeks

toe amputation 2-5

minimal balance loss no treatment

explain first degree strain

minimal damage, early resolution o Onset 24-48 hours after exercise o Sudden overstretch/contraction

Harris hemi-sling

minimize inferior sublux AD and IR weight over both shoulders

recovery of diaschisis occurs in __ portions of the brain

more distal

stellate fracture of the patella

multi-fragmented fracture usually the result of falling onto a flexed knee.

what is hip pinning

multiple threaded pins are applied at varying angles across the fracture site. It may take 3-6 months for the fracture to heal, during which weight bearing is limited or prohibited.

intrathecal baclofen pump

muscle relaxant blocks the release of excitatory NTs in the spinal cord restores the balance of excitatory/inhibitory input to reduce muscle hyperactivity

majority of strains involved _____

muscles that cross two joints

_____% of body weight at normal walking speed

300-400%

excessive anteversion

35 degrees

tibia should have ___ degrees of varus in frontal plane

4-6

___ facilitates stability thru a joint & is used w/ PNF for weight-bearing joints.

Approximation

approximation in PNF

Approximation is gentle compression of the joint surfaces and is done manually or through weight-bearing. Traction feeds information about position into the joint receptors. PNF advocates that (opposed to traction) approximation facilitates stability through a joint, thus is used for weight bearing joints.

typical development LE bone: knee

Birth: genu varum normal in newborns and non-walking infants 1-2 years old: reach neutral alignment 2-4 years old: progress toward genu valgum (compensate with intoeing) By 11 years old: genu valgum has decreased to an average of 5-6 degrees

neuromuscular scoliosis

Caused by neurologic or muscular disorders

conus medullaris lesion

Causes- injury to bottom of SC and lumbar nerve roots What happens? • Results in flaccidity and a lack of return of bowel, bladder, or sexual function. Prognosis • If the bottom of the cord is damaged, there will be no return of function

DTR triceps

C6-7

For stroke rehab, the practice the pts do must be ___ enough to force learning.

Challenging

___ is the enhancement of the response of a neuron to a stimulus following stimulation.

Facilitation

Is any one sling better than another?

Nope

Is pain directly associated w/ inferior subluxation of the GH post-stroke?

Nope

Is sexual activity contraindicated after stroke?

Nope

cortical blindness

posterior cerebral artery

DD tests for PCL

posterior drawer test tibial sag (Godfrey)

affected hip is low and symptoms increase with quadruped rock-back for ____

posterior glide syndrome multidirection hypermobility

with femoral anterior glide syndrome you have stiffness of ____

posterior joint capsule

flexion synergy of the LE

posterior tilt pelvic forward rotation hip flex hip ER and AB knee flex DF supination GT ext, 2-5 flex

GFRV of gastroc- loading

posterior to ankle, use eccentric DF

GFRV tibialis anterior- loading

posterior to ankle, use eccentric DF

GFRV glute max and hamstrings- midstance

posterior to hip joint, use eccentric flexion

ortolan's maneuver

reduces a dislocated hip

Metatarso-Phalangeal joint

-structure: biaxial (condylar) -axis: lateral; anterior-posterior -motion: flexion and extension; abduction and adduction

Interphalangeal

-structure: uniaxial -axis: lateral -motion: flexion and extension

ischemic stroke

(88%) occurs when arterial supply to the brain is blocked. Caused by narrowing of arteries Also caused by blood clots moving from the heart occurring as a result of irregular heartbeat, MI, or valve abnormalities

Causes of hallux valgus

-Poor fitting shoes *Too tight *Toe box too short -Activities *Ballet -Excessive pronation

Pump bumps (haglund syndrome)

-Poor fitting shoes -bump is frequently a bone spur

Contributing factors to posterior tibial tendonitis

-Poor flexibility *Short triceps surae complex *Shortened posterior tibialis (pes cavus) -Weak musculature *posterior tibialis *Flexor hallucis longus and flexor digitorum (work with post. Tib. As dynamic stabilizers of longitudinal arch) -Alignment abnormalities that lead to over pronation -Training conditions (poor foot wear, over training, training surface)

Broca's aphasia is (expressive/receptive?).

Expressive

A(n) (areflexic/reflexic?) bowel can lead to constipation.

Areflexic

In a(n) (areflexic/reflexic?) bladder, parasympathetic control is altered & results in urinary retention, dribbling, overflow, & incontinence as buildup occurs. Also can cause kidney failure over time.

Areflexic

Pts w/ T12/L1 injuries often have problems w/ (areflexic/reflexic?) bladder bc the sacral reflex arc is disrupted

Areflexic (flaccid)

If the defecation reflex arc remains intact, but the stronger parasympathetic defecation reflex is lost, the bowel will function (areflexively/reflexively?).

Areflexively

When tx'ing a pt in acute care, how aggressive should you be?

As aggressive as they are medically able

If the pt has a LE orthotic, do you put it on over the sequential compression device, or underneath?

Over the top; the compression device should be against the skin

the fibers of the _____ are almost vertical

PCL

T/F: Ppl at a level C7 are independent in all bed mobility & usually w/ all transfers.

T

red flags for cancer

Age 50 years or older History of Cancer Unrelenting Night Pain Unexplained weight loss/gain Malaise/fatigue Worsening or unresolved pain at 4 weeks

GFRV quads- midstance

anterior to knee, use flexors

muscular causes of torticollis

Birth trauma Fetal malposition Uterine compression Inflammatory conditions Palpable lump or pseudotumor in SCM muscle in slightly more than 1/3 of cases (fibrous tumor)

ASIA L5

big toe extensors

causes of coordination problems

Can be from decreased stability, altered motor programs, and increase in altered postural support

how are femoral neck fractures described

garden stage classification terminology

where do synapsis occur

gray matter

in extension the contact of the patella is more ____

inferior

what lesions lead to spatial neglect

inferior parietal lobe lesions

activating the hamstrings can pull on ____

medial meniscus

amnesia is damage to what lobe and structure

medial temporal lobe hippocampus

verbal 1 glasgow

no response

ranchos level I

no response- complete absence of observable change in behavior when presented with visual, auditory, tactile, proprioceptive, vestibular, or painful stimuli

most likely mechanism of achilles tendonitis

overpronation decreased DF weak PF altered blood supply

patient education with splint/orthotic

skin care circulation how to put brace on how to care for brace

what is trochanteric bursitis associated with

snapping hip

tests for syndesmotic injury

squeeze test clunk/side to side test one-leg hop test point test ER test DF (SCOPED)

indications for CMC thumb splint

stabilize MCP & CMC OA

indications for MCP thumb splint

stabilize MCP and allow CMC RA sprain of 1st ulnar collateral ligament (Gamekeeper's) sprain of 1st radial collateral ligament inflammation or instability of 1st MCP

verbal 3 glasgow

states recognizable words

___ test for CAM

Fabere's test is extremely limited and produces anterior groin pain .

causes of cuboid syndrome

subluxation of cuboid abnormal pull of fibularis longus increased pronation

DD for anterior talofibular ligament

test for ATF alternate test (anterior drawer)

joints of the knee (3)

tibiofemoral patellofemoral tibiofibular

five causes of knee hyperextension

tight PF weak DF proprioception weak hams at hip weak quads

when arm is in IR what is tight and what is stretched

tight- pec major and lats stretched- tires minor and infra

chronic compartment syndrome

tightness, pain, burning, numbness with activity tense compartment

zone of partial preservation

used only when complete injuries, partial preservation of motor or sensory fxn below the neurologic level of injury

T/F: Pts w/ a C1-C4 lesion need maximal assistance for respiratory hygiene.

T

When preparing to stand a pt in the ICU, make sure the ___ are stable & the ___ are long enough for standing.

Vitals; lines

DD for chondromalacia patella or anterior knee pain

Waldron's Bounce home

The severity of the initial deficit is ___ proportional to the prognosis for stroke recovery.

Inversely

what is hip OA associated with and what does it involve

Involves joint degeneration in response to overuse, obesity, childhood issues such as developmental dysplasia, LCP, SCFE, problems arising from adulthood including chronic movement impairment syndromes, trauma, infection, etc.

TIAs, thromboses, embolisms, & lacunar strokes are all types of (hemorrhagic/ischemic?) stroke.

Ischemic

___ strokes occur when arterial supply to the brain is blocked, usually by the narrowing of arteries or by clots moving from the heart (heart attack, AFib).

Ischemic

Drugs aimed at preventing stroke usually work to prevent (hemorrhagic/ischemic?) stroke.

Ischemic (bc usually they are clot-breaking drugs)

(Left/right?) hemisphere lesions are often characterized by anxiety & depression.

Left

The (left/right?) occipital lobe attends to the right visual field.

Left

The (left/right?) parietal lobe does R/L discrimination & praxis.

Left

The (left/right?) temporal lobe does language & interpretation.

Left (in the majority of ppl)

signs and symptoms of Osteochondral Fracture and Osteochondritis Dissecans

Localized tenderness Pain w/ WB Swelling inside joint capsule / hemarthrosis Floating cartilage causes increased fluid in joint (joint effusion) Fracture can cause leaking of blood into joint capsule with dissecans may have loose bodies, locking, effusion

how is diffuse axonal injury diagnosed

MRI

Metatarsalgia signs and symptoms

Pain over metatarsal heads or at MTP joints

Phase II of ankle sprain treatment

-Goals: Normal AROM, FWB without pain, increased proprioception -Ambulates PWB without Pain -Low level balance training -Low level Theraband strengthening (no pain) -Cardiovascular - 30 minutes of stationary cycling -Core strengthening

Stress fractures symptoms

-Gradual onset over a 2-3 week period -Initially pain with activity relieved by rest -Progress to pain for several hours after activity, may get worse at night -Swelling may occur after activity

Presentation of achilles tendonitis

-Gradual onset of pain -In the early stages pain decreases as exercise continues. As the pathology progresses, pain increases with activity throughout duration of exercise. -Pain in the morning upon waking in the inflammatory stage -Swelling may be present 2-3 cm proximal to the calcaneous -May be insertional tenderness only or more typically 2-6 cm proximal the distal insertion -Decreased PF strength/endurance -ARC test *Tendonopathy = swelling in the tendon will move with the DF *Paratendonitis = swelling does not move with DF *Positive when tenderness occurs 3 cm proximal to the Calcaneus with slight PF *Pain with palpation decreases with active DF

Lateral sprains

-Mechanism: PF-Inv -Exam *Ligaments (ATFL, CF, PTFL *PROM - PF/Inv painful *A/RROM - DF/ever painful *Anterior Drawer **Dimple sign (1st 48hrs)

Abnormal pronation patient history

-Mechanisms of injury: Normal stresses applied too frequently Abnormally high stresses applied at normal frequency -Onset of Pain Increased activity level After prolonged immobilization Site of pain localized to inflamed structure(s) Change in footwear

Metatarsalgia treatment

-Metatarsal pads (proximal to metatarsal head) -Increase flexibility of Achilles -Proper shoe wear -Mobilization & Manipulation (metatarsal whip)

Feiss line

-Point at bottom of medial malleolus, most lateral portion of metatarsal head, navicular tuberosity -Navicular above is high arch -Navicular below is low arch

Grade II ankle sprain treatment

-Possible Cam Walker Boot or Bracing for several weeks -PRICED, rehab as previously described *Proprioceptive training and Eccentric strengthening is essential -Typically able to return to full function in 2-6 weeks -May require brace or taping for sports for 6 months

Sesamoiditis

-Potential causes *Impact *Overpronation *Great toe injury -Treatment *Acute **Reduce inflammation **Reduce stress ***Limit hallices DF ***Metatarsal pad placed proximal to 1st & 2nd metatarsal head

FHL tendonitis

-Presents similar to posterior tibialis -Treat in a similar fashion except limit hallices extension

Interventions for pronated feet

-Reduce stress on inflamed structures -Strengthen the muscles that support the arch -Correct biomechanical dysfunction -Proprioception training -Modalities and soft tissue techniques -Shoe inserts and modifications

Retrocalcaneal bursitis

-Signs and symptoms *Pain anterior to Achilles tendon *Swelling is often present *Two finger squeeze test anterior to Achilles tendon and just superior to its distal insertion *Pain with passive DF -Treatment *PRICED *Heel lift *Potentially open backed shoe

Tibialis anterior tendonitis

-Similar complaints as posterior tibialis except location of pain -Treatment similar to post tibialis with more emphasis on triceps surae flexibility

Most ankle injuries fall into these basic categories

-Sprains -Strains -Contusions -Degenerative conditions -Fractures

Neutral wear pattern on shoe

-Tend to wear most heavily along lateral aspect of heal -Even wear along the remainder of the outsole -No excessive midsole wrinkling or tilt

Ankle fracture

-Tender over the fibula -Often unable to bear weight -Medial tenderness, widened mortise = unstable fracture

Functional tests

-The Ankle Joint Functional Assessment Tool -The Foot Function Index

Normal intracranial pressure (ICP) is b/w ___ mmHg.

0-15

normal ICP

0-15mmHg

who is most likely to sustain a TBI

0-4, 15-19, and over 65

In a completed stroke, the damage has been stable for at least ___.

1 day

What are 4 ways tape is used for someone post-stroke?

1. inhibition of overactive muscle synergies 2. facilitation of underactive muscle synergies 3. optimization of joint alignment 4. offloading of irritable tissue

anterograde amnesia

can't form new memories

locked in syndrome

cannot move anything but the eyes, no communication

garden stage classification terminology type 3

complete fracture with partial displacement (usually addressed via THA)

garden stage classification terminology type 2

complete fracture without displacement (requires surgical fixation)

types of transtibial amputation

conventional osteomyoplastic

verbal 4 glasgow

conversant but confused

plagiocephaly

cranial asymmetry

what is plagiocephaly

cranial asymmetry 80-90% also have CMT

how is plagiocephaly treated

cranial orthosis 3-11 months of age may need surgery

A pt should be repositioned every ___ & regular checks for pressure should be done. a. 15 min b. 30 min c. hour d. 2 hours

d. 2 hours

When assessing bowel & bladder control, a 2 =___. a. complete loss of control b. not tested c. reduced or altered control d. normal control

d. normal control

The ___ lobe synthesizes & integrates visual info, does visual memory, forms visuospatial relationships, & does visual reception. a. temporal b. parietal c. frontal d. occipital

d. occipital

how is the styloid process fractured

direct blow or avulsion

traumatic myositis ossificans

direct blow or muscle tear with bleed out

Which of the following NOT a muscle a person w/ a C7 lesion has control of? a. latissimus dorsi b. pec major c. triceps d. FCR e. FDP & FDS f. extensor digitorum

e. FDP & FDS

Things to work on for injuries at the ___ level: normal hand function and progressive occupational activity & exercise. a. C5 b. C6 c. C7 d. C8 e. T1

e. T1

At Rancho level ___ (purposeful, appropriate), they are consistently oriented to person, place, & time; can attend to familiar tasks in a distracting environment for an hr; can recall & integrate past & recent events; aware of deficits & their impact, but need assistance for corrective action; over/under-estimates abilities; depressed, irritable, argumentative, & self-centered; can recognize & correct socially inappropriate behavior. a. IV b. V c. VI d. VII e. VIII

e. VIII

___ brain herniation occurs when it displaces thru the cerebellar tonsils into the foramen magnum. a. uncal b. central c. cingulate d. transcalvarial e. upward f. tonsillar

f. tonsillar

vascular supply to the knee

femoral popliteal genicular

precautions for super path dislocation

femoral head removed in situ avoid abnormal or excessive positions

FAI Clinical Presentation

groin pain w/ little or no history of trauma

pediatric fractures usually occur where?

growth plates

femoral anterior glide with lateral or medial rotation occurs due to

inadequate posterior glide of the femoral head during hip flexion.

trans metatarsal amputation deficits

increased loss of lever arm and weight-bearing surface

principle: repetition matters

induction of plasticity requires sufficient repetition

principle: intensity matters

induction of plasticity requires sufficient training intensity

rehab considerations for focal articular cartilage injuries

initial NWB progressing to WB

steps in clinical decision making

initial data collection acquire info about patient use skilled observation establish hypothesis/relationship between tasks and impairments determine what additional info is needed

post operative care for DDH if not casted

Abduction wedge or hip abduction orthosis Check ROM but do not adduct past neutral for 3+ weeks Work on active motion and weight bearing as allowed by physician

femoral head deformation (CAM)

Abnormality at the level of the anterior femoral head resulting in a non-spherical head and increased femoral neck/head -acetabular rim contact. More common in young athletic men. Cartilaginous lesions form along either the postero-inferior or superior aspects of the acetabulum, causing stiffness and limiting the range of motion. A bony end-feel is characteristic.

An __ brain injury is a "basket" term that describes any trauma to the brain occurring after birth. In addition to traumatic injury, it also includes damage sustained from disease processes (brain tumors, stroke, infection, substances) or anoxia.

Acquired

secondary functions of menisci

joint lubrication prevent capsule/synovial impingement shock absorbers

broca's aphasia

left prefrontal motor cortex speech is slow pauses often perseverate can use automatic sayings/habits repeating aware of their errors

treatment of chronic ankle instability

leg brace, tape, orthotic, surgery (80% success)

concussions

most common TBI blood vessels can be torn CN damage can occur

slipped capital femoral epiphysis

most common disorder of hip in adolescents Growth plate of proximal femoral physis is weak and becomes femoral neck displaced (typically anteriorly) from normal position

central cord syndrome

most common incomplete spinal cord lesion • Often no disruption to the vertebrae, more of a bruising to the cord • Strains or sprains to the cervical tissue (hyperextension injury) • Could be cervical spondylosis (where bones develop arthritis and pinch/compress the cord

syndromes at the hip are named for the direction of the movement that ____

most consistently causes pain

tests for facial nerve

observe facial droop or asymmetry raise eyebrows, smile, frown, teeth, puff out cheeks corneal reflex

tests for oculomotor nerve

observe for ptosis test extra ocular movements pupillary rxn to light

post-op rehab for FAI (phase I)

phase I- protected weight-bearing 6-8 weeks gentle supine ROM quadruped rock back gentle standing AROM at hip

how to support joints and ligaments and correct posture with tape

place the part into the desired resting position and apply tape with greatest tension in line with the structure you with to support

MOI for ACL injury

plant and cut rapid deceleration hyperextension stiff-leg landing contact

short plantar ligament

plantar calcaneocuboid

pain from iliopsoas bursitis

radiates down the anteromedial side of the thigh to the knee and is increased on extension, adduction, and internal rotation of the hip

medical management of heterotrophic ossifications

radiation forcefull joint manip medication splints ROM positioning pain avoidance

what does the anterior visual pathway do

receive, detect, orient, locate

reconstruction vs repair ACLR

reconstruct- use other tissue repair- use remaining ACL

ASIA E

recovered

secondary response with diffuse axonal injury

release of WBC which can cause further damage

MOI for patellar or quad tendonopathy

repetitive eccentric loading rapid change in training improper mechanics poor base strength of quads

MOI for prepatellar bursitis

repetitive friction or pressure wound/infection direct blow

MOI pes anserine bursitis

repetitive use of the semitendinosus, gracilis, and sartorius direct blow to knee

After stroke: GH

sublux inferiorly and anteriorly

DD tests for meniscal tests

thessaly's McMurray test Apley's

which zone of the meniscus is least likely to heal by itself

white zone

weak serratus =

winging and tipping

When using an intraventricular catheter, what determines whether CSF will flow out & how much?

The height of the bag relative to the pt's head

What does Battle's sign look like?

Blood collects behind the ears causes bruising (sign of a basilar skull fx)

Lateral longitudinal arch

Bony architecture: calcaneous, cuboid, metatarsal

non-muscular causes of torticollis

Cervical skeletal malformation Subluxation of cervical vertebrae Herniated disk Ocular strabismus or nystagmus Extra ocular muscle paresis Gastroesophygeal reflux Clavicle fracture and brachial plexus injury from difficult delivery

effects of aging on ligaments

Decrease in mass, stiffness, strength, and viscosity (think dry spaghetti to cooked spaghetti) Ligament & bony insertion weakens Biochemical & mechanical properties decline

interventions for collateral ligament injury (MCL and LCL)

Decrease swelling/pain Early ROM WB advanced as tolerated Avoid valgus/varus stresses Biomechanical considerations Strengthening Balance & Motor control

What is one thing you can tell your pt to do if they have problems w/ hypohydrosis in the Oklahoma summer?

Carry a mister bottle all the time to help cool themselves off

timelines for serial casting

Cast removed and replaced every 1-2 weeks Continue until desired range is obtained, typically 4-6 weeks Results can last from 8 weeks to 12-18 months

How are areflexic bladders managed?

Catheters

How are reflexic bladders managed?

Catheters

congenital scoliosis

Caused by anomalous vertebral development Environmental factors affect spinal development at 45-60 days after fertilization Often becomes stable and does not progress

non-muscular causes of CMT

Cervical skeletal malformation Subluxation of cervical vertebrae Herniated disk Ocular strabismus or nystagmus Extra ocular muscle paresis Gastroesophygeal reflux Clavicle fracture and brachial plexus injury from difficult delivery

When moving an acute care pt around, there will be an increase in ___, which creates an increase in phlegm & oral secretions during therapy.

Coughing

What is one potential negative effect of using e-stim post-stroke?

Could cause spasticity in the mm

clinical tests for LE bone development at the hip

Craig's test Hip joint ROM

effects of aging on tendons

Crimp decreases (tendon lengthens, loses rebound, less recovery) Weaker & Stiffer Decrease in cellular & vascular Rapid force or releases more likely to cause damage

signs of compartment syndrome

Critical pressure increase within a compartment leading to a decline in perfusion to the tissue within that compartment Pain within a compartment at the same time, distance, or intensity of exercise/activity (ex: within 3 minutes of running I get a 5/10 pain) Pain increases with continued activity until patient must cease activity Pain resolves with rest

A visual field ___ can occur w/ lesions to either side of the brain in either the temporal field or nasal field & upper & lower quadrants.

Cut

complications of ACLR

Cyclops lesion Hypertrophic scarring, decreases ability of ACL to slide through tunnel Doesn't occur right away (6 weeks out) Re-tear- same knee or opposite, 10-15% Infection Osteoarthritis Associated with decreased ROM

pre-op treatment of ACL tears

Decreased effusion/inflammation Full ROM (focus on extension) Normalize Gait Increase Quad activation

Boyd amputation

Variation of Symes where calcaneus is transected and placed below tibia and fibula; complete loss of foot functions

___ accounts for 70% of all sensory receptors.

Vision

The greater the ___ issues a pt has, the more impaired their grip.

Visual

The clinical prognosis of a conus medularis lesion is: a. very poor b. poor c. fair d. good e. very good

a. very poor

Which of the following is ok to do w/ a pt who has a high ICP? a. wound care b. raise their legs c. position the pt in side-lying d. move their neck often

a. wound care

posterior tilt short muscles

abdominals and glutes

cranial nerve VI test

abducen extraocular movements

key findings for femoral anterior glide syndrome with lateral rotaiton

abnormal motion with: postural deviations SLS- femur rotates laterally supine hip flexion and ER- stiffness Supine SLR- femur head anterior sidelying ER prone knee flexion- lateral femoral rotation prone hip extension sitting hip rotation quadruped rock-back (<90 hip flex) swayback posture ER of LEs less developed glute max lumbar flexibiliity > hip flexibility short stiff hamstrings

problems with forefoot varus

abnormal pronation retards mid/terminal stance supination

deformities of the skeleton can lead to abnormalities in ___ and decreased _____

abnormalities in movement and decreased participation in functional activities

what is ischemia

an area of infarct surrounded by an area of moderate blood flow

what is a bursa

an expansion of synovial membrane that is found at sites of potential friction, typically between tendon and bone

ASIA L4

ankle DF

muscles needed midswing

ankle DF concentric everything else is inertia

ASIA S1

ankle plantar flexors

what artery: LE involvement more

anterior

what artery: grasp reflex and sucking reflex

anterior

what artery: lack of spontaneous behavior, motor inattention, perseveration, amnesia

anterior

what artery: muteness

anterior

what artery: urinary incontinence

anterior

three ligaments of distal tibiofibular joint

anterior and posterior tibiofibular and interosseous ligs

gait apraxia with flexor withdrawal

anterior cerebral

muteness, perseveration, amnesia

anterior cerebral

what artery: gait apraxia with flexor withdrawal

anterior cerebral

PCL injury MOI

anterior force with knee flexed fall with ankle plantarflexed hyperflexion severe hyper-extension blow to tibia

SLR axis shifts forward for ____

anterior glide syndromes multidirection hypermobility

the menisci shift ____ on the tibia in extension and ____ in flexion

anterior in extension posterior in flexion

with dorsi, roll and glide

anterior in open packed

signs and symptoms for synovial plica

anterior knee pain tenderness over medial femoral condyle clicking, popping, locking

four parts of the deltoid ligament

anterior tibiotalar, posterior tibiotalar, tibiocalcaneal, and tibionavicular

GFRV of gastroc- terminal

anterior to ankle, DF moment

GFRV of gastroc- midstance

anterior to ankle, use eccentric PF

GFRV glute max and hamstrings- loading

anterior to hip joint, use isometric extension

visual receptive components

anterior visual pathway retina, optic nerve, optic chasm, lateral geniculate

GFVRV of gastroc- preswing

anterior, causing DF, use PF

meniscofemoral ligaments

anterior- ligament of Humphrey posterior- ligament of Wrisberg

two functional units of the ACL bundles

anteromedial- taut in flexion, resistance anterior tibial translation posterolateral- taut in extension

medications required for medical management of BI

anti-seizure diuretics (reduce swelling) hyper osmotic agents (reduce swelling) anti-hypertensives (decrease cerebral vasospasm)

(Decorticate/Decerebrate?) posturing is due to damage to the internal capsule or cerebral hemispheres causing damage to the corticospinal pathways.

Decorticate

In (decorticate/ decerebrate?) posturing, UE adduction, IR, pronation, w/ elbow/wrist & finger flexion; LE extension, adduction, & IR, & ankle PF.

Decorticate

femoral anterior glide syndrome with medial rotation

Femoral head glides forward with hip movements, particularly flexion TFL(typically stiff) dominates over lateral rotators weak iliopsoas

femoral anterior glide syndrome with lateral rotation

Femoral head glides forward with hip movements, particularly flexion Lateral rotators (typically stiff) dominate over medial rotators

femoral posterior glide with IR

Femoral head is relatively hypermobile and glides posteriorly with hip movements, particularly flexion Posterior joint capsule is lax

types of surgical procedures for hip subluxation or dislocation

Femoral varus rotational osteotomy (VRO) Acetabuloplasty Innominate (pelvic) osteotomy Femoral head resection

red flags for infection

Fever Chills Night sweats Nausea or Vomiting Recent history of cold or flu-like symptoms Open wound or recent open trauma (even not in the immediate area) Redness Warmth

What is a problem with a backward trunk lean that keeps most pts from using this compensation?

It increases the flexion moment at the knee, meaning the quads have to work harder (must have strong quads to do this, & most don't)

risk factors for CMT

Large birth weight Male 1st born Breech position Nuchal cord Multiple births Vacuum or forceps used during delivery Maternal uterine abnormalities

risk factors of torticollis

Large birth weight Male 1st born Breech position Nuchal cord Multiple births Vacuum or forceps used during delivery Maternal uterine abnormalities

Which indicates a more severe brain injury, a low or high score on the Glasgow Coma Scale?

Low

In terms of splint use in the ICU, you should use (high/low?) load & (prolonged/short?) stretch.

Low; prolonged

___ is a term used for a vegetative state that has lasted more than a month.

Persistent vegetative state

medical intervention for JIA

Pharmaceutical Goal to control arthritis to prevent joint erosion and extra-articular manifestations NSAIDs, methotrexate, biologicals, glucocorticoids Surgical Used if conservative treatment fails Soft tissue release, synovectomy, osteotomy, growth plate stapling, total joint replacement

5 phases of UE stroke recovery

Phase 1- moving the body while the arm is stable (horizontal) Phase 2- moving the body and the hand together (horizontal) Phase 3- moving the hand without the body in a weightbearing position (horizontal) Phase 4- moving the hand without the body in weightbearing (vertical) Phase 5- beginning to move the arm out of weightbearing positions

phase I labral rehab

Phase 1: Muscular Activation and Hip Corrective Strategy: (1) regain normal flexibility, (2) restore corrective strategies with single-leg balance testing, and (3) activation/reactivation with multi-angle isometrics.

phase II labral rehab

Phase 2: Muscular Endurance, Static and Dynamic Postural Control: (1) progress static stabilization to isolate hip corrective strategy required with single-leg balance testing and functional tasks, and (2) improve endurance and control of stabilizing musculature required for sport tasks and address the identified deficit in lateral step-down (loss of pelvic horizontal position and tibial tuberosity medial to second toe).

phase III labral rehab

Phase 3: Return to Running and Skill: (1) initiate dynamic postural control activities necessary for sport activities and return-to-sport testing, such as Y balance; (2) engage in aerobic activities and closed motor skills required for running hurdles; and (3) improve tissue tolerance for return to functional activities.

phase IV labral rehab

Phase 4: Return to Sport and Plyometric Training: (1) progress adaptive postural control activities to maintain/improve the neuromuscular system for kinesthetic awareness and hip corrective strategies, (2) incorporate open motor skills in dynamic environments required for sport participation, and (3) develop muscle power and control required for enhanced sport performance and return-to-sport hop testing.

medial meniscus has firm bond to ___

MCL

contact injury with pop

MCL LCL meniscus fracture

Which method gives more/better information on the extent of damage from a stroke, CT or MRI?

MRI

how to diagnose labral tears

MRI arthroscopy

What is the most common cause of SCI?

MVA

standing walking and sitting hip orthosis

Maintains abd. in standing (abd. angle narrows in standing), walking and sitting

Lesions occurring in the retina, optic nerve, optic chiasm or lateral geniculate nucleus of the thalamus affect (perception/reception?) of visual info.

Reception

Wernicke's aphasia is (expressive/receptive?).

Receptive

In every age group, TBI rates are higher for (females/males?).

Males

PNF techniques use a quick stretch in the beginning of the pattern to encourage activity in the ___.

Muscle spindles

Of what do you need to be careful when performing retrograde massage on a pt to reduce their edema?

Must do it gently to avoid collapsing the lymphatic system

precautions for subacromial trauma

NO PROM >90 degrees of shoulder flexion or abduction

with lateral trunk flexion, the ____ side muscles are short

NON-weightbearing side

effects of dry needling

Needling causes mechanical (physical) disruption of the contraction of the local segment of muscle Mechanical stimulation of the hyper-excitable muscle often produces a twitch upon insertion into the trigger point Muscle 're-sets' and relaxes to its typical resting length Needling has shown in animal studies to normalize the chemical environment and diminishes the endplate noise / leakage of acetylcholine

(Negative/Positive?) symptoms are those a pt lacks after a CNS insult.

Negative

As a therapist, you should spend your time focusing on the (negative/positive?) symptoms of stroke.

Negative

Weakness, loss of fine/gross motor coordination, & poor muscle endurance are all (negative/positive?) symptoms of stroke.

Negative

when are nerves most vulnerable

Nerves are most vulnerable when they are relatively immobile or attached to adjacent structures

intervention for flexible flat feet in kids

No intervention recommended for flat feet if they are not symptomatic Shoes with good arch support will not correct flat feet but can reduce wear on shoes Typically developing children should wear soft shoes when learning to walk

During therapy, should you wait to attain proximal control before working on distal function?

No, don't wait or you'll never get there

Is there a test to specifically dx a vegetative state?

No, only repetition of neurobehavioral assessments

If a person has aphasia, are they also cognitively impaired?

No, their problem is communication

Strokes of the MCA in the (dominant/non-dominant?) hemisphere cause hemi-neglect & unawareness of distance & midline.

Non-dominant

Can we affect or fix tone?

Nope

What do you do if the tube falls off of the vent while the pt is upright?

Replace it immediately, call nurse if necessary

what is a TKA

Resection of arthritic articular surfaces followed by resurfacing with metal and polyethylene components"

cranial nerve III test

Oculomotor 6 cardinal gazes, PERRLA (Pupils Equal, Round, Reactive to Light and Accommodation)

Pain in the traps is common from ___ &/or ____, bc often times this is the largest arm muscle that is innervated.

Overuse; shortening

What does the PEAK acronym stand for when teaching a pt how to appropriately position their trunk/pelvis before UE mvmt?

P: pelvis in neutral E: equal weight on both buttocks A: angles (90 degrees) K: knees facing fwd

for a hinged AFO you need ___ control

PF control

After stroke: ankle

PF, supination

quick and prolonged stretch

PNF techniques use a quick stretch at the beginning of the pattern to encourage activity in the muscle spindles.

traction and PNF

PNF uses SLIGHT traction at the beginning of the pattern. PNF advocates that traction through the joints facilitates movement- particularly when done at the beginning of the movement. It is often used in patterns involving movement (arm activities, swing muscles in the leg) like throwing, swing pattern in gait, tennis swing, etc.

treatment of bursitis

PRICED heel lift open backed shoe

treatment for grade I ankle sprain

PRICEDEM brace 10-14 days peroneal strengthening proprioceptive training

phase I of ankle sprain tretment

PRICEDEM limit WB AROM exercises QID core strengthening

what is contraindicated in OI

PROM and MMT

PT assessment for OI

PROM and MMT contraindicated assess AROM/functional ROM

what to avoid with juvenile idiopathic arthritis

PROM in acutely inflamed or unstable joints UE weight bearing, tumbling, contact sports, high impact activities

signs of lateral sprain

PROM into PF/IR painful ARROM into DF/ER painful dimple sign

soto-hall maneuver

PSLR with cervical flexion

braggarts test

PSLR with passive DF

phase II ankle sprain treatment

PWB without pain balance core CV theraband

A TENS unit is an intervention primarily used to minimize ___ for a pt.

Pain

5 P's

Pain Pulse Pallor Paresthesia Paralysis

signs and symptoms of pes anserine bursitis

Pain Tenderness with palpation Localized swelling Gait dysfunction Decreased muscle strength Decreased ROM

signs and symptoms of collateral ligament injury

Pain Along course of ligament Variable joint line laxity Stiffness Gait deviations Swelling- Minimal to moderate (+) valgus/varus stress tests

signs/symptoms of bursitis

Pain anterior to Achilles tendon Swelling is often present Two finger squeeze test anterior to Achilles tendon and just superior to its distal insertion Pain with passive DF

symptoms of hypomobiity with superior glide

Pain in groin or lateral anterior superior hip above the greater trochanter, pain in knee (medial) upon standing after sitting for a longer period of time

common patterns of pain related to problems at hip joint

Pain in the groin Pain deep in the joint Buttock pain Pain in the anterior thigh Pain in the posterior thigh Pain in the medial thigh Knee pain

signs and symptoms of ITBFS

Pain over lateral femoral epicondyle Crepitus Localized swelling Pain with running

signs and symptoms of patellar or quad tendonopathy

Pain with deceleration jumping Quadriceps weakness Palpable tenderness Tendon crepitus

Although radiographs are the most accurate measure of subluxation, what other method do we use that is the 2nd most reliable & much less expensive?

Palpation by finger width

interventions for idiopathic toe walking

Passive and active gastrocnemeus and soleus ROM Ankle dorsiflexion strengthening Carbon fiber shoe inserts Shoe wedge Heel strike feedback (squeaky shoes) Bracing- articulated AFO, night splints Serial casting Surgical release- risk of overlengthening

DTR of lower extremity

Patellar L4 Achilles S1

Lesions occurring in the thalamus, primary visual cortex, or in any of the brain lobes affect (perception/reception?) of visual info.

Perception

What is a PEG tube used for? Is it used for long- or short-term?

Percutaneous endoscopic gastrostomy: feeding tube directly into the stomach thru a hole in the skin; long-term (2-3 weeks on feeding tube)

Higher intracerebral pressure leads to less cerebral ___, which leads to more damage to brain tissue.

Perfusion

PLISSIT model

Permission Limited Information Specific Suggestions Intensive Therapy

Pts experiencing brain injury are usually positioned in (trendelenberg/reverse trendelenberg?) to help relieve cerebral pressure.

Reverse trendelenberg (head of bed is elevated)

(Left/right?) hemisphere lesions are often characterized by denial, indifference rxns, emotional lability, & euphoria.

Right

A pt is likely to have some sort of agnosia if there is a (left/right?)-sided stroke.

Right

The (left/right?) occipital lobe attends to both the left & right visual fields (it's a back up system).

Right

The (left/right?) parietal lobe does visuospatial orientation.

Right

The (left/right?) temporal lobe does sounds, rhythm, & music, visual performance, & affective expression.

Right

effects of aging on muscles

Senescence sarcopenia 20-25% loss Decrease in muscle volume Type IIa affected greater than Type I Age-related muscle fiber loss can be reversed in some with proper training

The MANDT, PART-2000, and CAPE are tools used during the IV - V Rancho levels to ID possible trigger behaviors. They are used to increase the awareness of the ___'s physical & psychological self-reactions.

Staff

three primary purposes of the pelvis

Support organs involved with bowel, bladder or reproductive functions. Transmit weight of the trunk in sitting or forces through LE in standing Attachment of muscles for the LE and the trunk

___ subluxation is caused when pts have weakness in the infraspinatus, subscapularis, & teres minor & they activate the deltoid for initial abduction.

Superior

For inferior subluxations, the e-stim electrodes should be primarily placed over what 2 mm?

Supraspinatus & posterior deltoid

medical intervention for OI

Surgical intervention to fix fractures or correct scoliosis Intravenous Pamidronate for improved bone density

key exam findings for acetabular impingement

Symptoms aggravated with the hip in a flexed, internally rotated and adducted position impaired single-leg balance with eyes Muscular imbalances with hip weakness, predominantly into hip ER, and tight TFL and VL Positive response to intra-articular injection

T/F: According to the evidence, resistance to PNF patterns on one side increases the torque production on the opposite side.

T

T/F: Amnesia is a predictive factor for recovery.

T

T/F: Bc most mm move joints in 3 planes of mvmt simultaneously, PNF engages in diagonal patterns to allow strengthening throughout the entire ROM.

T

T/F: Immediate cryotherapy for the spinal cord can decrease the amount of secondary damage from hemorrahge, edema, ischemia, or hypoxia.

T

T/F: It is sometimes difficult to determine the exact stage of an individual on the Rancho scale, so it is used as a convenient guideline.

T

T/F: Nerve roots are often damaged as they exit the foramen at the site of the lesion, but return to function w/in 6 months, which can make it look like a pt has an incomplete injury.

T

T/F: Ppl w/ a C6 lesion still have no function in thoracic mm required for breathing & thus forced expiration is difficult.

T

T/F: Ppl w/ a T1-T6 lesion have full hand function & as levels progress, progressive trunk function.

T

T/F: Ppl w/ a level C7 injury have full elbow & wrist extention, but lack wrist flexion.

T

T/F: Pts w/ C1-C4 level injuries wear hand splints (usually at night) to prevent contractures, minimize pain, maintain ROM, & help w/ hygiene.

T

T/F: The center of the spinal cord is the most compromised bc it's the most vascular.

T

T/F: The incidence of carpal tunnel syndrome increases w/ time for pts w/ SCI.

T

T/F: When documenting the Rancho scale for an ICU pt, combining two levels (i.e. 3-4) is acceptable when describing a pt who demonstrates more than one clear level.

T

T/F: Where there is not a myotome test (i.e. only sensory is tested), motor & sensory function are assumed to be the same.

T

T/F: Pts w/ cauda equina lesions often will not regain calf/foot intrinsic strength & may need orthotics.

T (due to degeneration of the myoneural junction over the time the nerve requires to regenerate)

Strengths that a pt w/ a __ level SCI has include lumbricals, opponens & interossei function, but one challenge is that these mm may be weak.

T1

dorsal and palmar interossei are innervated at what level

T1

lumbricales are strong at what level

T1

What level is considered the "sensory level"

T2-L1

Autonomic dysreflexia (or hyperreflexia) only occurs in SCI pts w/ a lesion at ___ or above. Why?

T6 bc that is the area where there is sympathetic, but not parasympathetic influence

What are The Shoulder Rules?

T: thoracic spine S: scapula on thorax R: rotator cuff

what is included in acquired brain injury

TBI, tumor, stroke, infection, substance abuse, anoxia

DD for calcaneofibular

Talar tilt or inversion stress test

How should you talk to a pt who has a TBI & may not be fully awake or attentive?

Talk to them as if they are awake & understand everything you say

multi-disciplinary service delivery model

Team members conduct separate assessments Parents meet with team members individually Team members develop separate disciplinary goals Team members implement their separate discipline specific plans Team members are equal stake holders and recognize the importance of each other.

lower pole fracture of knee

avulsion fracture of patellar tendon

W/ ___, the pt gets ipsilateral flaccid paralysis & sensation loss at the level of the lesion. a. central cord syndrome b. anterior cord syndrome c. Brown Sequard syndrome d. conus medularis lesion

c. Brown Sequard syndrome

Literature suggests preserving tenodesis for ___ post-injury to be sure you don't miss residual function in pts w/ SCI. a. 3 months b. 6 months c. 12 months d. 18 months

c. 12 months

When using an intraventricular catheter, the NIH recommends that pts have the head of bed elevated to b/w ___ degrees. a. 10-15 b. 20-30 c. 30-45 d. 40-50

c. 30-45

Methylprednisolone or GM-1 can enhance the flow of blood to injured cords, but are only effective if given <___ hours after the injury. a. 2 b. 5 c. 8 d. 12

c. 8

Mrs. C speaks slowly & has anomia. She becomes frustrated as she tries to express herself. This is indicative of ___, caused by cortical damage to the ___ lobe(s). a. global aphasia; temporal & parietal b. Wernicke's; temporal c. Broca's; frontal d. dysarthria; motor strip of the frontal

c. Broca's; frontal

Below the level of the lesion, ___ leads to ipsilatateral loss of motor & posterior column function & spastic paralysis; contralateral loss of pain & temp sensation several levels below the level of injury. a. central cord syndrome b. anterior cord syndrome c. Brown Sequard syndrome d. conus medularis lesion

c. Brown Sequard syndrome

W/ ___, half of the spinal cord is damaged. a. central cord syndrome b. anterior cord syndrome c. Brown Sequard syndrome d. conus medularis lesion

c. Brown Sequard syndrome

Using the ASIA scale, at level C7, you test ___. a. elbow flexion b. wrist extension c. elbow extension d. finger flexion e. finger abductors

c. elbow extension

What deviation during initial foot contact in gait is used to compensate for quad weakness? a. hard heel strike b. increased supination c. flat-foot contact d. toe walking

c. flat-foot contact

The ___ lobe contains the motor areas, executive cognitive skills, personality & emotions, & expressive speech. a. temporal b. parietal c. frontal d. occipital

c. frontal

femoral anterior glide should be distinguished from ___ in quadruped rocking back by ____

cam deformity- doesn't allow beyond 90 degrees of hip flexion w/ bony end-feel

what is diffuse axonal injury

can occur with any type of TBI movement of the brain is slower than the movement of the skull brain structures tear (extensive tearing of white matter tracts)

retrograde amnesia

can't recall events that occurred immediately previous to head injury

the medial longitudinal arch is good at handling what kinds of forces

compressive

development of bones in foot is influenced by

compressive forces muscle pulls

contraindications and precautions for estim

con- pacemaker, active cancer precaution- abnormal sensation, pregnancy

rancho level IV

confused agitated, max assist alert, heightened activity. mood swings, incoherent verbalizations, aggressive behavior, absent short term memory

rancho level VI

confused, appropriate mod assist inconsistently oriented x3, attend to highly familiar tasks up to 30 min w mod A, assistive memory aide, emerging awareness of appropriate response, unaware of impairments

rancho level V

confused, inappropriate non agitated, max assist not oriented to person place or time, non-purposeful sustained attn, unable to learn new info, may be able to perform learned tasks

ranchos level IV

confused-agitated

Ranchos level VI

confused-appropriate

Ranchos level V

confused-inappropriate

physical exam for abnormal pronation may show ____

decreased medial arch AB forefoot hallux valgus rearfoot valgus internal tibial torsion femoral antetorsion genu valgus

what does a mechanical ventilator do

deliver oxygen enriched air to body and remove CO2

Barlow's maneuver

demonstrates an unstable or dislocated hip

everything the right brain controls

denial indifference euphoria visuospatial orientation sounds rhythms both visual fields problem solving safety concrete thinking time orientation neglect syndrome memory for procedures awareness left limbs global orientation perceive environment as whole grouping objects arousal and attention shifting attention

anosognosia

denial or lack of awareness of paralysis

Ischemia-reperfusion injury

depletion of intracellular energy stores generate toxic oxygen radicals excess fluid formation

If a pt had previous deficits from SCI, but is now normal, they score an ASIA level of: a. A b. B c. C d. D e. E

e. E

Syndesmosis sprain

eversion force. If the tibia and fibula spread on the talus, the ankle mortise is disrupted and the ankle can become very unstable. associated fibula fracture with an eversion mechanism.

test for deltoid ligament

eversion stress test

tears of the deltoid ligament usually happen with what injury

evulsion of medial malleoli

interventions for OA

exercise (OKC, aquatics, progressive walking) manual therapy motor control/balance TENS maybe: tape orthotic brace needling supplements

decerebrate posture UE

ext, AD, IR, elbow extension, hyperpronation, wrist and finger flexion

decerebrate posture LE

ext, AD, IR, knee extension, ankle PF and IR

motor 2 glasgow

extend arm

in LE ____ comes first (flexion or extension)

extension

decorticate posture LE

extension, AD and IR hip, knee ext, ankle PF

at less than 30 degrees the ITB is a _____

extensor

tests for syndesmotic injury

external rotation stress squeeze point heel thump crossed-leg

test for abducens nerve

extra ocular movements

test for trochlear nerve

extra ocular movements

ALL ligament

extracapsular inserts between Gerdy's and fibular head

eyes 4 glasgow

eyes open spontaneously

eyes 2 glasgow

eyes open to painful stimulus

This is the setting w/ maximum independence in all aspects of life & the goal is community re-entry. Comprehensive assessments center on activities, responsibilities, & roles. a. acute care b. LTAC c. SNF d. IP rehab e. home health f. OP

f. OP

Paraplegia is a result of a lesion in the ___ spinal cord. a. cervical b. thoracic c. lumbar d. sacral e. a - c f. b - d

f. b - d

cranial nerve VII test

facial raise eyebrows close eyes smile frown show teeth puff out cheeks corneal reflex

____ enhances the response of a neuron to a stimulus following stimulation

facilitation

test for PINCER

faebere sign (strongly positive)

principle: use it or lose it

failure to drive specific brain functions can lead to functional degradation

historical clues: PCL

fall on flexed knee acute swelling

95% of hip fractures result from

falling sideways

DDH is more common in

females

4 bones involved with the knee

femur tibia fibula patella

explain the tibiofibular joint

fibula is attached to tibia proximally this joint along with ligaments of the talus, calcaneus, and tibia prevent motion between the fibular and tibia

muscles than plantar flex and evert

fibulas longus and brevis

lateral LE compartment

fibulas longus and brevis superficial peroneal nerve

ASIA T1

finger abductors

ASIA C8

finger flexors

what is index minus

first metatarsal shorter than second

what is index plus

first toe metatarsal is longer than the second

____ times your body weight on talocrural joint during heel strike

five

Pes planus

flatfoot, pronated foot (flexible if arch increases when on toes)

what is a facet

flattened area covered in cartilage on patella

motor 3 glasgow

flex arm

After stroke: neck

flex toward and rotate away from involved side

AFO indicaitons

flexible deformity severe weakness of ankle or possibly knee musculature ankle requires stabilization genu recurvatum

the meniscus takes on more force in _____

flexion

five steps of spontaneous stroke recovery

functional/adaptive recovery recovery of post-stroke edema repercussion of ischemia recovery of diaschisis CNS reorganization

over time the sensory component of peripheral nerves also becomes altered, affecting ____

further conduction of nervous impulses

levator and rhomboids short =

further elevation and downward rotation

superficial posterior LE compartment

gastroc plantaris soleus sural n

muscles active in loading

hip extensors isometric knee enters eccentric ankle DF eccentric hip AB isometric TFL supinators eccentric

muscles active during heelstrike

hip extensors isometric knee flexors to prevent hyperextension ankle dorsiflexors eccentric

ROM needed initial swing

hip flex 20 knee flex 60 dorsi ?

ROM needed for terminal swing

hip flex 30 knee 0 ankle dorsi to neutral

ROM needed for loading

hip flex 30 knee flex 15 ankle PF 15

ROM needed midswing

hip flex 30 knee flex 30 ankle dorsi ?

ROM needed in midstance

hip flex 5 knee flex 5 ankle dorsi flex 5

muscles need initial swing

hip flex concentric (ilio at normal speed, rectus at fast) knee rectus when fast, sartorial/gracillis/BF when slow ankle DF concentric

inability to ER humerus avoid impingement

infraspinatus teres minor

inability to downward glide

infraspinatus teres minor subscapularis

what causes LE weakness after stroke

insufficient recruitment of motor neurons from cortical/subcortical areas

treatment for avulsion fracture

internal fixation prevent contractions if muscular cause active rehab after cast removed

fixation of tibial tubercle fracture

internal fixation in knee extension

treatment of intertrochanteric fracture

internal fixation via pin and plate frequently includes the use of intramedullary rods that run through the center of the bone to stabilize the fracture fragments. Failure of the fixation device and mal-union are the most common complications. Weight bearing status = touch down weight bearing.

MOI of osteochondral fracture and osteochondritis dissecans

internal- plant and cut; intercondylar eminence impacts posterolateral corner of medial femoral condyle external- direct blow; fall with flexed knee

how are fractures of the shaft fixated

internally via intramedullary (IM) rodding, or via external fixators, such as Hoffman devices

what is perception

interpretation of sensation

explain contusion

intra or inter muscular direct blow

treatment of subtrochanteric fracture

intramedullary rod or plates, screws, or nails (Zickel nail). The person is routinely non weightbearing or toe-touch weight bearing for 4-6 weeks. **Straight leg raises are absolutely contraindicated.

Plantarflexed first ray

intrinsic deformity of the first ray in plantarflexion

conduction aphasia

involvement of arcuate fasciculus difficulty repeating can't select correct words circumlocution

slow reversal

involves a slow dynamic contraction of the antagonist followed smoothly by a slow dynamic contraction of the agonist. Example - using the biceps, followed by using the triceps Why?? Many of our functional activities involve rhythmic patterns (walking, running, sawing wood, etc.)

subtrochanteric fractures

least common most unstable most difficult to treat direct forceful trauma most frequent in younger population

three levels of BI

mild, moderate and severe

stroke sites: anterior cerebral

o Contralateral paralysis and sensory loss (lower more than upper) o Gait apraxia with flexor withdrawal o Mutism- muteness o Urinary incontinence o Grasp reflex (worse with soft objects), sucking reflex o Lack of spontaneous behavior, motor inattention, perseveration, and amnesia

stroke sites: posterior cerebral

o Cortical blindness o Memory deficit o Ataxia o Contralateral hemiparesis o Involvement of thalamus result in contralateral sensory loss

People at S1-3 may ambulate fully at the community level but they will need what devices

one or 2 canes and AFO until PF are innervated at S2

clinical findings of hip OA

onset of sx is usually insidious and gradual. The patient typically feels pain in the groin, and as it progresses, into the anterior thigh and knee. Morning stiffness more common and decreases as they begin to move. Many patients experience aching pain after moderate activity, such as walking. No consistent capsular pattern exists. If 3 or more planes of movement are restricted, OA is more likely

where are blood clots most common

orbitofrontal cortex anterior temporal lobe posterior portion of the superior temporal gyrus adjacent parietal area

verbal 5 glasgow

oriented

possible motor impairments with TBI

orthopedic complications decreases in strength impaired functional mobility poor coordination balance impairments problems with fine motor and hand function

what are femoral neck fractures associated with

osteoporosis small forces caused by twisting, stumble, fall

three zones of the meniscus

outer- red zone middle- pink zone inner- white zone

red flags for colon cancer

over 50 bowel disturbance unexplained weight loss pain unchanged by position

85-95% of epidural hematoma result from ___

overlying fracture of the skull

posteromedial shin splints

overpronation late stance problems medial tibial stress syndrome

signs and symptoms of prepatellar bursitis

pain swelling warmth decreased ROM

type 4 salter harris fracture

physical fracture plus epiphyseal and metaphyseal

type 2 salter harris

physical fracture that extends through metaphysis, producing a chip fracture of the metaphysis

clinical findings with stress fracture

pinpoint tenderness normal radiographs

clinical findings with avulsion fracture

point tenderness at the avulsion site, therapist may be able to palpate the bone fragment. Active movement and passive stretch of the muscle is painful. Radiograph confirms.

contributing factors to posterior tibial tendonitis

poor flexibility weak muscles alignment abnormalities over pronation

goals of ranchos level I and II

positioing ROM edema chest PT sensory stim 10-15 minute intervals

what artery: ataxia

posterior

what artery: cortical blindness

posterior

what artery: involvement of thalamus results in contralateral sensory loss

posterior

what artery: memory deficit

posterior

classifications of OA

primary/idiopathic secondary (infection, dysplasia, trauma, AVN)

MCL: primary restraint is ____ assist ACL in ____ binds with ____

primary: AB and IR of tibia assist: transverse motion of tibia on femur Binds with: medial capsular ligament

three driving forces of screw home mechanism

shape of medial femoral condyle passive tension of ACL lateral pull of quads (q- angle)

what do you need for effective tenodesis

shortened FDP, FDS, FDL avoid stiffness in MP and IP

what is congenital muscular torticollis

shortening of SCM SCM rotates head to opposite side and laterally flexes to same side Named according to the side of the involved SCM. Often seen with other conditions including hip instability (DDH), plagiocephaly, scoliosis and foot deformities. Righting reaction delay Increased incidence since 1992 "Back to Sleep" campaign

test for the anterior tibiofibular ligament

side-to-side test

phase III ankle sprain treatment

single heel raises treatmill eccentrics planks

a more perpendicular midtarsal joint = ____& ____-

supinated and stiffer

closed packed position of subtalar joint

supination

After stroke: distal arm

supination or pronation wrist flex finger flex thumb AD and opposition

SMO

supramalleolar orthosis low profile design that crosses ankle less invasive trim lines

inability to abduct

supraspinatus

what is impinged

supraspinatus tendon LH of bicep subacromial bursa

Plantar fasciitis differential diagnosis

tarsal tunnel or may be a symptom of a systemic disorder (RA, gout, etc)

contraindicated with scoliosis

trunk rotation

tarsal tunnel syndrome

vague pain in the sole burning or tingling increase with activity, especially standing and walking for long periods. Reduced by rest Numbness and weakness become present as it progresses

MOI for medial collateral ligament injury

valgus force with foot fixed valgus, ext tibial rotation force

differential diagnosis tests for MCL

valgus stress test

MOI for lateral collateral ligament injury

varus force with foot fixed often with other ligaments

DD test for LCL

varus stress

common causes of amputation

vascular trauma infection cancer congenital

foot orthosis

when foot cannot attain neutral can help shim the gap to that fixed position or help foot attain neutral position unload compromised tissue

anoxic brain injury

when the brain receives no oxygen for a period of time

When does osgood-schlatter's resolve?

when tubercle fuses to tibia

where do tracts occur

white matter

test for plantar fasciitis

windlass test

motor 4 glasgow

withdraw from stimulus

ASIA C6

wrist extensors

when do stress fractures usually occur

young active repetitive microtrauma

when is surgery indicated for patellar subluxation/dislocation

younger recurrent continued instability mal-alignment co-morbid injury

predictors of TBI recovery

• Age, physical and mental condition, SES, & social support • Initial Glasgow Coma Scale and at the time of admission predicts disability and recovery • Severity of the injury; type and location of injury; other injuries and complications; time before initiation of rehab • Duration of the coma - longer is worse • Duration of recovery/time spent at different RLA recovery • Length of post-traumatic amnesia

T/F: Ppl w/ a SCI at levels C1-C3 need a ventilator.

T

eyes 3 glasgow

eyes open to voice

The (D1/D2?) PNF pattern is "unsheathing the sword".

D2

muscles that dorsiflex and evert

EDL fibularis tertius

muscles that dorsiflex and invert

EHL tib anterior

SINSS

severity irritability nature stage stability

ranchos level VIII

purposeful-appropriate routine SBA

Hammer toes

-DIP is straight, PIP is flexed, and MTP is extended

At which level is the latissimus dorsi innervated

C5, 6, 7

hip extension ROM of preswing and toe off

10 degrees 0 degrees

severe BI score

8-0

After stroke: hip

ER

DTR knee

L2-4

Estim

prevent atrophy muscle re-ed

multidirectional accessory hypermobility

"Mixture of anterior and posterior glide syndromes" The hip joint is hypermobile **you don't want to see this **has the worst elements of anterior and posterior glide syndromes- depends on the movement

jersey or FDP splinting

"Patients with confirmed or suspected jersey finger should be referred to an orthopedic or hand surgeon for treatment."

signs and symptoms of PCL injury

"Pop" Vague knee pain Giving way or buckling Variable swelling (delayed) Unable to fully extend Pain with prolonged sitting, going up/down stairs/hills, &/or jumping (+) posterior drawer test, posterior sag sign, dial test

healing time for grade 3 strain

50-180 days

ASIA S2 and on

sensory

what is torticollis

shortening of the SCM

At which level is the sternocleidomastoid innervated

C2-3

At which level is the trapezius innervated

C3-4

From what levels is the diaphragm innervated

C3-5

signs and symptoms of ACL injury

"pop" NOT associated with external trauma hemarthrosis restricted movement feeling unstable giving way or buckling + lachman, anterior drawer, and pivot shift

signs and symptoms and hamstring strain

"pop" with sudden sharp pain "tightness" localized swelling bruising location important - rule of thumb -> the higher the strain, the longer the rehab pain with resisted knee flex and passive stretch palpable tenderness potential palpable defect

ligamentum teres

(ligament of the head): attaches to fovea centralis and head of the femur. A weaker ligament, (more important in pediatrics) which becomes taut with adduction and slight flexion, although any IR or ER ROM combined with other motions increase tension.

Posterior tibial tendonitis treatment

*PRICED *Pulsed US -Reduce the stress *Orthotics Immobilization may be necessary Proper foot wear Flexibility Achilles, 1st MTP, joint mob as indicated of the ankle-foot complex Soft Tissue mobilization, Friction massage Strengthening Progress to closed chain & eccentric strengthening Controlled landing for a higher surface Sidestepping and cariocas away from involved LE Proprioception wobble board in both directions

signs of femoral anterior glide

+ anterior glide test pain in groin with hip flexion affected hip is high in quadruped rocking back

tests for achilles rupture

+ thompson palpation and visual Matles test

Talocrural joint

-70% of talus is covered with articular cartilage -Talus is convex in anterior to posterior direction -Concave medial to lateral -has lateral axis that is slightly oblique and allows dorsiflexion and plantar flexion -Closed packed position: Dorsiflexion -Capsular pattern: Plantar flexion loss greater than dorsiflexion -The capsule is thin anteriorly & posteriorly. It is reinforced laterally and medially by collateral ligaments.

Calcaneal Apophysitis (Sever's Disease)

-8-12 year olds -Accounts for about 8% of overuse injuries in children & adolescents -Bone growth is faster than muscle tendons can lengthen results in a traction apophysitis -Direct or microtrauma to the growth center of the posterior calcaneus -Other Possible Factors *hard playing surfaces *shoes - poorly padded *Cleats *poor support *cavus type foot *tight Achilles and or plantar fascia

Ankle sprain percentages

-85% - 95% of all ankle sprains involve some plantar flexion of the ankle and inversion of the foot. *lateral -The remaining 5%-15% consist of a syndesmosis or eversion sprain, which are often the result of an outside force such as being fallen on from the outside.

Stress fractures

-95% of all stress fractures occur in the LE in the athletic population *Training and/or forces exceed repair phase *Muscle imbalances *Tibial & fibular stress fx accounts for > 50%

Hallux valgus

->20 degrees (normally expect 8-20 degrees) -Congruous or Pathological

Lisfranc injury

-A fracture and or dislocation at the tarsometatarsal joint. -Now, they are most often seen in patients after a motor vehicle accident and in mountain bikers who get their feet caught in the pedal clips.

Metatarsalgia contributing factors

-Abnormal foot mechanics *Morton's foot - short 1st metatarsal compared to 2nd metatarsal - may lead to increased weight bearing through 2nd metatarsal head *Hypermobile foot *Pes cavus *Tight Achilles -Improper shoe wear *High heels -Other *Excessive weight gain, neurogenic, vascular, RA

Training variables

-Anthropometric variables *Pes cavus -Lack of regular stretching -Fatigue -Muscular weakness resulting in more than usual eccentric work -Hill running -Fast running -Improper shoes *Spiked shoes

Medial longitudinal arch

-Arches are good at handling compressive forces -Bony architecture of medial longitudinal arch is metatarsal bone, medial and intermediate cuneiform bone, navicular, talus, and calcaneus

Subtalar joint

-Axis: This axis is a single oblique axis *Runs through where navicular and talus hit each other back toward lateral side of calcaneus -Because the joint's axis is oblique, we observe a component of subtalar motion no matter which of the three reference planes (sagittal, frontal, or transverse) that we use as our point of view. -The axis may be a line that connects the points at which the talus contacts the navicular anteriorly and the calcaneus inferiorly. -Get pronation and supination at this joint -Close packed position : Supination

Normal subtalar joint pronation (closed chain)

-Calcaneus everts. -Talus adducts and plantar flexes -Tibia follows talus so that it rotates internally while knee flexes. -STJ pronation causes MTJ axes to become parallel, making foot flexible and adaptable to different surfaces. -STJ pronation functionally shortens the LE, assisting in shock absorption

Grade III ankle sprain treatment

-Casting or bracing are required -Likely will require >6 weeks before return to full function *Proprioceptive training and Eccentric strengthening is essential -May take 1-4 years before symptom free

Cuboid syndrome

-Causes - speculative *Subluxation of cuboid *Abnormal pull of peroneus longus *Some report increased pronation as a factor or symptom -Symptoms *Lateral midfoot pain *Feels like they are walking on pebbles *Unable to run, jump, or cut *Palpable tenderness to plantar aspect of foot over calcaneocuboid joint -Rx *Devo - Ah, whip it good **Some prefer the cuboid squeeze - same end position as the whip, wait for tissues to relax then use thumbs to relocate cuboid *Low-Dye taping to maintain arch for 1-2 weeks *Soft tissue mobilization - peroneals and long dorsiflexors

Chronic ankle instability: treatment

-Conservative 2-3 months *Phase I-IV *Leg brace, taping, or orthotic support -Surgery *Overall success rates for surgery >80%

Phase III ankle sprain treatment

-Criteria: Ambulates FWB no Pain (may still need brace or heel lift) -Unilateral balance training -Progress to single heel raises -Treadmill - progress to fast walking -Eccentrics & Planks

Phase IV

-Criteria: Pain free unilateral heel raises -Jog to run progression -Cutting maneuvers -Sports specific training

Chronic ankle instability: outcome measure

-Cumberland Ankle Instability Tool -Identification of Functional Ankle Instability

Hallux limitus/rigidus

-DF of big toes is limited *Arthritis *Anatomical abnormality - index plus forefoot *Pronation *Trauma -Test: quick scan - standing see if they can just lift their big toe without lifting the others. Should be greater than 10 degrees of DF. -Treatment: rest, PRICED, shoe modification (larger toe box - rocker bottom), joint mobilization, exercise, corticosteroid injection, surgery *Don't forget to include: Sesamoid mobilization, flexor hallicis strengthening, cues to push through hallicis with gait

Abnormal pronation physical exam

-Decreased medial arch -Abduction of forefoot -Hallux valgus -Rearfoot valgus -Rearfoot or forefoot varus -Internal tibial torsion -Femoral antetorsion Genu valgus

Compartment syndrome

-Defined as increased tissue pressure contained in a nonexpansile space -Raised pressure within a closed fascial space reduces capillary perfusion, placing the enclosed structures at risk -Most commonly observed after acute injury or ischemia in upper and lower extremities -Other variants do exist: abdominal, epidural, CHI, and glaucoma -acute and chronic

Rehab post fasciotomy

-Depends on the reason for onset of symptoms (tibial fracture vs exertional) *limited weight bearing on the involved limb, use of an assistive device as needed, *flexibility and range of motion exercises, *weight resistive exercises, *stretches, *soft tissue mobilization, *and modifying certain activities of the patient

Achilles tendonitis

-Dutton states, "most common overuse injury of LE" -5-18% of all running injuries

Under-pronation wear pattern

-Excessive wear along lateral outsole -Excessive wrinkling of lateral midsole -Lateral tilt

Over-pronation wear pattern

-Excessive wear along medial outsole -Excessive wrinkling of the medial midsole -Medial tilt

Hallux valgus

-Exostosis: bony formation -Cannot correct major bunion with manual therapy -Can do manual therapy for minor bunion or after bunionectomy -Joint mobs for adequate dorsiflexion -Exercises to strengthen the abductor hallucis -orthosis

Treatment of achilles tendonitis

-Focus for the patient are the symptoms. -Focus for the therapist are the causes! *Decreased DF, overpronation, etc -Acute *PRICED *Heel wedge *Rest **Immobilization may be required (e.g.., cam boot) *Modalities (pulsed US, iontophoresis) *Prevent increase PF -Rehabilitative phase *Some suggest a 12 week program *Manual therapy *Progress to eccentrics with the knee bent and straight **Heel drop down ex **3 set of 15 reps with body weight TID *Final stage should closed chain challenges and incorporate activity specific ex **Figure 8 running, one-legged hop, carioca, wobble board

Forefoot-strike wear pattern

-Focused outsole wear over mid and forefoot -No visible heel wear -Wrinkling of midsole in mid and forefoot

Chief complaint

-Have patient point to the area with one finger *They may complain of an "ankle sprain" while pointing elsewhere e.g.. 5th metatarsal fracture -What happened? -Which way did it bend? -Could you walk -how much swelling/ecchymosis -When did it happen? -What have you done for it? -Have you sprained it before?

Rear foot varus

-Heel strike is more lateral than usual -STJ pronates beyond normal -medial bunion from excess pressure -hammer toes -heel callus or bump due to pressure of the heel against the shoe -Therapists can eliminate the need for compensation by supporting the deformity using a medial heel wedge or an orthosis with a medial heel wedge.

Turf toe

-Hyperextension sprain typically with a compressive load (less often due to hyperflexion injury) *Varus or valgus stresses may have been imposed *Capsule can be torn *Sesamoid fractures can occur *Cartilage damage -Dutton (2017, p.1150) states that 50% of the athletes may continue to have unresolved symptoms even 5 years later. -Treatment: PRICED, limit DF, motion and resistance as soon as symptoms allow, may need limited WB, could be out of sports participation for up to 6 weeks. Modified shoe. Needs 90 DF before returning to play. Non-athletes need 75 decrees of 1st MTP DF for normal walking.

Lateral ligaments

-Lateral side does not have as many ligaments as medial side -Plantar ligaments -Lateral collateral ligaments- 3 components: posterior and anterior talofibular ligament, calcaneofibular ligament -ATF is intracapsular ligament, other two are extracapsular ligaments

Mechanisms of recurrent ankle sprains

-Ligaments healed in lengthened position -Less tensile strength than normal -Muscular weakness - incomplete rehab -Tibiofibular instability -Hereditary hypermobility -Loss of proprioception -Undiagnosed cuboid subluxation, subtalar instability, or other disorder -Functional instability (loss of ankle kinematics) -Some propose weak core trunk muscles

Stress fractures signs

-Localized tenderness (medial aspect of tibia or 2-3 inches superior to fibular malleolus) -WB may or may not be painful -Percussion sign -US over site - pain continues 1-2 hours after US -May not be visible on radiographs for 2-8 weeks after injury

Inspection and palpation

-Most helpful during the acute phase -Remember your anatomy! -If not, look it up! -Palpate the structures you know *Boney prominences *Ligaments *tendon insertions

Achilles rupture

-Most likely to occur in the middle-aged athlete with peak incidence occurring in the early 40s At push off Sudden DF with FWB during fall Violent DF with jumping from higher surfaces Immediate pain *Difficulty walking *Often an audible "snap" *Swelling *Palpable defect

Plantar fasciitis etiology

-Multifactorial -Decrease PF strength -Obesity -Antetorsion -Decreased DF -Over pronation -Pes cavus -Improper shoe wear -Overtraining -Occupational

Forefoot varus

-Normally you expect 10-12 degrees of varus -Some state that it is the leading cause of abnormal pronation -Abnormal STJ pronation occurs as a result of trying to bring the 1st ray to the ground -retards midstance/terminal stance supination -try to bring first metatarsal to ground so they pronate -correct with medial forefoot wedge

Grade I West Point

-One ligament injured. -Edema and ecchymosis are often present (caused by tearing of the vasculature); the person is able to ambulate with stability. -ATFL is tender with lateral sprains. -60-70% of all ankle sprains involve the ATFL -Anterior drawer test = Sen 73% & Spec 97% -Anterior drawer + pain with palpation + bruising = Sen 100% & Spec 77%

Mechanisms of Achilles tendonitis

-Overpronation -Decreased dorsiflexion -Weak PF -Altered blood supply -Improper training

Grade I ankle sprain treatment

-PRICEDEM -Brace for 10-14 days (stirrup brace or similar) when performing athletic events, average return to full activities in 11 days is expected. -Rehab: peroneal strengthening and proprioceptive training *Eccentric strengthening is essential -Educate that it may take several month to regain full ligamental stability

Phase I of ankle sprain treatment

-PRICEDEM -Temporary Stabilization -Limit weight bearing -AROM exercises 30 reps, QID (pain free) -Core strengthening

Acute presentation of compartment syndrome

-Pain is disproportionate to the injury; progressive; not relieved by morphine; spontaneous at rest; and worsened by passive stretching of the involved muscles -Involved compartment may also be palpably tense, have signs of numbness, pulselessness, partial paralysis, and paleness.

Plantar fasciitis signs and symptoms

-Pain upon initial weight-bearing -Pain at medial calcaneal tubercle -Pain with prolonged weight-bearing or increased activity (e.g., running or stair climbing) -Pain with DF, which is further exacerbated with 1st MTP DF *Great toe DF range is often limited to less than 90 degrees -Rest relieves symptoms

Signs and symptoms of posterior tibial tendonitis

-Painful during or limited heel off -Over pronation -Pes planus -MMT: weak & painful -Pain in 3 locations *Distal to malleolus near navicular *Proximal to malleolus *Musculotendinous origin (Medial Shin Splints)

Muscular support of lateral arch

-Plantar fascia -intrinsic muscles -Fibularis brevis -Fibularis longus -Fibularis tertius tendon (everts and dorsiflexes): 80-95% of pop has this tendon, if you do not have the tendon you are not at a greater risk for ankle sprains or decreased strength

Ankle dislocations with no fractures

-This takes a high degree of trauma and force.

Grade III West Point

-Three ligaments are involved. -Edema and ecchymosis are more diffuse than the previous grades; the person is typically unable to weight bear, the joint is grossly unstable, the foot usually drops and supinates (with lateral ankle sprains). -ATFL, CFL, and PTFL are tender to palpation with lateral ankle sprains. (Lateral Ankle Sprain Grade III = ATFL + CFL + PTFL are injured) -Fractures must be ruled out, especially with the skeletally immature population.

Muscular support of medial arch

-Tibialis anterior -Tibialis posterior -Flexor digitorum longus -Flexor hallucis longus -Other muscles that are important: abductor hallucis, flexor digitorum, interossi, and maybe fibularis longus

Chronic presentation of compartment syndrome

-Tightness, pain, burning, or numbness related to physical activities. -Involved compartment may also be palpably tense, *Less likely to have signs of numbness, pulselessness, weakness, and paleness than acute. -Improvement in symptoms with rest

Grade II West Point

-Two ligaments are injured. -Edema and ecchymosis are more extensive. -The person may or may not be able to complete what they were doing when injured (be it ADL, Work, or Leisure, i.e., basketball player may be able to finish the game). -ATFL and CFL are tender to palpation with lateral sprains. -20% of all ankle sprains involve ATFL & CFL

Plantar fasciitis treatment

-Typically does not resolve quickly -Reduce the pain & inflammation -Decrease the stresses imposed on the tissues *Night splint *Orthotics (i.e., heel wedge) -Strengthening *Muscles that support the arch of the foot *Intrinsic strengthening **Towel curls **Pick-up marbles **toe taps *Don't forget to address proximal areas of weakness (e.g.., Hip Abductors) -Flexibility *Triceps Surae *PF stretching **Rolling foot over a tennis ball **Rolling foot over a frozen plastic bottle *Friction massage at insertion -Training issues *Running constantly around a track in one direction

Shin splints/ tibial stress syndrome

-anterolateral *Pretibial muscles (TA, EHL, EDL) **Hard surfaces **Poor heel cushioning **Imbalance between pretibial and triceps sure *Symptoms typically at HS or swing phase -posteromedial *Primarily related to overpronation *Symptoms exacerbated in late stance *May lead to medial tibial stress syndrome

Closed chain pronation

-calcaneous: everts -talus: adducts and plantar flexes -forefoot: stable

Open chain pronation

-calcaneous: everts -talus: stable -forefoot: abducts and dorsiflexes

Closed chain supination

-calcaneous: inverts -talus: abducts and dorsiflexes -forefoot: stable

Open chain supination

-calcaneous: inverts -talus: stable -forefoot: adducts and plantar flexes

precautions for anterior dislocation

ER stepping back don't cross legs

Calcaneal Apophysitis (Sever's Disease) treatment

-expect return to activity in 2 wks to 2 months -NSAIDS -Reduce activity -Flexibility (as symptoms decrease, increase DF without causing symptoms) -Shoe wear *get out of cleats *shock absorbent heel pads *orthotic or heel stabilizers or heel cup -in resistant cases immobilization for 4 - 6 weeks may be needed

Muscles that do dorsiflexion and eversion

-extensor digitorum longus -peroneus tertius

Muscles that do dorsiflexion and inversion

-extensor hallucis longus -tibialis anterior

Anterior ankle bursitis

-figure skaters, hockey -can also get bursitis near malleoli -use donut pads

Index plus minus

-first metatarsal and second metatarsal are even -squared foot

Index plus

-first metatarsal longer than second metatarsal -Egyptian foot

Index minus

-first metatarsal shorter than second metatarsal -morton's or greek foot

Claw toe

-flexion at DIP and PIP with extension at MTP

Overpronators recommendations

-foot mechanics: excessive inward roll -foot shape: flat foot to low arch -shock absorption in stride: too good -recommended shoes last: straight -recommended shoe type: motion control

Supinators recommendations

-foot mechanics: excessive outward roll -foot shape: medium to high arch -shock absorption in stride: poor -recommended shoes last: curved -recommended shoe type: cushioning

Neutral foot recommendations

-foot mechanics: normal- the lateral of midline of heel through push off over 1-2 MTP -foot shape: neutral -shock absorption in stride: good -recommended shoes last: semi-curved -recommended shoe type: stability provided they don't over pronate

Clinical exam of the ankle

-history -chief complaint -onset -characteristics -past medical history -activity history -functional tests -inspection and palpation -check range of motion -neurovascular status -strength -ligamentous testing -special testing

Pronation leads to

-internal tibial rotation so -the q-angle changes during the gait cycle, with possible effects on patellofemoral mechanics *Q angle increases which could lead to knee or hip problems

Chopart joint

-midtarsal joint or transverse tarsal joint -connection between talus and navicular and calcaneus and cuboid -This joint has an oblique and a longitudinal axis. These axes do not necessarily correspond to talo-navicular or calcaneo-cuboid articulations. -Moreover, the joint is functionally uniaxial, not biaxial. The joint only pronates or supinates.

Medial tibial stress syndrome

-or tibial periostitis -Basically posteromedial shin splints with a more focal and painful periosteal inflammation *Near origin of posterior tibialis & medial soleus -Treatment involves is similar to posterior tibial tendonitis -Differential Dx includes: tibial stress fx, compartment syndrome, posterior tibial tendonitis

Muscles that do plantar flexion and eversion

-peroneus brevis -peroneus longus

Lisfranc joint

-tarso-metatarsal joint -between metatarsals and cuneiform and cuboid -structure: gliding joints -motion: glide

Tendinopathy: update on pathophysiology

-tendinopathy can develop after a single event or a series of subacute injuries that have a cumulative inflammatory effect on the tendon -chronic tendinopathy is a non inflammatory process -chronic cycle of microscopic injury and incomplete healing -stress shielding -suddenly increasing loading after inactivity can cause tendon injury-weekend warrior -tendon changes *thicker but less ability to store energy *morning stiffness or delayed onset of pain may indicate return of inflammation and need to decrease strain *may take 36-78 hours after activity before there is increased collagen deposits -risk factors *increased BMI, LDL *meds: statins and fluroquinolones -other *central sensitization component

Muscles that do plantar flexion and inversion

-tibialis posterior -flexor digitorum -flexor hallucis longus -achilles tendon

Tarsal tunnel syndrome symptoms

-vague pain in the sole -burning or tingling -increase with activity, especially standing and walking for long periods. -Reduced by rest -Numbness and weakness become present as it progresses

Who were the 2 developers of PNF?

1. Elizabeth Kenny 2. Maggie Knott

People who have thrombotic strokes also usually have which 3 comorbidities?

1. HTN 2. diabetes 3. vascular disease

In the sagittal plane during midstance, the GRFV falls (anterior/posterior?) to the: 1. ankle joint, causing a (DF/PF?) moment 2. knee joint, causing a(n) (ext/flex?) moment 3. hip joint, causing a(n) (ext/flex?) moment

1. anterior; DF 2. anterior; extension 3. posterior; extension

In the sagittal plane during terminal stance, the GRFV falls (anterior/posterior?) to the: 1. ankle joint, causing a (DF/PF?) moment 2. knee joint, causing a(n) (ext/flex?) moment 3. hip joint, causing a(n) (ext/flex?) moment

1. anterior; DF 2. anterior; extension 3. posterior; extension

In the sagittal plane during preswing, the GRFV falls (anterior/posterior?) to the: 1. ankle joint, causing a (DF/PF?) moment 2. knee joint, causing a(n) (ext/flex?) moment 3. hip joint, causing a(n) (ext/flex?) moment

1. anterior; DF 2. posterior; flexion 3. posterior; extension

What are the 2 types of bladder function issues after SCI?

1. areflexic 2. reflexic

List some interventions to reduce edema. (6)

1. ask pts to sleep w/ arm in 30 degrees of elevation 2. compression wraps 3. splints 4. activity & exercise 5. heat/cryotherapy 6. retrograde massage

Put the rules in order for naming an SCI: a. determine if the injury is complete/incomplete b. determine sensation for R/L for pain & touch c. determine the neurological level of injury d. determine motor level on R/L

1. b. determine sensation for R/L for pain & touch 2. d. determine motor level on R/L 3. c. determine the neurological level of injury 4. a. determine if the injury is complete/incomplete

Put the following in order of occurrence starting immediately post-stroke & progressing as time passes: a. extension synergy b. flaccidity w/ some hand/elbow flexion c. mix of flexion & extension synergies d. flexion synergy w/ volitional mvmt w/in the synergy only

1. b. flaccidity w/ some hand/elbow flexion 2. d. flexion synergy w/ volitional mvmt w/in the synergy only 3. a. extension synergy 4. c. mix of flexion & extension synergies

three stages of shoulder/hand syndrome

1. diffuse aching, swollen, tender, **shiny dry skin, increase hair/nail growth, severe pain with PROM 2. Marked decreased ROM, Severe and diffuse pain, Brittle nails, Shiny/cold/flaky skin 3. Severe tissue deformity, joint contracture, IRREVERSIBLE

What 3 things do we assess when looking at autonomic function?

1. general autonomic fn 2. lower urinary tract, bowel, & sexual fn 3. urodynamic evaluation

Which 2 muscle groups often get very tight in a pt in a WC?

1. hamstrings 2. heel cords

What are the top 3 causes of death in the US?

1. heart attack 2. cancer 3. stroke

What 2 things do you need to do for pts w/ superior impingement?

1. joint mobs for inferior glide 2. rotator cuff retraining (very hard to get back, some never do)

In the frontal plane, the GRFV falls (lateral/medial?) to the: 1. subtalar joint, causing a (pronation/supination?) moment 2. knee joint, causing a (valgus/varus?) moment 3. hip joint, causing an (abduction/adduction?) moment

1. lateral; pronation (I know, this STILL makes no sense to me) 2. medial; varus 3. medial; adduction

What 2 problems are the primary causes of death for individuals w/ lesions at C5 or above?

1. pneumonia 2. pulmonary embolism

DELETE What are 2 interventions commonly used to minimize ROM restrictions in a flexed wrist/hand?

1. posterior glides 2. hand opening therapy

In the sagittal plane during loading, the GRFV falls (anterior/posterior?) to the: 1. ankle joint, causing a (DF/PF?) moment 2. knee joint, causing a(n) (ext/flex?) moment 3. hip joint, causing a(n) (ext/flex?) moment

1. posterior; PF 2. posterior; flexion 3. anterior; flexion

What 2 major complications of SCI can be aided by abdominal binders?

1. respiratory complications 2. orthostatic hypotension

What are the 4 most common joints where heterotrophic ossification occurs in TBI pts w/ prolonged coma?

1. shoulder 2. elbow 3. hip 4. knee

What are the 3 most common structures that get impinged at the shoulder? (c'mon let's just review)

1. supraspinatus tendon 2. long head of the biceps 3. subacromial bursa

What are the 2 main limitations of using a bottom up approach to eval?

1. time-consuming & costly 2. can lose sight of the functional performance

What % of strokes affect the ACA? The PCA?

10%; 7%

healing time for grade 1 sprain

10-14 days (1-2 weeks)

SNF

100 day extension of hospital

coxa vara

100 degrees or < closer to 90 = shorter the limb

What is the duration of the on time for a sequential compression device? Off time?

11 sec on; 60 sec off

forces during walking reach ____% of body weight

120%

normal inclination adult

125

mild brain injury score

13-15

at ____ degrees of flexion, there is compression of all facets

135 degrees

normal tibial torsion ____ degrees of toe out

15-30 degrees of toe out

What is the recommended ON:OFF ratio for e-stim to prevent fatigue of the mm?

1:5

80% of pts w/ post-traumatic amnesia that lasted < ___ weeks had good recovery.

2

80% of amnesia patients with symptoms lasting less than ___ weeks had good recovery

2 weeks

healing time for grade 1 strain

2-21 days

leg length inequality

2.5 cm or greater difference in leg length Causes include: trauma congenital neuromuscular acquired diseases infections causing physeal growth arrest tumors vascular disorders

ICP between ___ and ___ is usually fatal

20-30mmHg

healing time for grade 2 strain

20-90 days

ICP above ___ is usually treated

20mmHg

healing time for grade 2 sprain

21 days- 5 months (3-20 weeks)

antetorsion of kids

23-26 degrees

Secondary damage to the spinal cord is usually complete w/in __ - __ hours post-trauma.

24 - 72

___ months and older open reduction required

24 months

healing time for exercise induced muscle soreness

24-48 hours

ICP levels b/w ___ mmHg are usually fatal if prolonged, except in which group (who can tolerate higher pressures for longer)?

25-30; children

hip flexion ROM towards end of terminal swing

30 degrees

At levels C5 - C8, vital capacity is ~___% of normal.

30%

neuroplasticity accounts for __% of stroke recovery

30%

healing time for ligament graft

3 months - 28 months

healing time for tendonitis

3-4 weeks

Motor level is defined as the lowest key muscle group that tests a grade of ___ or above, provided that the next highest segment tests a ___.

3; 5

If a pt doesn't reach stage ___ of the Stages of Motor Recovery of the Chedoke McMaster scale, remedial intervention for recovery won't be successful, & a compensatory approach is recommended. (Options are Stages 1 thru 7)

4

healing time for intracapsular ligament injury

4-26 months

eye opening levels Glasgow

4. eyes open spontaneously 3. eyes open to voice 2. eyes open to painful stimulus 1. remain closed

retroversion

5 degrees

post-surgical intervention for scoliosis

5-7 day hospital stay, 2 weeks of limited activity, 4-6 weeks return to school, 3 months light activity (bicycling, swimming, driving), 6 months non contact sports, 1 year return to normal activity Orthotic use after surgery depends on surgical method, stability and position of curve, and physician

verbal response levels of Glasgow

5. oriented 4. conversant, but confused 3. states recognizable words 2. makes unintelligible sounds 1. no response

healing time for tendon lacerations

50-180 days

mean rehab for syndesmotic injury

55 days

___% of body weight during fast walking or jogging

550%

when is open reduction required for hip subluxation or dislocation

6-18 months old with dislocatable or dislocated hips open reduction is usually required

motor levels of Glasgow

6. obeys commands 5.localizes stimulus 4. withdraws from stimulus 3. flexes arm 2. extends arm 1. no response

healing time for grade 3 sprain

7 weeks - 1 year

what is trigger point dry needling

A form of manual therapy in which a small, sterile, fusiform (non-hollow) needle is inserted into the muscle at the trigger point to cause the muscle to contract and then release, improving the flexibility of the muscle and therefore decreasing the symptoms of pain and dysfunction

___% of the surface of the talus is articular cartilage

70%

femoral torsion complete between ___ and ___ years

8 and 16

adult femoral torsion

8-15 degrees

normal anteversion

8-18 degrees

____% body weight during stumble

870%

moderate BI score

9-12

open packed position of the hip

90 degrees of hip flexion Slight abduction Slight ER

medial tibial torsion

<15 degrees toe out

angle of inclination newborn

<150 degrees

LTAC

>100 days coma or ventilator dependent

coxa valga

>125 degrees

Cumberland ankle scores

>28 is no instability <27 increased level of instability

external tibial torsion

>30 degrees toe out

ASIA A

A = Complete. No sensory or motor function is preserved in the sacral segments S4-5. Also record any zone of partial preservation

Type 5 Salter-Harris Fracture

A compression fracture of the growth plate

what is myofascial pain

A myalgic condition characterized by local and referred pain that originates in a myofascial trigger point Compression of an active point produces pain in stereotypical patterns

osteosarcoma

A primary malignant tumor of bone Represents 50% of bone cancers in children in the U.S. Peak incidence with pubertal growth Most frequently at metaphyseal portion of the most rapidly growing bones in adolescents (distal femur, proximal tibia, proximal humerus)

what is a trigger point

A trigger point (TP) is a hyperirritable spot of soft tissue, usually found within a taut band of skeletal muscle that is painful on compression and that can give rise to characteristic referred pain, tenderness, and autonomic phenomena. Trigger points may be active (ie: they are symptomatic with respect to pain, usually causing radiating pain when compressed) or latent (clinically quiet with respect to pain until compressed)

ligament restraining IR of tibia 0-30 degrees flexion

ACL

what ligament guides the screw home rotation of the knee as it approaches terminal extension

ACL

Ligaments of the tibiofemoral joint (6)

ACL PCL MCL LCL ALL arcuate ligament complex

tibial plateau fractures are associated with

ACL injury MCL or LCL injury Meniscus injury Articular Cartilage injury

acute swelling

ACL, PCL, fracture, knee dislocation, patellar dislocation

knee gave out or buckled

ACL, patellar dislocation, instability, joint effusion

decorticate posturing UE

AD, IR, pronation, elbow flexion, wrist flexion, finger flexion

People at L4-5 may ambulate for exercise, in their household, and limited in the community and will need a. AFO b. KAFO c. HKAFO

AFO

People at S1-3 may ambulate fully at the community level a. AFO b. KAFO c. HKAFO

AFO until PF innervated at S2

If a piece of the skull is displaced during a traumatic injury or removed due to extreme swelling, it can be stored b/w the layer of fat & muscle in the ___ to keep it healthy & then it can be surgically replaced later.

Abdomen (holy crap!)

PT following limb lengthening

Active assistive and isometric strengthening Positioning Functional activities training Weight bearing and gait training Pin care

muscle pulls with LE bone development at the hip

Active pull of the muscles while the infant and child are moving influences bony development Abnormal muscle pull or bony torsion

Osgood-Schlatter disease

Activity related pain and swelling at the insertion of patellar tendon on tibial tubercle Seen in children 11-18 years of age More common in boys Typically unilateral Associated with patella alta Child may have acute severe pain with limp or low grade discomfort over period of months brought on by sports Reproducible pain with resisted knee extension

three subtypes of slipped capital femoral epiphysis

Acute: significant trauma causes immediate severe pain and restricted hip abduction and internal rotation Acute-on-chronic: aching of hip, thigh or knee for weeks or months from chronic slip and then a trauma causes greater displacement and acute symptoms 45% knee or lower thigh pain are their initial symptoms Chronic: most common form; child has limp and pain for weeks or months and loss of hip motion

femoral anterior glide syndrome with lateral rotation therapy

Address Postural, Habitual Positioning, and Muscle Imbalances: Thoracic extensors and abdominals are weak Lateral rotators of the femur are short Hamstring muscles dominant over gluteus maximus and likely short/stiff Medial rotators are weak Iliopsoas is weak

potential sources of anterior knee pain

Anterior synovium Retinaculum Fat pad Joint capsule Referred from quadriceps

therapy for femoral anterior glide syndrome with MEDIAL rotaiton

Address Posture and Muscle Imbalances: Thoracic extensors and external obliques are weak Stiff and dominant TFL Weak gluteus medius Weak gluteus maximus Stiff and weak iliopsoas Stiff hamstrings, particularly medial hamstrings Movement Re-education: Improve posterior glide of femur Enhance gluteus maximus activity over hamstrings

Which pts in the ICU will have sequential compression devices on their LEs?

All pts in the ICU (unless they already have a DVT)

acetabula-femoral dislocation

Almost always traumatic in nature and involve injury to other soft tissue structures. Posterior hip dislocations occur with hip flexion, adduction, and internal rotation. Anterior dislocations occur with slight flexion, abduction, and external rotation.

different forms of arthrogryposis multiplex congenita

Amyoplasia (classic)- Anterior horn disorder, fibrous and fatty tissue replaces normal muscle development, symmetric contractures Distal- hands and feet involved but large joints typically spared

two forms of AMC

Amyoplasia (classic)- Anterior horn disorder, fibrous and fatty tissue replaces normal muscle development, symmetric contractures Distal- hands and feet involved but large joints typically spared

Peripheral nn grow at a rate of ~___ per month.

An inch

Ottawa Ankle Rules

Ankle film if: -Bone tenderness at lateral malleolus -Bone tenderness at medial malleolus -Bone tenderness at base of 5th metatarsal -Bone tenderness at navicular bone -Inability to to walk 4 steps after injury and in ER

(Anosognosia/Somatagnosia?) is the denial or lack of awareness of paralysis, particularly a hemiparetic arm.

Anosognosia

What is the difference b/w anoxia & hypoxia?

Anoxia: brain receives no O2 Hypoxia: brain receives some, but not enough O2

The (anterior/posterior?) spinal artery vascularizes 2/3 of the spinal cord.

Anterior

___ subluxation is also common post-stroke in pts w/ a downwardly rotated scapula, esp if they prefer to hold their shoulder in extension.

Anterior

(Anterograde/Retrograde?) amnesia is the last function to return after trauma.

Anterograde (forming new memories)

Osgood-Schlatter's

Apophysitis of the tibial tuberosity (inflammation at the site of tendon insertion) Partial avulsion of patellar tendon off of insertion Around time of growth spurt (puberty) Males more than females Repetitive quad use

activity limitations of torticollis in older child

Asymmetric weight bearing Incomplete development of automatic postural reactions Increased compensatory strategies

torticollis activity limitations in the infant

Asymmetry of early reflexes Neglect of ipsilateral hand and visual field Delayed righting reactions Limited vestibular, proprioceptive & sensorimotor development

Where does a diffuse axonal injury usually occur?

At the gray - white matter junction

pubofemoral ligament

Attaches to the anterior pubic ramus and to the anterior surface of the intertrochanteric fossa. Resists primarily abduction and, to a lesser extent, internal rotation

Juvenile Idiopathic Arthritis

Autoimmune disorder that causes swelling of joint along with heat, limited motion, and/or pain with motion that lasts for more than 6 weeks Onset prior to age 16 Can lead to joint contractures, weakness, postural deviations and pain

___ is when the sympathetic NS overreacts to a stimulus usually below the level of the lesion, so the BP rises uncontrollably.

Autonomic dysreflexia (or hyperreflexia)

SCI disrupts the ___ & ___ input to the GI tract, & thus mvmt thru the bowels is slowed.

Autonomic; somatic

Legg-Calve-Perthes Disease

Avascular necrosis of the femoral head caused by loss of blood supply. Cause of blood supply loss is unknown but often follows repeated episodes of transient synovitis of the hip Occurs in children ages 3-12 and is most common in boys ages 5-7 After age of 8 the accommodation of acetabular head to femoral head may not be possible. Children are typically small for their age, very active and have a high incidence of learning disability Bilateral in 20% of cases

Ballerina fracture

Avulsion of 5th tuberosity "Ballerina" = proximal tip of 5th tuberosity

W/ lesions at the T1-T6 levels, which mm should you allow to be hypomobile?

Back extensors still, until the trunk is strong

___ works as a muscle relaxant by blocking the release of excitatory neurotransmitters in the spinal cord to restore the balance of excitatory/inhibitory input to reduce muscle hyperactivity (spasticity).

Baclofen

Sensory problems post-stroke tend to cause ___ issues.

Balance

In a ___ skull fx, the fx is located at the base of the skull & may include the foramen magnum. Raccoon eyes & Battle's sign are indicators of this type of fx. It is extremely dangerous.

Basilar

Why would using slow reversals be functional?

Bc many functional activities involve rhythmic patterns, like running

Why is constant blood flow to the brain so important?

Bc the brain doesn't store O2 or glucose effectively

Why is shoulder pain such a serious complication for a pt w/ an SCI?

Bc they are total dependent on UEs for ADLs, transfers, & mobility

Why is pulley work of the UE inappropriate for pts post-stroke?

Bc you can get the arm up, but w/o proper scapular mvmt & positioning of the HOH in the glenoid, you'll end up w/ impingement & pain

moving in diagonal patterns

Because most muscles move joints in 3 planes of movement simultaneously, PNF engages these diagonal patterns. This allows strengthening throughout the entire range of motion - from the most shortened to the most elongated position

Thermoregulation problems commonly occur (above/below?) the level of the lesion.

Below

mixed femoral and acetabular deformation

Both internal and external rotation are limited, with a bony end-feel. Always with the characteristic sign: BONE ON BONE, HARD END-FEEL. It's quite unexpected in the young person, and the arthritis associated with hard end-feel that we would expect to find simply isn't seen on x-ray.

The following describes a (bottom up/top down?) approach to a stroke eval: -assess all impairments -helps determine a particular deficit -helps a new therapist to refine observation skills

Bottom up

Thrombotic strokes often occur at places where arteries ___ & where ___ may have narrowed the arteries for years.

Branch; plaques

precautions of dry needling

Breast implants Blood thinners Hepatitis / HIV Heart valve replacement (increased potential for inducing infection) Pace maker Overstimulation from too much needling

At which level are the head and neck extensors innervated

C1-3

At which level is the deltoid innervated

C5

At which level is the infraspinatus innervated

C5

At which level is the teres minor innervated

C5

Name the first level to add muscle control in the elbow

C5

at which level is the supraspinatus innervated

C5

At which level is the serratus anterior innervated

C5, 6, 7

At which level is the pectoralis major innervated

C5, 6, 7, 8

At which level is the *clavicular* portion of the pectoralis major available

C5-6

At which level is the Biceps innervated

C5-6

DTR bicep

C5-6

DTR brachioradialis

C5-6

myotome- flexion of elbow

C5-6 biceps

Injuries at the ___ level are the only ones where ER strength at the GH joint > IR strength. Why?

C5; no pectoralis mm at this point to IR

At which level does tenodesis grip become available

C6

At which level is *full* elbow flexion available

C6

At which level is the *full* rotator cuff available

C6

At which level is the Subscapularis innervated

C6

At which level is the extensor carpi radialis longus (ECRL) innervated

C6

Name the first level to add muscle control in the wrist

C6

the *first* level with a motor and sensory level to allow for the *possibility* of total independence

C6

myotome- wrist extension

C6-C8 radial nerve

myotome- elbow extension

C6-C8 triceps

At which level is the extensor digitorum innervated

C7

At which level is the flexor carpi radialis innervated

C7

When are the scapular upward rotators fully innervated

C7

first *CONSISTENT* potential possibilities of living totally independently without assistance

C7

At which level are the triceps innervated

C7-8

myotome- grip

C7-T1

At which level are the flexor digitorum profundus and superficialis innervated

C8

The extensor carpi ulnaris (ECU) and flexor carpi ulnaris (FCU) are innervated at which level

C8

The flexor carpi radialis is innervated at

C8

myotome- opposition of thumb

C8-T1 median nerve

myotome- finger abduction

C8-T1 ulnar nerve

CAM = limited ____ pincer = limited ___

CAM= limited IR pincer = limited ER

damage to what CNs cause double vision

CN III, IV, VI

Tone problems are always a sign of ___ insult.

CNS

normal subtalar joint pronation in a closed chain

Calcaneus everts. Talus adducts/plantar flexes Tibia follows talus so that it rotates internally while knee flexes. STJ pronation causes MTJ axes to become parallel, making foot flexible and adaptable to different surfaces. STJ pronation functionally shortens the LE, assisting in shock absorption

intervention if gene varum or valgum is problematic

Check for internal tibial torsion, hip angle/ROM, foot progression angle Controversial tx: no treatment, splints/night wear, surgery if no improvement and significant

The ___ is a scale used to quantify the levels of recovery post-stroke.

Chedoke McMaster Stroke Impairment Inventory Scale

assessment of slipped capital femoral epiphysis

Child usually limps and complains of pain in groin often referred to anteromedial aspect of thigh and knee Leg held in external rotation in both standing and supine Decrease hip motion noted in flexion, abduction, and internal rotation Attempts at active hip flexion results in external rotation "Frog" position radiographs

environmental factors of DDH

Children routinely kept in lower extremity extension have higher incidence of DDH than those in abduction and flexion

Heel spurs

Chronic plantar fasciitis can lead to formation of heel spurs. Plantar Fasciitis is the most common injury seen among long distance runners. It is very painful and can be chronic, extending over several years. The heel spur does not cause the plantar fasciitis, the fasciitis causes the heel spur.

In terms of the organization of the lateral spinothalamic & corticospinal tracts, which of the following levels is closest to the center of the cord? Which is farthest away (most lateral)? a. cervical b. thoracic c. lumbar d. sacral

Closest: a. cervical Farthest: d. sacral

symptoms of elevated ICP

Cn palsy headache with nausea mental status change confusion, agitation, lethargy

In addition to eye-hand coordination, adequate reach & grasp, & the ability to manipulate an object, pts must have sufficient ___ to generate & execute a plan, & maintain intention throughout the plan, post-stroke.

Cognition

intervention for RLA levels VI-VIII (cognitive and physical)

Cognitive • Reorient patient as needed • Consistent staff response to confusion • Use familiar objects and real-life tasks to reinforce therapy goals • Allow patient time to respond and self-correct • Begin previous vocational or educational training as soon as possible • Use community-based outings to work on psycho-social and cognitive skills Physical • Improve strength, coordination and endurance through meaningful and purposeful activity • Use physical tasks to decrease frustration

Duration of a ___ for more than 20 days is predictive of poor functional outcome for pts w/ TBI.

Coma

common postures with femoral anterior glide syndrome with medial rotation

Common sustained posture: swayback posture, anything that can produce medial rotation (pelvis on femur or femur on pelvis) Common activities: sitting with femur medially rotated

(Compensation/Remediation?) may be the only option for pts w/ severe impairments & functional losses w/ little expectation for additional recovery.

Compensation

The following are cons for (compensation/remediation?): may suppress some parts of recovery & promote learned non-use of impaired segments; development of splinter skills (not generalizable).

Compensation

The following are pros for (compensation/remediation?): early resumption of functional independence using uninvolved segments; environmental adaptation; practice of new skills.

Compensation

Maintaining a comfortable, pain free, mobile arm & hand, and teaching strategies such as switching hand dominance are part of a (compensatory/restorative?) approach to therapy.

Compensatory

A ___ is an injury resulting in a total absence of sensory & motor function in the lowest sacral levels (S4-S5)

Complete lesion

symes amputation deficits

Complete loss of all foot function; shock absorption at heel contact; balance and all weight bearing surfaces of foot

___ is caused by sympathetic nervous system overflow & it causes hypersensitivity to pain & of mechanoreceptors.

Complex regional pain syndrome (aka reflex sympathetic dystrophy; aka shoulder/hand syndrome)

In a ___ skull fx, the scalp is cut & the skull is fx'd.

Compound

forces that influence bone development in young children

Compressive forces: weight bearing and muscle pulls Asymmetrical forces can result in asymmetrical growth at the epiphyseal plate Shear forces: muscle pulls Results in torsional or twisting forces in bones

About 75% of TBIs that occur are ___ or other forms of mild TBI.

Concussions

Normally, if systemic BP rises, the cerebral vessels will (constrict/dilate?) & if BP falls, the opposite occurs to maintain even perfusion pressure to the brain. This system often fails after TBI.

Constrict

stroke sites: middle cerebral

Contralateral paralysis/sensory loss- UE Hemianopia Limb-kinetic apraxia (mainly L hemisphere) Superior division (left side)- Broca's or expressive aphasia (say one word over and over) Inferior division (left side)- Wernicke's or receptive aphasia (patient doesn't process what people are saying) Non-dominant hemisphere strokes lead to hemi-neglect, unawareness of distance and midline, visual perceptual disorders, and visual neglect

A ___ lesion occurs at the L1-L2 level, just at the bottom of the cord.

Conus medularis

In (decorticate/ decerebrate?) posturing, UE extension, adduction, IR, elbow extension, hyperpronation, & wrist/finger flexion; LE extension, adduction, & IR, & ankle PF; trunk & neck extension.

Decerebrate

GFRV tibialis anterior- swing

DF act concentrically

excessive knee flexion over long periods of time can cause

DVT

Who gets posterior tibial tendonitis

Dancers, joggers, ice skaters, increased BMI, over pronators

(Decorticate/Decerebrate?) posturing is due to damage to the upper midbrain & lower pons, or can be a sign of brain herniation. It is the more serious posture.

Decerebrate

why have a labrum

Deepens the socket Contributes to strong suction seal of femur (neg atmospheric pressure) Prevents first 1-2 mm of distraction Force distributor Shock absorber Elastic nature of tissue better for mobility than osseous barrier

The (deeper/shallower?) the location of the clot in the brain, the less damage it does.

Deeper (bc it's more localized & causes less generalized damage)

___ refers to a state of low reactivity that occurs after injury to the brain in areas of the brain previously stimulated by this portion of the brain (where the neuronal connections lie).

Diaschisis

signs and symptoms of anterior knee pain

Diffuse pain With prolonged sitting, knee flexion, or RROM knee extension Poor mechanics

For remedial interventions, evidence suggests that the results (do/do not?) generalize &/or endure.

Do not

Besides replacing any restraints you removed during therapy, what else must you do if you've removed them?

Document the amt of time the restraints were off

Strokes of the MCA in the (dominant/non-dominant?) hemisphere cause a speech deficit.

Dominant

What is the general rule of thumb when deciding whether to use a humeral support w/ a pt?

Don't use them universally, but eval each pt for appropriateness; use it early as a preventative measure

Most pts in the ICU can't report ___, but may demonstrate fear, anxiety or agitation w/ mvmt or requests to attend visually.

Double vision

The kyphotic & side-bent position of the trunk, exacerbated by the wt of the weak/flaccid arm causes scapular ___ rotation.

Downward

precautions for dislocation lateral

ER ADD past neutral

Cardiac pacemakers, anterior neck & head placement, active cancer, abnormal sensation, & pregnancy are contraindications/ precautions for the use of ___.

E-stim

ACLR rehab considerations

Early WB (isolated ACLR) Early ROM Especially extension Limit early OKC RROM Core & NM Control Progressive interventions related to impairments

When is it most effective to use e-stim in recovery?

Early in recovery for prevention & later for muscle re-education

hamstring strain interventions

Early treatment avoidance of isolated resistance Start integrated and move to isolated motor control within symptom free ROM Lumbopelvic isometrics Begin with functional strengthening/activities in transverse/frontal planes followed by sagittal plane Eccentric activities beginning in mid-range and gradually progressing to end ROM Avoid open chain stretching

Lack of muscle contraction to act as a vascular pump, entrapment/impingement due to postural change, sympathetic nerve response to hemiplegia (RSD), & blood clot/DVT are all potential causes of ___ post-stroke.

Edema

At which joint do you see the main difference b/w the flexion/extension synergies?

Elbow

Sitting up in bed is used to build ___ in ICU pts, even if they only spend 15 min a day in that position.

Endurance

How often does a new order for restraints come in from nursing?

Every 24 hours

Internal intercostals are mm of (expiration/inspiration?) & are innervated from levels ___ to ___.

Expiration; T1-T12

T/F: Female function is impaired as far as child bearing ability after an SCI.

F

T/F: If a muscle is weak bc it's only partially innervated, you shouldn't waste time trying to strengthen it.

F

T/F: The bony levels of injury correspond to the levels of function of SCI.

F

T/F: Cauda equina lesions injure peripheral nn, which leads to spastic paralysis.

F (flaccid paralysis w/o spasticity)

T/F: White matter in the spinal cord is compromised first bc its metabolic & perfusion needs are greater, thus putting it at more risk during periods of compromised circulation.

F (gray matter)

T/F: The spinal cord below the level of injury atrophies & is no longer functional.

F (it is functional, & sensory inputs can cause spastic motor outputs bc it's no longer regulated by the CNS)

T/F: The Zone of Partial Preservation is used only w/ incomplete SCI.

F (only w/ complete)

T/F: At level C8, pts can extend their fingers, but cannot flex them (except the MP joint).

F (they can flex their fingers, but not extend--CAN extend MP)

T/F: A pt w/ a lesion at level C5 cannot employ the side to side shifting method (w/ arm thrown back & hooked over backrest) of independent pressure relief bc it is too difficult.

F They can, but it is indeed very difficult

T/F: Many ppl w/ TBI fully recover cognitive & psychosocial abilities, but never fully recover physical function.

F (do recover physical, don't fully recover cog/psychosocial)

trans-disciplinary service delivery models

Fewest number of service providers to improve the child's abilities conduct the assessment together Parents are equal team members Team members and family develop joint goals and priorities A single service provider implements the plan with the family. Team members work across traditional discipline lines to implement a joint service plan

W/ central cord syndrome, bilateral (flaccid/spastic?) paralysis & sensation loss occur at the level of spinal cord injury.

Flaccid

talipes equinovarus (TEV) (clubfoot)

Forefoot curved medially, hindfoot in varus, equinus of the ankle, small calcaneus Congenital clubfoot caused by restricted positioning in utero Genetic influence is suggested Bilateral in half of the cases Often associated with myelomeningocele or arthrogryposis

What is heterotrophic ossification? It's common in severe brain injury w/ prolonged coma & limb ___.

Formation of bone in soft tissue & peri-articular areas; spasticity

heterotrophic ossification

Formation of bone in soft tissue and peri-articular areas • Common in severe BI with prolonged coma and limb spasticity

Jones fracture

Fracture of the base of the fifth metatarsal, at least 1.5 cm. distal to the metatarsal styloid.

closed pack of hip

Full extension, IR, slight AB

What assessment might you do for a pt when you first see them in the ICU following a TBI?

Glasgow Coma Scale

treatment of slipped capital femoral epiphysis

Goal is to keep displacement to a minimum, maintain motion, and delay or prevent premature degenerative arthritis Surgical fixation used to secure growth plate; refer to orthopedic surgeon if SCFE suspected

torticollis grades

Grade 1- Early mild: 0-6 mos, <15˚ rotation Grade 2- Early moderate: 0-6 mos, 15˚- 30˚ Grade 3- Early severe, 0-6 mos, >30˚ or mass Grade 4- Late mild, 7-9 mos, <15˚ Grade 5- Late moderate, 10-12 mos, <15˚ Grade 6- Late severe, 7-12 mos, >15˚ Grade 7- Late extreme: 7+ mos with mass OR 12+ mos with >30˚

CMT levels of severity

Grade 1- Early mild: 0-6 mos, <15˚ rotation Grade 2- Early moderate: 0-6 mos, 15˚- 30˚ Grade 3- Early severe, 0-6 mos, >30˚ or mass Grade 4- Late mild, 7-9 mos, <15˚ Grade 5- Late moderate, 10-12 mos, <15˚ Grade 6- Late severe, 7-12 mos, >15˚ Grade 7- Late extreme: 7+ mos with mass OR 12+ mos with >30˚

presentation of visceral pattern

Gradual, progressive, cyclical Constant Intense Unrelieved by rest or change of position Does not fit the expected mechanical or neuromusculoskeletal pattern

most common fractures in peds

Greenstick Avulsion LE - ASIS, AIIS, ischial tuberosity, base of 5th metatarsal UE - medial humeral epicondyle and proximal humerus Growth Plate (physeal)

People at T1-9 may ambulate for exercise only and will need a. AFO b. KAFO c. HKAFO

HKAFO

sidelying positioning for uninvolved side down

Head Pillow to allow neutral position LE Bottom (uninvolved): flexed Top (involved): extended w/ 1-2 pillows under UE Scapula neutral rotation and slight AB Slight GH AB Elbow slight flexed

sidelying positioning for involved side down

Head Prevent lateral flexion to involved side LE Bottom leg (involved): flexed Top lef (uninvolved): flexed or extended; pillow between knees to prevent breakdown Trunk Pillow behind back to maintain position if needed UE Scapula slight AB BEFORE rolling onto side

What issue post-stroke has the greatest negative impact on functional recovery & discharge to home?

Hemineglect

___ is when bone lays down in joints or mm & occurs frequently in ppl w/ mid-thoracic & cervical lesions.

Heterotrophic ossification

Signs & symptoms of autonomic dysreflexia (or hyperreflexia) include (high/low?) BP & (high/low?) HR.

High BP; low HR (body trying to compensate for the out of control BP)

What is one coordination problem that is prevalent post-stroke?

High levels of co-activation of agonists & antagonists

A (hinged/solid?) AFO is a compensation for weak DFs.

Hinged

A backward trunk lean is a compensation for weak ___ muscles.

Hip extensor

ASIA L2

Hip flexors (iliopsoas)

post operative care for DDH if casted

Hip spica cast or long leg casts with abduction bar Address joints that aren't casted Monitor skin integrity and cast condition Address hip and knee ROM as soon as cast is removed Increase weight bearing as tolerated

DD for plica

Hughston Plice test

Hyperextension of MTP causes

Hyperextension of the MTP pulls the plantar plate distally leaving the head of metatarsal less protected during weight bearing leading to metatarsalgia pain

A patient has complete absence of observable change in behavior when presented visual, auditory, tactile, proprioceptive, vestibular or painful stimuli. They are likely at level...

I

Cranial nerve palsies, headache w/ nausea/vomiting, mental status changes, & confusion, agitation, or lethargy can all be signs of increased ___.

ICP

A patient who exhibits a reflex response to painful stimuli and responds to auditory, external and other responses often with generalized physiological changes is likely at a level....

II

A patient who is able to follow a moving object that is passing through her visual field but is not necessarily alert is likely at level

III

A patient who responds inconsistently to simple commands and directly related to the type of stimulus, this patient is most likely at level

III

At what point is it most appropriate to begin to introduce basic bed mobility, sitting, standing, and ADL with *total* assist into intervention

III

At which RLA level might your patient begin to exhibit responses to phrases such as "help me to raise your arm" ?

III

Which 3 cranial nerves control the 6 external eye muscles?

III, IV, & VI

ranchos level III

III. Localized Response- demonstrate withdrawal/vocalization to pain, turn toward/away from auditory, blink with strong light, follow moving objects, pull at tubes, inconsistent response

ALL ligament: primary restraint to ____

IR of tibia while in flexion (>30 degrees flexion)

capsular pattern of the hip

IR then hip flexion then some limitation of ABd No limitations with aDD or ER

A patient at level ______ may perform motor activities such as sitting, reaching and walking but will not do so on request.

IV

At which RLA level is a patient who is able to verbalize, but verbalizations are incoherent or inappropriate. Patients at this level are unable to converse on even a social, automatic level for brief periods of time.

IV

Patients at level _____ are alert and in a heightened state of activity. They might make purposeful attempts to remove restraints and tubes. They are at danger for falling out of bed.

IV

Patients may overreact to stimuli by crying or screaming, may exhibit aggressive or flight behavior. Mood swings from euphoria to hostile without cause are most common at level ____

IV

Patients at level _____ are unable to cooperate with treatment, whereas patients at level ______ may be able to perform previously learned tasks when cues and structure are provided

IV; V

Patients at level _______ may have very brief, divided attention and decreased, if any, short term memory. vs. Patients at level ____ have *frequent* brief periods of sustained, nonpusposeful attention

IV; V

Which RLA phase is considered the *testing phase* where patients tend to test their limits

IX

signs and symptoms of a gastroc strain

Immediate pain Loss of function Localized swelling Bruising Potential palpable defect Pain with RROM &/or stretch

when is meniscus surgery indicated

Immediate symptom onset and unable to continue with activity Locking of knee Associated ACL tear Failure of up to 6 weeks of conservative management non-degenerative tear

If you have a pt that hyperextends the knee during gait, how would you set the AFO?

In 5 degrees of DF if they have strong quads; in neutral if they don't have strong quads (more likely)

What is a "blown pupil"?

In a closed head injury, the increased ICP pushes into the eye socket & compresses CN III, making a pt's pupil appear dilated

signs and symptoms of patellar tendon rupture

Inability to extend knee Significant swelling Pain High riding patella Patella alta R/O patellar dislocation

General arm weakness, combined w/ lack of rotator cuff strength can cause ___ subluxation of the GH joint.

Inferior

The __ lig may become stretched/torn, which allows for greater upward mvmt of the humerus & superior impingement.

Inferior GH

problems with ischial bursitis

Inflammation commonly arises as a result of direct trauma, which is typically prolonged sitting on a hard surface/in the same position (spinal cord injury). Pain may radiate down the back of the thigh and mimic sciatic nerve inflammation. pain can be reproduced by pressure over the ischial tuberosity.

osteogenesis imperfecta

Inherited disorder that causes weak muscles, lax joints and diffuse osteoporosis with recurring fractures

patellar dislocation interventions

Initial period of long leg brace Decrease swelling Progressive ROM Flexion Quadriceps strengthening Biomechanical considerations

factors related to prolonged recovery of hamstring strain

Injury involving proximal free tendon Smaller distance between tear and ischial tuberosity Greater cross sectional area of tear

External intercostals are mm of (expiration/inspiration?) & are innervated from levels ___ to ___.

Inspiration; T1-T12

88% of all strokes are (hemorrhagic/ischemic?).

Ischemic

If you got to pick your stroke, it would be better to have a(n) (hemorrhagic/ischemic?) stroke.

Ischemic

Stroke (is/isn't?) a progressive neurological condition.

Isn't

What is the problem w/ putting a support in the axilla w/ a humeral support (2)?

It increases the horizontal pressure, pushing the HOH out of glenoid; can also impact neural & vascular bundle

About how long after a pt enters the flexion synergy will it be before they begin an extension synergy (if they do)?

It's at a random rate; we can't predict this

How can you tell if a forward trunk lean is caused by compensation due to a weak glute max or quad or both?

It's very difficult to tell

People at L1-3 may ambulate for exercise and in their household and will need a. AFO b. KAFO c. HKAFO

KAFO

People at T10-12 may ambulate for exercise and in their household and will need a. AFO b. KAFO c. HKAFO

KAFOs (although some still need HKAFOs)

Post-stroke, pts often have a (kyphotic/lordotic?) spine and lateral flexion (away from/toward?) the weak side.

Kyphotic; toward

LE Myotomes

L1, L2: hip flexion L3-4: knee extension L4: dorsiflexion/ heel walk L5: great toe extension S1: plantarflexion/toe walk S1-2: hip extension S1-2: knee flexion

At what level is the full quadratus lumborum innervated

L1-2

At what level is the iliopsoas innervated?

L1-2

myotome- adduction at the hip

L2-L4 adductors

myotome- hip flexion

L2-L4 iliopsoas

myotome- extension at the knee

L2-L4 quads

What is the first level at which knee extension becomes innervated

L3

At what level are the quadriceps innervated

L3-4

What is the first level os SCI at which functional ambulation is probable

L3-4

What is the first level at which dorsiflexors become innervated

L4

At what level is the tibialis anterior innervated?

L4, 5 S1

Myotome- dorsiflexion at the ankle

L4-5

myotome- abduction at hip

L4-S1 glute med and max

myotome- flexion at knee

L4-S2 hamstings

At what level is the extensor hallucis longus innervated

L5

At what level is the gluteus medius innervated

L5

At what level is the gluteus minimus innervated

L5

At what level are the hamstrings innervated

L5-S1

moderate BI criteria

LOC from minutes to hours confusion for days to weeks moderate disability

What causes inadequate knee flexion in swing phase?

Lack of push off

medial soft-tissue stabilizers of patella

Layer 1:Superficial medial retinaculum, Medial patellotibial ligament Layer 2: Medial patellofemoral ligament, Superficial medial collateral ligament Layer 3:Medial patellar meniscal ligament

The following impairments are usually seen with (left/right?) hemisphere damage: contralateral hemiparesis & sensory loss, aphasia w/ decreased reading/writing, apraxia/motor planning issues, left/right confusion, compulsiveness, slowness, & depression.

Left

where is the ischial bursitsi

Lies between the ischial tuberosity and the overlying gluteus maximus muscle.

treatment for transient synovitis

Limit activities Bed rest Limit weight bearing by using crutches if child is old enough NSAIDS for pain management Symptoms usually resolve in about 7 days

limitations with PINCER

Limited range of motion of the hip, particularly EXTERNAL ROTATION of the flexed hip with a bony end-feel is characteristic.

factors that contribute to prenatal boney deformity

Limited space for fetus to move, including pregnancies with multiples Decreased amniotic fluid External forces from tightly stretched uterine and abdominal walls

flexible flat feet in kids

Longitudinal arch disappears with weight bearing but is present in sitting and walking on toes Caused by normal ligamentous laxity Children under 2 may appear to have flat feet due to plantar fat pad in medial foot that masks longitudinal arch Children typically have rapid spontaneous longitudinal arch development between 2-6 years of age

physiologic factors of DDH

Maternal hormonal influence of estrogen and relaxin on female infant's ligaments

___ is the greatest amt that will still allow the pt to move smoothly thru the available ROM.

Maximal resistance

central slip extensor tendon injury splinting

Mechanism of action: forcibly flexed while actively extended volar dislocation at PIP Presentation Unable to actively extend at PIP Passive extension is achievable, if not refer to orthopeadic or hand specialist Splint is same as Mallet Finger

volar plate injury splinting

Mechanism: Hyperextension at the PIP, not uncommon to have collateral damage too. Referral to specialist: unstable joint large avulsion fracture If stable w/out large avulsion fx: 30⁰ for 2-4 weeks then progressively increase ROM

surgical options for focal articular cartilage injuries

Microfracture- fills in with less functional hyaline cartilage Osteochondral Autologous Transplantation Surgery (OATS) Take area where you are missing tissue, go to another area of NWB subchondral, take a chunk of cartilage out, rough up the hole and then plug it Mosaicplasty

Muscles are strongest at (mid/end?)-range.

Mid-range

treatment of OA hip

Minimize iliopsoas activity due to it's compression moment Have patient slide leg on bed versus lifting it Wedge in chair decreases hip flexion and will reduce pain Standing on step, hang leg with cuff weight over the edge > distraction will ease the person's pain Cane can decrease compression at hip by 50%

minor pelvis fractures

Minor fractures involve avulsions and simple bone disruptions, and are routinely not fixated, due to the stable nature of these injuries.

A score of 0 on the ___ indicates no increase in muscle tone, while a 4 indicates the affected part is rigid (extremely high tone).

Modified Ashworth Scale

complications of TKA

Mortality: 0.5% to 1% per year Operative events DVT Infection Fractures Nerve Injury Dehiscence

Transient synovitis

Most common cause of hip pain in children under 10 years Affects males more than females, 4:1 Gradual or acute onset of limp Pain in hip or knee (referred) Cause unclear but often follows an upper respiratory infection or other illness

interventions for clubfoot non-invasive

Most effective immediately after birth and in extrinsic (supple) clubfoot When serial casting, first correct cavus then equinus Often followed by surgery between 3-12 months

A stroke (or brain injury) causes more than just ___ dysfunction.

Motor

ASIA C

Motor function is preserved in voluntary anal contraction (VAC) OR the patient meets the criteria for sensory incomplete status (sensory function preserved at the most caudal sacral segments (S4-S5) by LT, PP or DAP) AND the patient has sparing of motor function more than three levels below the ipsilateral motor level on either side of the body. (This includes key or non-key muscle functions to determine motor incomplete status.) central cord lesion

typical responses to needling

Motor twitch Muscle soreness (can last 12 - 24 hours) Pain referral in stereotypical myofascial pattern Sweating Emotional release

___ cause nearly half of all head injuries.

Motor vehicle accidents (MVAs)

What is a common complication of having a nasogastric tube?

Nosebleeds (also the pt may pull it out)

At what point does respiratory therapy stop?

Not until the abdominals are completely innervated

With Wernicke's aphasia, is the pt aware of their communication errors?

Not usually

what is the most common disease process affecting the hip

OA

intertrochanteric fractures

Occur along a line between the greater and lesser trochanters and are more common among elders. Almost always is the result of a fall.

aspects of fibula fractures

Occur following direct blow. These fractures heal well and rapidly. Upper fibular fractures (at the neck and upper shaft) may occur with severe rotary ankle injuries, where the tibial malleolus is fractured and the fibula fractures as part of this rotary fracture-dislocation of the ankle (Dupuytren fracture). The treatment based on management of the ankle fracture dislocation - internal fixation is required.

MOI for synovial plica

Overuse Repetitive bending/straightening Direct trauma Biomechanical abnormalities Post-op (creates synovitis)

interventions for clubfoot- surgical

Often needed for intrinsic (rigid) clubfoot Usually follows manipulation and casting regimen Soft tissue surgery- cast in best position following surgery and then serial cast if needed Bony surgery of foot and ankle

classification system for tibial tubercle fractures

Ogden and Murphy

cranial nerve I test

Olfactory Have them block one nostril, smell, switch

When re-donning a sequential compression device after therapy, where should the velcro straps be?

On top of the leg

Why is Baclofen administered intrathecally rather than orally most of the time? (2)

Only a small portion of oral Baclofen makes it into the CSF & it causes more side effects than when injected directly into the spine

When naming an SCI, when do you designate right/left or sensory/motor?

Only if the function is different on each side

Can you tx complex regional pain syndrome (shoulder/hand syndrome) thru all 3 stages?

Only stages 1 & 2; at stage 3 the damage is irreversible

Does e-stim decrease subluxation?

Only while it's being applied, not permanently

arthrokinematics during knee extension

Open chain Tibia glides anteriorly on the femur Tibia also rotates externally (screw-home mechanism*) Closed Chain Femur glides posteriorly on the tibia Femur rotates internally on a stable tibia (screw-home mechanism) Creates relative external rotation of tibia

arthrokinematics during knee flexion (full extension to 20 degrees flexion)

Open chain Tibia glides posteriorly on the femur Tibia also rotates internally (reverse of screw-home mechanism) Closed Chain Femur glides anteriorly on the tibia Femur rotates externally on a stable tibia (reverse of the screw-home mechanism)

cranial nerve II test

Optic Visual acuity snellen pupillary rxn to light rxn to accommodation snellen

Due to complications from an SCI (particularly those w/ lesions from C2-C6) pts often have problems w/ ____ bc blood pools in the abdomen.

Orthostatic hypotension

Are pts more likely to over- or under-grip an object when re-learning gripping tasks?

Over-grip bc they are afraid to drop it

post-op rehab of FAI (phase 3)

Phase III: Advanced Exercises (8 - 10 weeks) Hip ROM should be equal to opposite side Involved side is 70% or greater when compared to uninvolved side Improve cardiovascular endurance Improve balance reactions Strengthening exercises continue Addition of lunges Single-leg squats (10 weeks +)

post-op rehab for FAI (phase 4)

Phase IV: Activity specific (10 + weeks) Work on coordination, strength, agility, etc. Side shuffles Cariocas W-cuts Z-cuts Specific requirements of activity/sport Discharge

___ & ___ are the primary causes of death in ppl w/ SCI, particularly at the beginning.

Pneumonia & pulmonary embolism

(Negative/Positive?) symptoms hinder or mask normal movement.

Positive

Spasticity, abnormal tone, & a Babinski response are all (negative/positive?) symptoms of stroke.

Positive

Pts experiencing ___ syndrome may report problems w/ dizziness, irritability, fatigue, double vision, personality changes, concentration, abstract thinking, disequilibrium, & recent memory.

Post-concussion

finger collateral ligaments splinting

Presentation "jammed finger" pain isolated affected ligament PIP primarily involved Exam 90 flexion at MCP -prevent shortening of collaterals 30 and zero degrees of flexion at PIP Radiograph negative & joint stable then buddy tape or splint positive - refer to specialist

rehab considerations of TKA

Prevent infection Prevent DVT Manage pain and scarring Early ROM Post-op ROM determined by pre-op ROM Restore normal gait Address impairments: strength, balance, ROM

osteosarcoma PT

Prevent secondary complications of cancer and medical treatment Wound healing Promote health, wellness, fitness, normal development and functional mobility Use of prosthesis, assistive technology

signs and symptoms of traumatic myositis ossificans

Previous muscle strain or direct blow with significant bruising Pain with palpation of muscle Pain with activation of muscle Restricted ROM

rehab considerations post-op meniscus

Progressive ROM Progressive WB Repair = period of NWB Strength training: quadriceps & hamstrings Quadriceps endurance gradually add compression

During the volitional flexion synergy in the progression of tone, the forearm is usually (pronated/supinated?).

Pronated

What does PNF stand for?

Proprioceptive Neuromuscular Facilitation

actions of glute med

Provides pelvic support during stance -posterior fibers extend and laterally rotate the hip > antagonistic to TFL

UE muscle weakness starts (distally/proximally?)

Proximally

Ottawa Knee Rules

Pt is >55 y/o, inability to bear weight both immediately and in the emergency department (4 steps) isolated tenderness of the patella tenderness at head of fibula inability to flex to 90 degrees.

when early stance pronation is excessive...

Push-off occurs on a relatively mobile foot

interventions for plica

Quad strengthening & stretching Avoid repetitive activities/pressure Biomechanics training

interventions for anterior knee pain

Quadriceps strengthening Biomechanics training Motor control & Balance Hip Abductor/ER strengthening Soft tissue mobility/flexibility Ham and quad

You should pay "extreme attention" to what in the ICU?

ROM

when is surgery indicated for torticollis

ROM deficit greater than 15° No improvement after 6 months of conservative intervention

rhythmic stabilization

RS) involves isometric contraction of the antagonist, followed by isometric contraction of the agonist. This can be used anywhere in the range of a pattern. For improving strength For improving stability For motor learning

indications for TKA

Radiographic advanced arthritis Failed conservative management: therapy, activity modification, assistive device use, medication, weight loss

The ___ scale was developed to provide a means of determining various levels of recovery following a head injury.

Rancho Los Amigos (RLA)

In a(n) (areflexic/reflexic?) bladder, the bladder empties reflexively when there is sufficient stretch to the wall of the bladder.

Reflexic

Pts w/ complete SCI b/w C2-T11 often have problems w/ (areflexic/reflexic?) bladder bc of a lack of connection to the cortex despite an intact reflex arc.

Reflexic

If S2-S4 has an intact reflex arc, the bowel functions (areflexively/reflexively?).

Reflexively

goals of labral repair

Relieve pain by eliminating the torn flap Restore the anatomy of the hip Prevent osteoarthritis

The following are cons for (compensation/remediation?): delay of functional independence; requires significant hands-on approach that is labor intensive & prolonged; evidence supporting use of specific techniques is poor.

Remediation

The following are pros for (compensation/remediation?): exercise & facilitation techniques to reduce sensorimotor deficits & promote motor recovery & improved function of impaired segments; requires some degree of voluntary mvmt; training focuses on making impairments better.

Remediation

A task-oriented approach leading to motor learning & cortical reorganization is a part of a (compensatory/restorative?) approach to therapy.

Restorative

The following impairments are usually seen with (left/right?) hemisphere damage: contralateral hemiparesis & sensory loss, visuospatial impairment, poor body scheme, poor attn, neglect syndrome, memory problems for procedures.

Right

The following impairments are usually seen with (left/right?) hemisphere damage: time disorientation, problem solving issues, poor awareness, impulsiveness/safety issues, & trouble w/ concrete thinking.

Right

The lack of visual spatial attn (spatial neglect) in those who have a (left/right?) hemisphere lesion indicates unilateral neglect syndrome.

Right

Overall, the (left/right?) side of the brain is more global, while the (left/right?) side does mostly contralateral things.

Right; left

Ppl w/ (left/right?) hemisphere damage have more primary deficits that create greater dysfunction & have poorer functional outcomes than ppl w/ (left/right?) hemisphere damage.

Right; left

At what level are back extensors (superficial and deep) *fully* innervated?

S1

At what level is the gastroc innervated

S1

myotome- plantarflexion

S1

myotome- extention at the hip

S1 glute max

At what level is the gluteus maximus innervated

S1-2

At what level is the soleus innervated

S1-2

DTR ankle

S1-2

Strengths that a pt w/ a C7 SCI has include ___ & ___ innervation, & challenges are an only partially innervated ___, which can make it difficult to push up in WC for pressure relief.

SA & triceps; lats

torticollis

SCM rotates head to opposite side and laterally flexes to same side named according to side of involved SCM righting rxn delay

anterior subluxation places pressure on

SH of the bicep

what must you retrain with superior sublux

SIT

ASIA D

Same as for ASIA C, with the exception that at least half (half or more) of key muscle functions below the single NLI have a muscle grade ≥ 3

slow reversal holds

Same as slow reversals EXCEPT Slow reversal-hold (SRH) adds an isometric contraction at the extremes in the range (or anywhere there is weakness!)

Strengths that a pt w/ a C6 SCI has include more ___ control, but a challenge for them is that they only have partial ___ innervation.

Scapular; SA

Muscle shortening, contractures, pain, subluxation, DVTs, dysphagia, etc are (primary/secondary?) stroke impairments.

Secondary

signs and symptoms of patellar subluxation or dislocation

Sensation of knee "going out" Intense pain with effusion Inability to actively flex knee Laterally displaced patella Significant soft tissue damage Deformity Often reduces on its own with extension of the knee

Fine motor coordination is often (sensory/visually?) based.

Sensory

Hand shaping is highly driven by ___ & ___ input & you need to remind the pt to plan ahead before attempting a reaching/grasping task.

Sensory & visual

ASIA B

Sensory but not motor function is preserved below the neurological level AND sensation includes the sacral segments S4-5 (light touch or pin prick at S4-5 or deep anal pressure) AND no motor function is preserved more than three levels below the motor level on either side of the body. anterior horn syndrome

When determining the neurological level of injury, it's the most caudal segment of the cord in which ___ is intact & motor grade is at least a ___.

Sensory; 3

Name an underactive muscle you might consider taping to facilitate in a pt post-stroke.

Serratus anterior

patella

Sesamoid bone Imbedded in quadriceps and patella tendon Serves similar to a pulley for improving angle of pull (results in greater mechanical advantage in knee extension)

rickets

Severe genu varum Vitamin D deficiency leading to soft bones Rule out if genu varum is present after age 4 or if worsening over time Often seen in malnourished children

verbal and visual cues in PNF

Sharp verbal cues allow patients to understand and prepare for the movement expected. In addition the verbal cues can accompany the expected output. Patients are asked to follow the movements with the eyes to enhance control of the movement through visual cues and pathways.

Anterior subluxation puts pressure on the ___ & can cause tendonitis.

Short head of the biceps brachii

How is sensory input from the therapist (generally over the agonist muscle) theoretically supposed to help during PNF?

Should allow the pt to understand the motor output expected; should be faded out over time

In addition to hamstrings & heel cords, special attn should be paid to ROM at the ___ joint for ppl w/ a C5 lesion.

Shoulder

clinical exam of DDH

Significant limitation of hip abduction with flexion (typical is 75-90 degrees) or asymmetry of ROM of 5-10 degrees Asymmetric thigh folds Positioning Apparent femoral shortening with uneven knee heights Hip stability testing for infants up to 3 months Ultrasound imagery

pirogoff amputation

Similar to Boyd except calcaneus is transected vertically and rotated 180 degrees; results in no limb length discrepancy

What are the first things you want to work on w/ a pt post-TBI? (5)

Sitting balance, standing balance, transfers, gait, ADLs

common sustained postures with femoral anterior glide syndrome with lateral rotation

Sitting with ankle on knee or thigh Common in activities that require lateral rotation (ballet, hockey, soccer)

contraindications of dry needling

Skin lesions Pregnancy Over metal implants 3-6 months post-surgery (possibility of infection) Bacterial endocarditis Bleeding disorders Don't manipulate the patient within 48 hours after needling

mechanical factors with DDH

Small intrauterine space Breech presentation Fetal hip positioned against mother's sacrum

interventions for compressive syndromes

Soft tissue mobility Joint mobilization Taping? Avoid OKC 0-30 & CKC >60

(Anosognosia/Somatagnosia?) is the inability to recognize body parts.

Somatagnosia

W/ central cord syndrome, the pt will have (flaccid/spastic?) paralysis below the level of the spinal cord injury in the C-T-L-S order.

Spastic

In pts post-stroke, ___ often results when mm undergo a quick stretch, such as when the pt shifts wt during transfers, ADLs, or gait.

Spasticity

Post-stroke, ___ is a result of a lack of inhibition of muscle tone & activity by the CNS (upper motor neuron lesion).

Spasticity

Professionals should be wary of making a dx of complete/incomplete 2-7 days post-injury due to ___.

Spinal shock

To force arousal, briefly normalize muscle tone, & stretch joints are 3 reasons you would ___ a pt in the ICU, even it takes 2 ppl w/ total assist.

Stand

four subtypes of torticollis

Sternomastoid tumor Palpable mass, Radiographs normal Muscular torticollis Tightness, No tumor, Radiographs normal Postural torticollis No tightness, No tumor, Radiographs normal Postnatal muscular torticollis Environmental-induced Plagiocephaly-induced Positional preference-induced

interventions for meniscus

Strengthening: musculature crossing knee Especially quadriceps Avoid deep squatting, kneeling, twisting, pivoting, repetitive bending, running (compression + shear) Balance & Motor Control* LE Biomechanics*

A weak glute med can be compensated for with a cane on the (strong/weak?) side to move the GRFV more laterally.

Strong

Failure to compensate for glute med weakness can be seen as a pelvic drop on the (strong/weak?) side, & is called a glute med limp.

Strong

Amnesias occur due to damage to the medial ___ lobes & the ___.

Temporal; hippocampus

Assessment legg calve perthes disease

Tend to limp and often have a positive Trendelenberg sign from pain or weakness Limited ROM especially with hip abduction and internal rotation Complain of pain in groin, hip, or knee (referred) Muscle spasms common in early stages

signs and symptoms of osgood-schlatter

Tender "bump" below the knee Pain with extension RROM Pain during rigorous activity Jumping Deep knee bends Weight-lifting

What explains "sacral sparing"?

That level is the furthest outside (away from the center of the cord) in the lateral corticospinal & spinothalamic tracts, so it's the last to be injured in a central cord syndrome

What do doctors have to verify before giving a pt thrombolytic therapy for stroke?

That the stroke is not hemorrhagic (the drug will cause further bleeding if it is)

what is IM nail fracture fixation

The IM nail is placed into the endosteal canal of the bone and performs load-sharing duties, which usually permits weight bearing across the fracture site.

acetabulum deformation (PINCER)

The acetabulum gives increased coverage of the femoral head - femoral neck junction. More common in middle-aged athletic women.

what is an avulsion fracture

The attachment of a tendon or ligament is pulled from the bone; mechanism of injury involves a violent muscle action or tremendous acceleration/deceleration of the body.

What is one "good" thing about open head injuries?

The brain has room to swell, which can reduce compression of brain tissues

What does a JOT chart allow us to evaluate?

The effect of a sling, or other humeral supports, on an individual (including force, impact on tone, access to environment, & cosmesis)

Arthrology

The major joints of the ankle and foot are the talocrural, subtalar, and transverse (midtarsal) tarsal joints. The talus is mechanically involved with all three of these joints

Os trigonum

The os trigonum is a common accessory bone of the foot that usually ossifies and fuses with the talus between the ages of 8-11 as a secondary ossification center. If the os remains unfused it is termed os trigonum.

What happens if one of the 3 sensory systems used for balanced is compromised?

The other 2 become hypersensitive & take over the altered system (post-stroke, often all 3 are altered & extensive balance re-training is needed)

sensory input PNF

Therapist's hands guide movement and resistance. This sensory input theoretically allows the patient to understand the motor output expected. This sensory feedback should be faded over time! Whenever possible, manual contact is made over the agonist muscle groups or their tendons

limitations with CAM

There is limitation of flexion, adduction, and characteristically limited INTERNAL ROTATION, with a hard end-feel

What does the literature say about whether subluxations inhibit muscle return or potential?

There is no literature support either way

Why do embolic strokes tend to be more severe than thrombotic strokes?

They occur rapidly & there is no opportunity for a collateral blood supply to develop (so they tend to have more serious effects)

According to Prof James, she recommends that a pt wear (knee/thigh?)-high compression pantyhose. Why?

Thigh high; to prevent venous restriction at the knee

femoral anterior glide syndrome at the hip is similar to

This is very similar to the anterior humeral glide syndrome at the shoulder

rhythmic initiation

This technique employs phases of voluntary relaxation, passive movement and repeated dynamic contractions of the major muscle groups involved in the agonistic pattern of movement. Therapists can use this technique to work from a passive ROM technique into a more active assistive or active ROM Therapists can thus use this technique to improve flexibility and to begin motor learning.

contract relax

This technique utilizes the golgi tendon organ, which inhibits a muscle after a sustained contraction that lasts longer than 6 seconds.

test for achilles rupture

Thompson matles

The shoulder rules

Thoracic Scapula Rotator cuff

Hinged AFOs prevent foot drop during swing phase, but do NOT prevent the ___ from collapsing during stance phases.

Tibia

anterior LE compartment

Tibia anterior EDL EHl fibularis tertius anterior tibial artery deep peroneal nerve

active soft-tissue restraints of the knee

VMO lateral retinaculum- VL and ITB

timing in PNF

Timing is distal to proximal. PNF gurus expect the distal component of the movement to be completed by half way through the PNF pattern

Why would you turn a glove inside - out when putting it on a pt's edematous hand?

To put the seams on the outside so they're not pressing into the swollen skin

What problem in the swing phase of gait does a glute med limp cause?

Toe clearance issues on the strong side

A (bottom up/top down?) approach to a stroke eval streamlines the eval, helps focus on specific probs, focuses intervention to its impact on functional limitations, & ensures a functional outcome.

Top down

The following describes a (bottom up/top down?) approach to a stroke eval: -determine meaningful roles, environments, & tasks -establish present competency in roles/tasks -observe function to ID underlying impairments

Top down

Slight ___ is used at the beginning of the pattern to help facilitate mvmt thru the joint.

Traction

three main bursar at the hip and pelvis

Trochanteric bursa - look for gluteal tendinopathy Iliopsoas bursa Ischial bursa

cranial nerve IV test

Trochlear 6 cardinal gazes

post-surgical PT for scoliosis

Trunk rotation contraindicated Patient education/training on log rolling, bed mobility, transitions, ambulation, dressing (in tailor sitting), orthotic wear General ROM and strengthening exercises without resistance for extremities Incision care

meniscal injury MOI

Twisting Pivoting Plant & Cut Deep squatting Injury to related structures MCL, ACL Degeneration

4 types of OI

Type I: generalized osteoporosis, joint hyperlaxity, hearing loss Type II: often still born, extreme bone fragility, small bodies and short, curved limbs Type III: severe, progressive deformity of long bones, skull and spine, hearing loss common, death occurs in childhood Type IV: mild to moderate deformity, short stature, hearing loss variable, bone fragility varies, excellent prognosis for ambulation

four types of OI

Type I: generalized osteoporosis, joint hyperlaxity, hearing loss Type II: often still born, extreme bone fragility, small bodies and short, curved limbs Type III: severe, progressive deformity of long bones, skull and spine, hearing loss common, death occurs in childhood Type IV: mild to moderate deformity, short stature, hearing loss variable, bone fragility varies, excellent prognosis for ambulation

innominate osteoplasty

Typically acetabulum face is too shallow and often faces too outward Pelvis is cut and rotated downward around the pubic symphysis. A wedge of bone from the innominate locks it in. Does not change the shallowness, an acetabuloplasty does that If a spastic muscle is the cause (i.e., iliopsoas) then it is lengthened

idiopathic scoliosis

Unknown cause Most common form of scoliosis Can occur at any age in childhood

Name an overactive muscle you might consider taping to inhibit in a pt post-stroke.

Upper trap

___ incontinence is the type of incontinence most likely to occur after stroke bc of an uninhibited neurogenic bladder.

Urge

___ infections affect 80% of ppl in the acute phase of SCI & continue to be problematic in most pts.

Urinary tract

why are LE splints and orthotics used

Used to improve alignment and positioning Used to prevent contracture or progression of contracture Dynamic splints and night splints can be used for prolonged static stretch

femoral varus rotational osteotomy (VRO) and acetabuloplasty

Usually VROs are not to undo excess valgus, but to offset less vertically stable sockets. Most commonly done to correct excessive femoral antetorsion (toeing in) Unfortunately it typically makes the leg shorter and causes odd posturing of the LE in standing and sitting.

MOI for patellar tendon rupture

Usually a deceleration force *Landing off balance from jump *Stepping in a hole *Previous tendon pathology

In patients at level _____ Behavior not goal directed and is not self-monitored

V

Patients at level ______ are able to converse on social, automatic level for brief periods of time but are Unable to understand joking, sarcasm, etc.

V

A patient at RLA level _____ is not oriented to person, place, or time, but a patient at RLA level ______ is inconsistently oriented to person, time, and place

V; VI

Patients at level _____ have severely impaired recent memory with confusion of past and present in reaction to ongoing activity. vs. Patients at level ___ are able to use assistive memory aide with maximum assistance

V; VI

Patients at level ______ show responses to simple commands w/o external structure that are random and non-purposeful vs. Patients at level _______ are able to follow simple directions consistently

V; VI

Patients at level _______ are unable to learn new information vs. patients at level ______ show an emerging ability to do so

V; VI

When patients at level ______ verbalizations about present events become inappropriate and confabulatory when external structure and cues are not provided vs. Patients at level _____ use verbalizations that are appropriate in highly familiar and structured situations

V; VI

At which level does the ability to follow directions consistently emerge

VI

A patient at level ____ is able to use assistive memory aide with maximum assistance vs. A patient at level _____ has shallow recall of what he/she has been doing after completion of familiar task

VI; VII

Patients at level _____ are inconsistently oriented to person, place, and time vs. Patients at level _____ are consistently oriented to person and place (in familiar environments) but need cueing for orientation to time

VI; VII

Patients at level _____ may be able to show carryover from relearned, familiar tasks vs. Patients at level _____ are able to perform NEW learning with minimal assistance

VI; VII

Patients at level ______ are able to attend to highly familiar tasks in a non-distracting environment for 30 min with *moderate cues* vs. patients at level _____ able to attend to highly familiar tasks for 30 minutes with *minimal assistance*

VI; VII

Patients at level ______ are unaware of impairments, disabilities, and safety risks vs. Patients at level ____ have superficial awareness of their condition but they are in denial and may begin planning for the future

VI; VII

A patient at level _____ has shallow recall of what he/she has been doing after completion of familiar task vs. A patient at level _____ is able to recall and integrate past and recent events. Use assistive memory devices with cues

VII; VIII

A patient at level ______ may be in denial, unrealistically planning for the future vs. a patient at level _______ is aware of deficits and acknowledges them when they interfere with tasks. They may be depressed, easily angered, argumentative

VII; VIII

patients at level _____ able to attend to highly familiar tasks for 30 minutes with minimal assistance vs. patients at level ______ can attend and complete familiar tasks in a distracting environment for 1 hour

VII; VIII

At which level is the person consistently oriented to person, place, and time

VIII

A patient at level _____ is able to recall and integrate past and recent events. Use assistive memory devices with cues vs. a patient at level ______ uses assistive devices independently

VIII; IX

Patients at level ____ may over- or under-estimate abilities and require assistance to take corrective action when deficits interfere with tasks vs. Patients at level ______ accurately estimate abilities but requires SBA to adjust to task demands or anticipate a problem or take action to avoid it

VIII; IX

Patients at level ______ are able to recognize socially inappropriate behavior and able to take corrective action with *minimal assistance* vs. Patients at level ______ are able to monitor social appropriateness with *standby assist*

VIII; IX

In a ___ state, arousal is present, but the ability to interact w/ the environment is not. General responses to pain exist (inc HR, respiration, sweating), sleep-wake cycles, respiratory & digestives functions are present.

Vegetative

MOI for tibial plateau fracture

Vertical stress on a flexed knee Force to the lower leg while in varus/valgus position

How quickly does learned non-use occur post-stroke for the affected side? What can you do about it?

Very quickly learned; can do constraint-induced therapy to help reverse/prevent this

What is the best thing to do to facilitate return of function?

Weight bearing

What is the main tx for complex regional pain syndrome (shoulder/hand syndrome)?

Weight bearing

ilifemoral ligament

Y-ligament of Bigelow): the strongest ligament in the body. Attaches to the AIIS, inserts along the intertrochanteric line of the femur. Resists extension and internal rotation.

Is sensory re-training effective for pts post-stroke?

Yep!

Can a mild brain injury be debilitating?

Yes

Can you ambulate a pt who is on a vent?

Yes, probably with an ambu bag & the help of a 2nd person

With Broca's aphasia, is the pt aware of their communication errors?

Yes, which can create frustration, anger, & depression

Is heredity a risk factor for aneurysms?

Yes: you're more likely to have an aneurysm if you have family members who have had one

acetabular labral tear: two scenarios

Young person with a sudden pivoting or twisting action into external rotation and hyperextension Older person with Hx of hip and/or acetabular dysplasia or from repetitive trauma, pivoting, or twisting (Positive correlation with a history of Legg-Perthes disease, Slipped Capital Femoral Epiphysis, Developmental Hip Dysplasia, and/or Impingement)

___ refers to any partial preservation of motor or sensory function below the neurological level of injury in a complete injury.

Zone of Partial Preservation

what is spasticity a result of

a lack of inhibition of muscle tone and activity by the CNS and over production of muscle activity (or activity that does not match the task)

why is anterior knee pain the black hole of orthopedics

a lot of people have it and we aren't good at treating it

physical signs of neural tissue involvement

a) Does it reproduce the Signs & Symptoms? b) Are the test responses altered by distal movements? c) Are there differences from right to left? d) Beware that the good side may be affected too. e) "Signs of adverse neural tissue tension, when present, must be complementary to some condition determined by the overall examination before their meaning can be discerned

precautions and contraindications of nerve tension treatment

a) Irritable disorders or Severe pain b) Neurological changes are worsening c) acute compartment syndrome d) injury likely to cause neurological deficit e) Inflammatory, systemic, and ineffective disorders that affect the nervous system i) abscess ii) Guillain Barré f) Tethered spinal cord g) Marked injury or abnormality h) spinal instability

recovery of diaschisis

a. "state of low reactivity that occurs after injury to brain in areas previously stimulated by this portion of the brain" b. Recovery occurs in more distal portions of the brain

A pt in a power WC should change positions for pressure relief every ___. a. 15 min b. 30 min c. hr d. 2 hrs

a. 15 min

If an injury is complete, it scores an ASIA level: a. A b. B c. C d. D e. E

a. A

Ppl w/ ___ aphasia have a lesion in the left prefrontal motor cortex in the frontal lobe. a. Broca's b. Wernicke's c. conduction d. global

a. Broca's

Using the ASIA scale, you only test sensory when looking at spinal cord levels: (multiple answers) a. C1 - C4 b. C5 - C8 c. T2 - L1 d. S1 - S3 e. S2 - S4

a. C1 - C4 c. T2 - L1 e. S2 - S4

Things to work on for injuries at the ___ level(s): gain limited head control & strength thru head wand & mouth stick activities; pain management/prevention in UEs. a. C1-C3 b. C4 c. C5 d. C6 e. C7

a. C1-C3

Ppl w/ a ___ lesion have a very stable shoulder & strong elbow flexion. Only the fittest, leanest, most athletic ppl accomplish total independence at this point, but it is the first point at which total independence is possible. a. C6 b. C7 c. C8 d. T1

a. C6

functional/adaptive recovery

a. Functional recovery from learned to do previously learned tasks and become independent b. Amount of functional recovery inversely proportional to degree of damage from stroke

The ___ sling provides vertical support of the HOH & displaces the wt of the arm over both shoulders for better symmetry. But, it places the shoulder in adduction & IR & minimizes functional use of the arm. a. Harris hemi b. Bobath c. Single strap hemi d. Roylan humeral cuff

a. Harris hemi

Indicate the Rancho level: No response--complete absence of observable change in behavior when presented visual, auditory, tactile, proprioceptive, vestibular, or painful stimuli. a. I b. II c. III d. IV

a. I

Impairment-based goals, such as increasing the level of responsiveness, positioning, ROM, edema/tone mgmt, skin integrity, etc, are most appropriate for pts in which Rancho levels? a. I & II b. III & IV c. V & VI d. all of the above

a. I & II

The Rappaport Coma Scale, Coma Scale, or Disability Rating Scale may be used to measure progress at which Rancho levels? a. I - III b. IV - VII c. VIII - X d. appropriate at all levels

a. I - III

At Rancho level ___ (confused/agitated), the pt is alert & in a heightened state of activity. They make purposeful attempts to remove restraints/tubes, they may perform motor activities, but not on request; have brief attn & short term memory; aggressive behavior & mood swings; incoherent & inappropriate verbalizations. a. IV b. V c. VI d. VII e. VIII

a. IV

Intervention strategies at Rancho level __ include providing a calm, soothing, & relaxed atmosphere, giving simple, repeated instructions, & frequently orienting them to person, place, & time; they need 24-hr supervision. a. IV - V b. VI -VII c. VIII - IX d. IX - X

a. IV - V

repercussion of ischemia

a. Ischemia is an area of infarct surrounded by area of moderate blood flow (ischemic penumbra) b. Can allow non-functioning neurons to resume function

If you have an injury below level ___, you don't have a CNS injury, it's a PNS injury & PNS nerves regenerate, so these pts have a better prognosis. a. L2 b. L4 c. S1 d. S3

a. L2

How are diffuse axonal injuries dx'd? a. MRI b. CT c. PET d. X-ray

a. MRI

Which of the following mm are particularly prone to spasticity after stroke? a. PFs b. DFs c. quads d. hamstrings

a. PFs (apparently also quads, but this is rare)

A lesion of the right optic nerve would cause what type of loss? a. R eye blindness b. bitemporal hemianopsia c. L homonymous hemianopsia d. upper quadrant hemianopsia e. homonymous hemianopsia w/ macular sparing

a. R eye blindness

recovery of post-stroke edema

a. Secondary edema disrupts neuronal functioning b. Edema resolution up to 8 weeks c. One reason hemorrhagic strokes recover more dramatically

CNS reorganization

a. Synaptogenesis is enhanced through motor learning b. Re-acquiring motor skills is positive change while injury os negative change c. Reorganization is a reflection of demands we put on patients d. Alter demand by altering task and environment

Rehab goals are secondary to medical stability & are short-term in readiness for more extensive rehab. Assessment of motor impairment, cognition, language, balance, visual perception, & executive function should occur in which setting? a. acute care b. LTAC c. SNF d. IP rehab e. home health f. OP

a. acute care

Which of the following is the most common incomplete spinal cord lesion? a. central cord syndrome b. anterior cord syndrome c. Brown Sequard syndrome d. conus medularis lesion

a. central cord syndrome

___ can be caused by an acute hyperextension injury, a chronic or congenital condition resulting in progressive stenosis, spondylosis, or osteophytes creating a "pincer" effect. a. central cord syndrome b. anterior cord syndrome c. Brown Sequard syndrome d. conus medularis lesion

a. central cord syndrome

Quadriplegia/Tetraplegia is a result of a lesion in the ___ spinal cord. a. cervical b. thoracic c. lumbar d. sacral e. b & c f. a & b

a. cervical

Timing is ____. The expectation is that the distal component of the mvmt be completed by halfway thru the PNF pattern. a. distal to proximal b. proximal to distal

a. distal to proximal

___ is an alteration of speech sounds that results from impaired muscle control of the speech mechanism. Speech is garbled & difficult to understand. a. dysarthria b. apraxia c. anomia d. dysphagia

a. dysarthria

Using the ASIA scale, at level C5, you test ___. a. elbow flexion b. wrist extension c. elbow extension d. finger flexion e. finger abductors

a. elbow flexion

Which of the following are NOT key functional mm at the T1-T6 level? a. external obliques b. dorsal/palmar interossei c. lumbricals d. intercostals e. long mm of the upper back

a. external obliques

Compensatory intervention to improve the dysfunction thru repetitive practice & feedback. a. functional/adaptive intervention b. remedial/restorative intervention

a. functional/adaptive intervention

Pts w/ SCI don't always retain accurate autonomic control of the ___. a. heart b. liver c. kidneys d. GI tract

a. heart

___ is loss of vision in one half (temporal or nasal) of the field in one eye. a. hemianopsia b. bilateral hemianopsia c. homonymous loss of field d. homonymous hemianopsia

a. hemianopsia

___ is a PNF technique where the pt isometrically contracts the tight muscle against the therapist, holds, then relaxes & the therapist takes the muscle farther into the range w/ a stretch. a. hold relax or contract relax b. contract relax w/ antagonist contract c. hold relax swing/bounce

a. hold relax or contract relax

Which of the following does a sling actually address? a. increases vertical rise of humerus b. improves scapular alignment c. corrects trunk imbalances d. prevents IR

a. increases vertical rise of humerus

The ___ transmits motor signals. a. lateral corticospinal tract b. dorsal column c. lateral spinothalamic tract d. anterior spinothalamic tract

a. lateral corticospinal tract

___ syndrome is a rare neurological condition in which a person can't physically move any part of the body except the eyes. They are conscious & able to think. a. locked-in b. 2nd impact c. diffuse axonal d. post-concussion

a. locked-in

Contralateral paralysis & sensory loss of the UE, hemianopsia, & limb-kinetic apraxia are all effects of a stroke in the: a. middle cerebral artery (MCA) b. anterior cerebral artery (ACA) c. posterior cerebral artery (PCA)

a. middle cerebral artery (MCA)

The ___ supplies almost all of the exterior of the brain. a. middle cerebral artery (MCA) b. anterior cerebral artery (ACA) c. posterior cerebral artery (PCA)

a. middle cerebral artery (MCA)

With embolic stroke of cardiac nature, the ___ artery is involved 80% of the time. a. middle cerebral artery (MCA) b. anterior cerebral artery (ACA) c. posterior cerebral artery (PCA)

a. middle cerebral artery (MCA)

A ___ brain injury is dx'd only when there's a change in the mental status at the time of injury (dazed, confused, loss of consciousness) & indicates the brain's functioning has been altered. a. mild b. moderate c. severe

a. mild

A score of 13-15 on the Glasgow Coma Scale indicates a ___ brain injury. a. mild b. moderate c. severe

a. mild

The following describes which phase of spinal shock: complete loss of all reflexes below level of injury. a. phase 1: 24-48 hrs b. phase 2: days 2-3 c. phase 3: 1-4 wks d. phase 4: 1-12 months

a. phase 1: 24-48 hrs

Using the ASIA scale, at level S1, you test ___. a. plantar flexors b. ankle dorsiflexors c. hip flexors d. great toe extensors e. knee extensors

a. plantar flexors

___ have(has) a growing body of evidence to support their(its) use in tx'ing diplopia, but few settings use them(it). a. prisms b. vision therapy c. patching d. spot patching

a. prisms

Which of the following is the most common of the 4 major hemorrhagic hypertensive sites? a. putamenal b. thalamic c. pontine d. cerebellar

a. putamenal

___ is a technique that employs phases of voluntary relaxation, passive mvmt, & repeated dynamic contractions of the major muscle groups involved in the agonistic pattern of mvmt. a. rhythmic initiation b. slow reversals c. slow reversal holds d. rhythmic stabilization

a. rhythmic initiation

Which of the following NOT a muscle a person w/ a C5 lesion has control of? a. serratus anterior b. deltoid c. biceps d. teres minor, suprapinatus, & infraspinatus (most of rotator cuff)

a. serratus anterior

Which mm become stretched & weak due to a downwardly rotated scapula? (2) a. serratus anterior b. rhomboids c. levator scapulae d. lower trap

a. serratus anterior d. lower trap

Which of the following is NOT a good suggestion for promoting optimal conditions for UE maintenance & control after SCI? a. slings b. positioning c. education on overuse/training d. muscle strengthening

a. slings

A(n) ___ hematoma is a collection of blood below the inner layer of the dura, but external to the brain & arachnoid membrane. a. subdural b. epidural c. subarachnoid d. intraparynchemal

a. subdural

Which is the most common type of traumatic intracranial mass lesion? a. subdural hematoma b. epidural hematoma c. subarachnoid hematoma d. intraparynchemal hematoma

a. subdural hematoma

Inability to initiate abduction is the fault of weakness in which mm post-stroke? a. supraspinatus b. infraspinatus c. teres minor d. subscapularis e. entire rotator cuff

a. supraspinatus

Which of the following is an example of muscle facilitation? a. tapping b. pinching c. hitting d. passive placement into a position

a. tapping

The ___ lobe contains auditory information & memory, & is considered the "what system". a. temporal b. parietal c. frontal d. occipital

a. temporal

When assessing gait on a pt post-stroke, you should always do some sort of ___ gait test. a. timed b. distance c. cadence d. step-counting

a. timed

___ brain herniation occurs when it displaces downward into the pons area. a. uncal b. central c. cingulate d. transcalvarial e. upward f. tonsillar

a. uncal

femoracetabular impingement

abutment between proximal femur and acetabular rim

cranial nerve XI tests

accessory atrophy of traps shrug shoulders turn head

what is a epidural hematoma

accumulation of blood between the skull and the dural membrane from focused blow to the head excellent prognosis

normal compressive forces at the hip help form the _____ and ____ to create stable hip joint

acetabulum and femoral head

why is sitting good for normalizing tone and for arousal

activates the reticular activating system

visual perceptual skill development (three steps)

acuity, field, oculomotor control attention, scanning, pattern recognition memory and cognition

treatment of subacute

address impairments increase ROM increase strength (Active-assist to active to resistive) neuromotor control

sensation can be hampered by impairment of _____

afferent impulses at the cortical level

after age ___ children should have orthotics instead of splint

age 5

bobath sling

allows arm to rest at side and be used support through axilla minimize vertical displacement ONLY for minimal subluxation

dysarthria

alteration in speech sounds that results from impaired muscle control of the speech mechanism (garbled speech)

sensory disturbances can occur as a result of ___

altered or absent sensory input

clinical manifestations for OI

apparent normal appearance with occasional fractures to severe involvement with growth retardation additional features including: hearing impairment, scoliosis, pectus deformity, deformed teeth, excess diaphoresis, cardiovascular complications, metabolic defects

how to enhance circulation and reduce bruising edema with tape

apply fanned tape with none to light tension to injured tissue with skin passively stretched/joint near end ROM

to facilitate a weak muscle with tape

apply tape with light to moderate tension from O toI with target muscle passively elongated

DD for patellar stability/pain

apprehension test patellofemoral grinding test/Clarke's

test for paratendonitis vs tendonopathy

arc test royal London hospital

posterior lateral corner of knee contains ____

arcuate ligament complex popliteal tendon LCL posterior collateral joint capsule

possible autonomic impairments with TBI

arousal awareness sleep disturbances

tests for hypoglossal nerve

articulation of words stick out tongue and move it side to side

tests for accessory nerve

atrophy of traps shrug against resistance turn head against resistance

graft choices of ACLR

autograft- patellar tendon or hamstring allograft- cadaver, not as strong

juvenile idiopathic arthritis

autoimmune swelling of joint, heat, limited motion, pain with motion

ranchos level VII

automatic-appropriate

maximal resistance

aximal resistance is the greatest amount that will still allow the patient to move smoothly through the range available. Maximal resistance encourages "overflow" of muscle activity into other related muscles, joints, extremities, the trunk, or the neck. PNF employs offering maximal and prolonged resistance in order to allow for prolonged firing of muscle spindles and joint receptors. Remember! Muscles are strongest mid-range - grade your resistance accordingly.

ICP levels above ___ mmHg are usually tx'd. a. 15 b. 20 c. 25 d. 30

b. 20

The long term effects of ___ syndrome can be muscle spasms, increased muscle tone, rapidly changing emotions, hallucinations, & difficulty thinking/learning. a. locked-in b. 2nd impact c. coup-contrecoup d. post-concussion

b. 2nd impact

___ syndrome (aka "recurrent traumatic brain injury") can occur when a person sustains another traumatic brain injury before the damage from the first traumatic brain injury has healed. a. diffuse axonal b. 2nd impact c. locked-in d. post-concussion

b. 2nd impact

If an injury is incomplete, & sensory but not motor function is preserved below the level of the injury & at S4/S5, it scores an ASIA level: a. A b. B c. C d. D e. E

b. B

The ___ sling allows the arm to rest at the side of the body & allows functional use of the arm. Disadv's include horizontal displacement & possible circulation compromise in the axilla. a. Harris hemi b. Bobath c. Single strap hemi d. Roylan humeral cuff

b. Bobath

The ___ sling should be used for pts w/ minimal subluxations only & who have some muscle tone support. a. Harris hemi b. Bobath c. Single strap hemi d. Roylan humeral cuff

b. Bobath

Things to work on for injuries at the ___ level(s): gain normal head control & limited scapular control, assist pt in avoiding UE pain & hand contractures; facilitate strengthening of upper, middle, & lower trap function & normal ROM at every joint. a. C1-C3 b. C4 c. C5 d. C6 e. C7

b. C4

At injury level ___, ppl can start going out on their own w/ minimal help. a. C4 b. C5 c. C6 d. C7

b. C5

At which level of injury is elbow flexion available? a. C4 b. C5 c. C6 d. C7

b. C5

Things to work on for injuries at the ___ level: achieve limited trunk control thru UE placement of hands/elbows, maintain WB positions on outstretched arms in supine, prone, & sidelying, preserve tenodesis grip (for up to a year in case of return of add'l fn), & teach them to do as much for themselves as possible. a. C4 b. C5 c. C6 d. C7 e. C8

b. C5

Things to work on for injuries at the ___ level: progressive AA/A/RROM for shoulder, balanced forearm othoses, positioning to prevent finger flexor elongation, WB w/ wrist ext, preservation of tenodesis grip (for up to a year, in case of return of add'l fn), & elbow extension splints. a. C4 b. C5 c. C6 d. C7 e. C8

b. C5

At level __ some pts can use a ladder trapeze, loops or straps to be independent in bed mobility. a. C5 b. C6 c. C7 d. C8 e. T1

b. C6

___ is the first level where ppl have the possibility of using a manual WC for all mobility, but it must be an ultra lightweight & they have to use a "pull to" strategy to be mobile. a. C5 b. C6 c. C7 d. C8 e. T1

b. C6

"Super" clients at which level are the first to have the potential for floor to chair transfers? a. C6 b. C7 c. C8 d. T1

b. C7

A level ___ is the first consistent potential possibility of living totally independently & using a manual WC. a. C6 b. C7 c. C8 d. T1

b. C7

Clients at ___ level & below are independent w/ all self-care, feeding, eating, & hygiene. Some require assistance w/ bowel & bladder care (particularly women). Most need adaptations to hold & manipulate objects. a. C6 b. C7 c. C8 d. T1

b. C7

Indicate the Rancho level: Generalized response--reflex to painful stimuli; response to external stimuli w/ generalized physiological changes, gross body mvmt &/or non-purposeful vocalization; responses may be the same regardless of type & location of stimuli; delayed responses. a. I b. II c. III d. IV

b. II

At which levels would you use the Agitated Behavior Scale? Are lower (14) or higher (56) scores better? a. IV - V, higher b. IV - V, lower c. VI - VII, higher d. VI - VII, lower

b. IV - V, lower

Pts w/ a stroke who are still medically unstable & require more than 100 days of care & are often in an induced coma or on a vent. Rehab goals are to prevent physical & mental deterioration as they become more medically stable in which setting? a. acute care b. LTAC c. SNF d. IP rehab e. home health f. OP

b. LTAC

___ are not really considered strokes, but rather precursors to stroke. There's a brief, focal loss of function, but full recovery w/in 24 hrs. a. thromboses b. TIAs c. lacunar strokes d. embolisms

b. TIAs

At Rancho level ___ (confused, inappropriate, non-agitated), behavior is not goal directed or self-monitored; they can't learn new info or understand joking/sarcasm; verbalizations become inappropriate or confabulatory w/o external structure or cues. a. IV b. V c. VI d. VII e. VIII

b. V

At Rancho level ___ (confused, inappropriate, non-agitated), the pt is alert, & may wander randomly w/ the vague intention of going home; not oriented to person, place, or time; have frequent brief periods of non-purposeful sustained attn; have severely impaired recent memory, but may perform previously learned tasks w/ cues & structure. a. IV b. V c. VI d. VII e. VIII

b. V

The following are intervention strategies for Rancho levels ___: reorienting the pt, use of familiar objects in real life tasks, allowing pt time to respond or self-correct, beginning previous vocational or educational training asap, using community-based outings to work on psycho-social & cognitive skills, & improving strength, coordination, & endurance thru meaningful & purposeful activities. a. IV - V b. VI - VIII c. V - X d. VIII - X

b. VI - VIII

Ppl w/ ___ aphasia have a lesion in the left temporal lobe. a. Broca's b. Wernicke's c. conduction d. global

b. Wernicke's

___ aphasia is considered "fluent" aphasia & often involves strings of unintelligible words and neologisms. a. Broca's b. Wernicke's c. conduction d. global

b. Wernicke's

___ aphasia results in no auditory feedback when speaking & difficulty comprehending spoken language. a. Broca's b. Wernicke's c. conduction d. global

b. Wernicke's

A pt w/ ___ will be aware & can compensate after initial training & minimal cues. A pt w/ ___ has difficulty compensating no matter how many cues. a. neglect b. a field cut

b. a field cut a. neglect

Which of the following will NOT affect the Rancho level of a recovering pt on any given day? a. fatigue b. age c. stimulation d. familiarity & predictability of environment

b. age

Using the ASIA scale, at level L4, you test ___. a. plantar flexors b. ankle dorsiflexors c. hip flexors d. great toe extensors e. knee extensors

b. ankle dorsiflexors

Contralateral paralysis & sensory loss of (more) LE, appearance of a grasp or sucking reflex, lack of spontaneous behavior, motor inattention, perseveration, & amnesia are all effects of a stroke in the: a. middle cerebral artery (MCA) b. anterior cerebral artery (ACA) c. posterior cerebral artery (PCA)

b. anterior cerebral artery (ACA)

___ has an extremely poor prognosis for return of motor function (it's the injury you least want to have). a. central cord syndrome b. anterior cord syndrome c. Brown Sequard syndrome d. conus medularis lesion

b. anterior cord syndrome

___ is caused by flexion injuries that compromise the integrity of the anterior spinal artery, & by teardrop or burst fx's of the vertebral body. a. central cord syndrome b. anterior cord syndrome c. Brown Sequard syndrome d. conus medularis lesion

b. anterior cord syndrome

Speech ___ is the inability to program the sequence of movements for speech despite the absence of motor deficits (not a problem w/ the muscles). a. dysarthria b. apraxia c. anomia d. dysphagia

b. apraxia

___ is the loss of the same quadrant or half of vision in both eyes (i.e. R & L temporal fields). a. hemianopsia b. bilateral hemianopsia c. homonymous loss of field d. homonymous hemianopsia

b. bilateral hemianopsia

A lesion at the optic chiasm would cause what type of loss? a. R eye blindness b. bitemporal hemianopsia c. L homonymous hemianopsia d. upper quadrant hemianopsia e. homonymous hemianopsia w/ macular sparing

b. bitemporal hemianopsia

___ brain herniation occurs when it displaces into the ventricles. a. uncal b. central c. cingulate d. transcalvarial e. upward f. tonsillar

b. central

___ is a PNF technique where the pt isometrically contracts the tight muscle against the therapist, holds, then relaxes & then contracts the antagonist muscle (not tight) before the therapist stretches the tight muscle into the new range. a. hold relax or contract relax b. contract relax w/ antagonist contract c. hold relax swing/bounce

b. contract relax w/ antagonist contract

A ___ is an injury that produces an area of localized bleeding on the brain (macroscopic). a. concussion b. contusion c. diffuse axonal d. 2nd impact e. coup-contrecoup

b. contusion

The ___ transmits touch, proprioception, pressure, & vibration a. lateral corticospinal tract b. dorsal column c. lateral spinothalamic tract d. anterior spinothalamic tract

b. dorsal column

A(n) ___ hematoma is traumatic accumulation of blood b/w the inner table of the skull & the stripped off dural membrane. a. subdural b. epidural c. subarachnoid d. intraparynchemal

b. epidural

A(n) ___ hematoma usually occurs from a focused blow to the head & results in an overlying fx to the skull. Bc the underlying brain has been minimally injured, prognosis is excellent if tx'd aggressively. a. subdural b. epidural c. subarachnoid d. intraparynchemal

b. epidural

___ breaks down complex mvmts into discrete tasks & then practices them as a whole. The shift is away from impairments & a hands-on approach is taken to facilitate mvmt & active participation. a. compensation training b. functional/task-oriented training c. remediation training

b. functional/task-oriented training

Which of the following is NOT a predictor for recovery of arm function post-stroke? a. grade 3 or above on active finger extension scale of day 7 post-stroke b. hand sensation 11 days post-stroke c. presence of initial shoulder shrug 11 days post-stroke d. presence of synergistic hand mvmt 11 days post-stroke e. active ROM 3 weeks post-stroke, esp shoulder & middle finger flexion

b. hand sensation 11 days post-stroke

Inability to ER the humerus in order to avoid impingement is the fault of weakness in which mm post-stroke? a. supraspinatus b. infraspinatus c. teres minor d. subscapularis e. entire rotator cuff

b. infraspinatus c. teres minor

Inability to have downward glide of the HOH is the fault of weakness in which mm post-stroke? a. supraspinatus b. infraspinatus c. teres minor d. subscapularis e. entire rotator cuff

b. infraspinatus c. teres minor d. subscapularis

What method of ICP measurement is the most accurate & has the advantage of allowing for drainage of fluid? a. spinal tap b. intraventricular catheter c. subarachnoid screw/bolt d. epidural sensor

b. intraventricular catheter

Which of the following is NOT something to remember regarding an effective tenodesis grip? a. shortening FDP/FDS b. lengthening of FPL for thumb approximation to fingers c. avoid excessive stiffness of MPs & IPs d. encourage stiffness of IP for thumb for better pinch stability

b. lengthening of FPL for thumb approximation to fingers (should be shortening)

A lateral trunk lean is a gluteus ___ compensation that moves the GRFV closer to the hip's axis, decreasing the moment arm. a. minimus b. medius c. maximus

b. medius

A ___ brain injury involves a loss of consciousness for minutes to hours, confusion lasting days, & physical, cognitive, or behavioral impairments that may last for months or are permanent. a. mild b. moderate c. severe

b. moderate

A score of 9-12 on the Glasgow Coma Scale indicates a ___ brain injury. a. mild b. moderate c. severe

b. moderate

The ___ lobe contains the primary sensory areas for sensation of touch (contralateral), sensory association areas, & is considered the "where system". a. temporal b. parietal c. frontal d. occipital

b. parietal

Spasticity of which 2 mm causes shoulder pain in pts post-stroke? a. pec minor b. pec major c. subscapularis d. supraspinatus e. teres minor

b. pec major c. subscapularis (internal rotators)

The brain processes incoming information into ___. a. sensations b. perceptions c. cognition d. behaviors

b. perceptions

The following describes which phase of spinal shock: some return of polysynaptic reflexes. a. phase 1: 24-48 hrs b. phase 2: days 2-3 c. phase 3: 1-4 wks d. phase 4: 1-12 months

b. phase 2: days 2-3

Intervention for the cause of the dysfunction w/ expected transfer of training; practice & repetition of underlying processing skills. a. functional/adaptive intervention b. remedial/restorative intervention

b. remedial/restorative intervention

___ is a technique that involves a slow dynamic contraction of the antagonist followed smoothly by a slow dynamic contraction of the agonist (i.e. using the biceps then triceps). a. rhythmic initiation b. slow reversals c. slow reversal holds d. rhythmic stabilization

b. slow reversals

In the Functional Independence Measure (FIM), a score of 1 means ___ & a score of 7 means ___. a. complete independence b. total assist

b. total assist a. complete independence

Which of the following NOT a muscle a person w/ a C6 lesion has control of? a. rotator cuff, including subscapularis b. triceps c. ECRL d. serratus anterior

b. triceps

Using the ASIA scale, at level C6, you test ___. a. elbow flexion b. wrist extension c. elbow extension d. finger flexion e. finger abductors

b. wrist extension

compensations for weak hip ext

backward or forward trunk lean

compensation for glute max and hamstrings

backward trunk lean

jones vs ballerina fracture

base of 5th at least 1.5 cm distal to metatarsal styloid proximal tip of 5th

sports not recommended after hip dislocation

baseball basketball football jogging singles tennis squash hockey soccer volleyball

how does tape correct joint problems

by helping to improve Mal-alignment can provide support to joint structures, enhances kinesthetic awareness

where is the subgluteus Maximus bursa

beneath the tensor fasciae latae and gluteus maximus as they converge to form the iliotibial tract over the greater trochanter

total contact AFOs

better control light weight concealable hotter

where is the lisfranc joint

between metatarsals and cuneiforms, cuboid

where is chopart joint

between midtarsals, talus, navicular, calcaneus, and cuboid

what is a subarachnoid hemorrhage

blood leaks into the CSF CSF is between the delicate arachnoid membrane and the brain itself **critical

what is intraparenchymal hemorrhage

blood pools in white matter of the brain can result in white matter shear injury with extensive loss of axons

heterotrophic ossificans

bone formation at abnormal site

sever's disease

bone growth faster than muscle tendons can lengthen direct or microtrauma

why does osgood occur in kids

bone, muscle, tendon are still developing stress on quads causes additional stress on tibial tubersity

causes of abnormal pronation

bony abnormalities soft tissue abnormalities compensation for extrinsic abnormalities of foot (femoral ante torsion, internal tibial torsion)

treatment grade II ankle sprain

boot PRICED same rehab as Grade I return to function 2-6 weeks brace/tape for 6 months

a small infarct where will cause more severe damage because of condensed spinal tracts

brainstem

___ is caused by stabbing/gunshot wounds or other penetrating injuries, unilateral facet lock injuries, or burst fx's at the lateral body of the vertebrae. a. central cord syndrome b. anterior cord syndrome c. Brown Sequard syndrome d. conus medularis lesion

c. Brown Sequard syndrome

If an injury is incomplete, motor function is preserved below the neurological level & more than 1/2 of the "key mm" below that level score a grade of <3, it scores an ASIA level: a. A b. B c. C d. D e. E

c. C

If an injury is incomplete, there must be voluntary anal sphincter contraction or sacral sensory sparing w/ sparing of motor function more than 3 levels below the motor level for that side, in order to score an ASIA level of: (2) a. A b. B c. C d. D e. E

c. C OR d. D

A pt w/ a lesion at level ___ is totally dependent in bed mobility & usually dependent in transfers, but can assist w/ a sliding board. They usually use a hydraulic lift & body sling. a. C3 b. C4 c. C5 d. C6 e. C7

c. C5

Things to work on for injuries at the ___ level: more trunk control thru arm placement, independent arm placement/control in WB positions, tenodesis hand function, greater use of UE to increase ability to stabilize the trunk during ADLs. a. C4 b. C5 c. C6 d. C7 e. C8

c. C6

Things to work on for injuries at the ___ level: to use tenodesis for hand function, progressive exercise, wearing a wrist-driven tenodesis splint & a short opponens splint, and many of the interventions for the level above. a. C4 b. C5 c. C6 d. C7 e. C8

c. C6

Things to work on for injuries at the ___ level: independent control of trunk balance thru UE control, self-placement of arms to do push-ups, weight shifts, & bed mobility independently, and tenodesis hand function. a. C5 b. C6 c. C7 d. C8 e. T1

c. C7

Things to work on for injuries at the ___ level: progressive strengthening using tenodesis function to manage weight training devices, practice arm placement accuracy w/ increasing trunk challenges. a. C5 b. C6 c. C7 d. C8 e. T1

c. C7

At level ___ a pt lacks lumbricals & interossei, so finger function is impaired, but their arms are fairly normal. a. C6 b. C7 c. C8 d. T1

c. C8

Clients at ___ level & below are independent w/ all self-care, feeding, eating, & hygiene. Clients are independent w/ bowel & bladder care. Most are independent w/o equipment for all self-care needs. a. C6 b. C7 c. C8 d. T1

c. C8

Indicate the Rancho level: Localized response--demonstrates withdrawal or vocalization to painful stimuli; responds inconsistently to simple commands; responses are directly related to type of stimulus; may respond to some familiar ppl, but not others. a. I b. II c. III d. IV

c. III

The primary goals when working w/ a pt in Rancho level ___ are to increase attn span, consistency rate, & speed of response. a. I b. II c. III d. IV

c. III

Using simple, one-step directions, providing frequent orientation to time, day, & location, allowing the pt ample time to respond to instructions, & reassuring them that they are safe, are part of the tx process most appropriate at Rancho level: a. I b. II c. III d. IV

c. III

A lesion at the R optic tract would cause what type of loss? a. R eye blindness b. bitemporal hemianopsia c. L homonymous hemianopsia d. upper quadrant hemianopsia e. homonymous hemianopsia w/ macular sparing

c. L homonymous hemianopsia

These pts cannot withstand the rigors of acute rehab & their goals of therapy are to either go on to IP rehab or to home. It is either hospital-based or nursing care facility-based. It includes a comprehensive assessment to understand all aspect of performance in which setting? a. acute care b. LTAC c. SNF d. IP rehab e. home health f. OP

c. SNF

The ___ sling provides vertical support of the HOH & displaces weight via a strap over the opposite shoulder. a. Harris hemi b. Bobath c. Single strap hemi d. Roylan humeral cuff

c. Single strap hemi

___ is the leading cause of death & disability among young adults. a. stroke b. cancer c. TBI d. stupidity

c. TBI

At Rancho level ___ (confused, appropriate), the pt is inconsistently oriented to person, time, place; can do highly familiar tasks in non-distracting environments; able to follow simple directions consistently; appropriate verbal expressions in highly familiar & structured situations; unaware of impairments, disabilities, & safety risks. a. IV b. V c. VI d. VII e. VIII

c. VI

___ is the inability to recall the names of common objects. a. dysarthria b. apraxia c. anomia d. dysphagia

c. anomia

At levels C6 - C8, which of the following mm should you allow to be hypomobile? (multiple answers) a. hamstrings b. heelcords c. back extensors d. shoulders e. long finger flexors

c. back extensors e. long finger flexors

___ is how perceptions are used for immediate action or stored for future actions. a. sensations b. processes c. cognition d. behaviors

c. cognition

___ aphasia results when the arcuate fasciculus is involved. Speech production & auditory comprehension is usually intact, but pts can't repeat sentences. a. Broca's b. Wernicke's c. conduction d. global

c. conduction

A ___ injury is caused by shaking or strong rotation of the head. a. concussion b. contusion c. diffuse axonal d. 2nd impact e. penetration

c. diffuse axonal

Which of the following is NOT one of the key mm available at level C8? a. FDP & FDS b. ECU c. dorsal/palmar interossei d. FCU e. pollicis mm

c. dorsal/palmar interossei

Which of the following is NOT a risk factor for shoulder pain post-SCI? a. > 50 years old b. decreased PROM c. gender - female d. inability to initiate ROM w/in 1st 2 wks of injury e. all are risk factors

c. gender - female

Using the ASIA scale, at level L2, you test ___. a. plantar flexors b. ankle dorsiflexors c. hip flexors d. great toe extensors e. knee extensors

c. hip flexors

___ is a PNF technique where where the pt isometrically contracts the tight muscle against the therapist, holds, then relaxes &/or then contracts the antagonist muscle (not tight) before using dynamic/ballistic stretching. It's a risky technique. a. hold relax or contract relax b. contract relax w/ antagonist contract c. hold relax swing/bounce

c. hold relax swing/bounce

___ is loss of field (right or left, upper or lower) that is the same in both eyes. a. hemianopsia b. bilateral hemianopsia c. homonymous loss of field d. homonymous hemianopsia

c. homonymous loss of field

___ strokes occur in smaller penetrating "end" arteries, tend occur in ppl w/ multiple medical dx's in the circulatory system, and are the "best" type of stroke to have. a. thrombotic b. TIA c. lacunar d. embolic e. hemorrhagic

c. lacunar

The ___ transmits pain & temperature signals. a. lateral corticospinal tract b. dorsal column c. lateral spinothalamic tract d. anterior spinothalamic tract

c. lateral spinothalamic tract

Which of the following is NOT a muscle a person w/ a C1-C3 lesion has control of? a. SCM b. infrahyoids c. levator scapulae d. scalenes

c. levator scapulae

The following describes which phase of spinal shock: some return of monosynaptic reflexes & hyperreflexia due to axon-supported synapse growth. a. phase 1: 24-48 hrs b. phase 2: days 2-3 c. phase 3: 1-4 wks d. phase 4: 1-12 months

c. phase 3: 1-4 wks

Cortical blindness of the contralateral visual field, memory deficit, ataxia, & contralateral or ipsilateral hemiparesis are all effects of a stroke in the: a. middle cerebral artery (MCA) b. anterior cerebral artery (ACA) c. posterior cerebral artery (PCA)

c. posterior cerebral artery (PCA)

In strokes of the ___, the limbic system & memory can be greatly involved. If the damage reaches the pons or brainstem, the person usually dies. a. middle cerebral artery (MCA) b. anterior cerebral artery (ACA) c. posterior cerebral artery (PCA)

c. posterior cerebral artery (PCA)

For pts w/ feeding tubes, which position should you use the most caution with? a. supine b. side-lying c. prone d. hook lying

c. prone

Which of the following mm would ppl NOT have trouble activating post-stroke due to postural changes? a. glute max b. lats c. rectus abdominis d. back extensors

c. rectus abdominis

During the flexion synergy in the progression of tone, which of the following mm are usually high tone? a. triceps b. deltoid c. rhomboids d. upper trapezius e. biceps

c. rhomboids d. upper trapezius

A score of 8 or less on the Glasgow Coma Scale indicates a ___ brain injury. a. mild b. moderate c. severe

c. severe

Which of the following is/are challenging for someone w/ a C5 level SCI? a. shoulder ER b. elbow flexion c. shoulder IR d. shoulder elevation & scapular fixation e. shoulder flexion

c. shoulder IR d. shoulder elevation & scapular fixation (due to weak SA)

___ is a technique that involves a slow dynamic contraction of the antagonist followed smoothly by a slow dynamic contraction of the agonist, but also adds an isometric contraction at the extremes of range (or anywhere there is weakness). a. rhythmic initiation b. slow reversals c. slow reversal holds d. rhythmic stabilization

c. slow reversal holds

Mild tone in pts w/ TBI is managed w/ which of the following? a. casting b. dynasplints c. splints d. a & b e. b & c

c. splints

A(n) ___ hemorrhage is when blood leaks into the CSF. Pts describe this as "the worst headache of their lives". a. subdural b. epidural c. subarachnoid d. intraparynchemal

c. subarachnoid

Which of the following is true regarding sling use & choice? a. they have a lasting impact on existing subluxation b. they help address the scapular/trunk alignment that is the primary cause of subluxation c. they prevent stretching of the joint capsule when the supraspinatus is inactive d. they can be used universally with pts post-stroke

c. they prevent stretching of the joint capsule when the supraspinatus is inactive

___ is the most common type of stroke. a. embolic b. hemorrhagic c. thrombotic d. they are equally likely

c. thrombotic

features of severe pes cavus

calcaneus can't pronate past 5 degrees varus, heel in varus, foot in valgus

Rearfoot valgus

calcaneus in EVERTED in subtalar neutral

Rearfoot varus

calcaneus is INVERTED relative to lower leg in subtalar neutral

treatment for grade III ankle sprain

casting/brace >6 weeks 1-4 years before symptom free

hemorrhagic stroke (12%)

caused by AV malformation, aneurysm, head injury, weakness of arterial wall Oval shaped clot (four major sites: putamenal, thalamic, pontine, cerebellar) Hypertensive Aneurysm

Subtalar pronation

causes the MTJ axes to become more parallel.

Subtalar supination

causes the MTJ axes to become non-parallel (more perpendicular)

when you weight bare their is a small ____ glide of the fibula

cephalic

if SBP rises

cerebral vessels constrict

if SBP falls

cerebral vessels dilate to allow better flow

vital sign monitors

check BP, HR, O2 sats, RR, heart waves monitor changes with mobility and function

who can tolerate higher pressure for longer times

children

garden stage classification terminology type 4

complete fracture with full displacement, continuity of fragments disrupted (usually addressed via THA).

signs of trochanteric bursitis -pain -tenderness -reproducers of pain -similar to

chronic, intermittent, aching pain over the lateral hip. Walking or lying on the affected side exacerbates the pain. In the seated position, local tenderness is present over the greater trochanter or more posteriorly for deep bursa. reproduced by hip adduction (superficial bursitis) or resisted active abduction (deep bursitis). symptoms similar to L-5 radiculopathy pain could radiate along an L-2 distribution to the knee Swelling and tenderness to palpation over the trochanteric bursa aching pain after running due to overuse SLR or femoral nerve stretch could be painful.

what exercise to not do with femoral anterior glide with lateral rotation

clamshells

toe deformities and their causes

claw- weak lumbrical/interosseous, pes cavus, fallen metatarsal arch, neuro hammer- interosseous, hereditary, poor shoes, hallux vagus mallet- poor shoes

embolism (38%)

clot commonly caused by A-fib or MI 80% middle cerebral a. Occur rapidly and more severe

____ is how perceptions are used for immediate action or stored for future action

cognition

what is Charcot foot

collapse of arches from poor circulation and vulnerable tissue contributing factor is insensate feet

what is compression screw-plate

commonly called the Dynamic Hip Screw. This is often used for extracapsular fractures of the proximal femur. The screw grips the femoral head, then slides telescopically into the barrel. Tightening of a screw in the base of the barrel creates compression across the bone fragments.

explain third degree strain

complete tear

double vision

constant or intermittent if you block one eye, it goes away damage to the six external muscles of the eye

historical clues: MCL

contact with pop lateral blow to knee

historical clues: LCL

contact with pop medial blow to knee

goals of ranchos level III

continue levels I and II goals bed mobility increase arousal and speed of response frequent orientation

the right parietal lobe has a ____ and ____ orientation

contralateral ipsilateral

other ligaments of meniscus

coronary ligaments (meniscotibial)- attaches horns to the tibia transverse ligament- anterior, runs horizontal, attach medial and lateral meniscus together

bilateral arm training activates the damaged hemisphere via the ____

corpus callosum

well leg test

crossed SLR

At which 2 levels do pts use a tenodesis grip? a. C3 b. C4 c. C5 d. C6 e. C7 f. C8

d. C6 e. C7 (a little at C8 bc even w/ finger flexors, the grip isn't strong due to lack of intrinsics til T1)

Things to work on for injuries at the ___ level: progressive activity/exercise (adapted writing, putty), small hand-based splint to support arches as needed, & meaningful & functional fine motor coordination activities. a. C5 b. C6 c. C7 d. C8 e. T1

d. C8

Things to work on for injuries at the ___ level: trunk stability thru arm use, natural (if limited) hand function, advancing PNF patterns, occupational sitting tasks, & bilateral UE tasks from WC level. a. C5 b. C6 c. C7 d. C8 e. T1

d. C8

If an injury is incomplete, motor function is preserved below the neurological level & at least 1/2 of the "key mm" below that level score a grade of >=3, it scores an ASIA level: a. A b. B c. C d. D e. E

d. D

Pts in this setting are medically stable & ready for intensive therapy. They receive 3 hrs of therapy daily w/ the goal of highest level of performance necessary to go home. There's a comprehensive assessment of their ability to perform all ADLs & IADLs. a. acute care b. LTAC c. SNF d. IP rehab e. home health f. OP

d. IP rehab

The IRF-PAI, which is based on the FIM, is an assessment that would be administered at the time of admission into which rehab setting? a. acute care b. LTAC c. SNF d. IP rehab e. home health f. OP

d. IP rehab

At Rancho level ___ (purposeful, appropriate SBA on request), they can independently shift b/w tasks accurately for 2+ hrs, initiate/carry out familiar tasks independently & unfamiliar ones w/ SBA; accurately estimate abilities, but uses SBA to adjust demands, anticipate probs or take action to avoid it; depression may continue due to awareness of their deficits. a. VI b. VII c. VIII d. IX e. X

d. IX

If the ___ are not firing appropriately during stance, the pt can't used a hinged AFO. a. DFs b. quads c. hamstrings d. PFs

d. PFs

The ___ sling provides vertical support of the HOH w/ the force enveloping the humerus & pulling it superiorly. The force is transmitted to the opposite trunk. It allows functional use of the arm but is very difficult to don. a. Harris hemi b. Bobath c. Single strap hemi d. Roylan humeral cuff

d. Roylan humeral cuff

Respiration is usually impaired to some degree in pts w/ SCI above level ___. a. C5 b. T1 c. T6 d. T12

d. T12

At Rancho level ___ (automatic, appropriate), the pt is consistently oriented to person & place & only needs mod cues for time; can perform new learning w/ minimal assistance; has shallow recall of what they've been doing after completion of a familiar task; superficial awareness of their condition, but not aware of specific deficits or impact on family; unrealistic future planning. a. IV b. V c. VI d. VII e. VIII

d. VII

Moderate to severe tone in pts w/ TBI is managed w/ which of the following? a. casting b. dynasplints c. splints d. a & b e. b & c

d. a & b

The ___ transmits touch and pressure signals. a. lateral corticospinal tract b. dorsal column c. lateral spinothalamic tract d. anterior spinothalamic tract

d. anterior spinothalamic tract

Arousal, awareness, & sleep disturbances are ___ impairments associated w/ TBI. a. motor b. sensory c. cognitive d. autonomic

d. autonomic

Which of the following is NOT a risk factor for stroke? a. HTN b. smoking c. obesity d. contraceptives w/ high levels of progesterone e. all are risk factors

d. contraceptives w/ high levels of progesterone (should say estrogen)

___ results in flaccidity & a lack of return of bowel, bladder, & sexual function. a. central cord syndrome b. anterior cord syndrome c. Brown Sequard syndrome d. conus medularis lesion

d. conus medularis lesion

___ is difficulty swallowing due to weakness, tone, motor incoordination of the oral structures. It results in compromised nutrition & hydration, & potentially life-threatening aspiration. a. dysarthria b. apraxia c. anomia d. dysphagia

d. dysphagia

Which of the following is NOT a method of managing respiratory complications? a. put them in an upright position b. retrain breathing patterns for CV endurance c. teach alt methods of coughing d. encourage shallow breathing

d. encourage shallow breathing (should be deep breathing)

Using a ___ is a less invasive way to monitor ICP, but no fluid can be drawn from it. a. spinal tap b. intraventricular catheter c. subarachnoid screw/bolt d. epidural sensor

d. epidural sensor

Using the ASIA scale, at level C8, you test ___. a. elbow flexion b. wrist extension c. elbow extension d. finger flexion e. finger abductors

d. finger flexion

Cognitive deficits are most associated w/ lesions in the ___ lobe. a. parietal b. temporal c. occipital d. frontal

d. frontal

___ aphasia results from extensive damage to the frontal & temporal areas of the brain. It involves a nearly complete loss of both speech production & comprehension. a. Broca's b. Wernicke's c. conduction d. global

d. global

Using the ASIA scale, at level L5, you test ___. a. plantar flexors b. ankle dorsiflexors c. hip flexors d. great toe extensors e. knee extensors

d. great toe extensors

___ is the loss of the nasal field in one eye & the temporal field in the other, resulting in the loss of an entire visual field. a. hemianopsia b. bilateral hemianopsia c. homonymous loss of field d. homonymous hemianopsia

d. homonymous hemianopsia

A(n) ___ hemorrhage is when blood pools in the white matter of the brain. It can cause diffuse axonal injury. a. subdural b. epidural c. subarachnoid d. intraparynchemal

d. intraparynchemal

For what might you use a taping job called the "tri-pull method" on a pt post-stroke? a. inhibit overactive muscle synergies b. facilitate underactive muscle synergies c. optimize joint alignment d. offload irritable tissue

d. offload irritable tissue

The following describes which phase of spinal shock: hyperreflexia & spasticity due to soma-supported synapse growth. a. phase 1: 24-48 hrs b. phase 2: days 2-3 c. phase 3: 1-4 wks d. phase 4: 1-12 months

d. phase 4: 1-12 months

___ is a technique that involves isometric contraction of the antagonist, followed by isometric contraction of the agonist anywhere in the ROM of a pattern. It's good for improving strength, stability, & motor learning. a. rhythmic initiation b. slow reversals c. slow reversal holds d. rhythmic stabilization

d. rhythmic stabilization

Which of the following NOT a muscle a person w/ a C4 lesion has control of? a. trapezius b. levator scapulae c. diaphragm d. teres minor

d. teres minor

Generally speaking, as the person becomes less acute & the person is coming to the end of their rehab intervention, i.e. from acute hospitalization to home health, selection of assessments (& intervention) should be: a. bottom up & impairment/remedial-based b. top down & impairment/compensatory-based c. bottom up & disability/compensatory-based d. top down & disability/compensatory-based

d. top down & disability/compensatory-based

___ brain herniation occurs when it displaces thru any opening in the skull made by the impact. a. uncal b. central c. cingulate d. transcalvarial e. upward f. tonsillar

d. transcalvarial

A lesion at the R optic radiation would cause what type of loss? a. R eye blindness b. bitemporal hemianopsia c. L homonymous hemianopsia d. upper quadrant hemianopsia e. homonymous hemianopsia w/ macular sparing

d. upper quadrant hemianopsia

Which of the following is NOT a take-home point on GH subluxation? a. it prolongs rehab b. associated w/ reduced ROM c. impedes balance d. usually has a single, definite cause e. causes dificulty with transfers & mobility

d. usually has a single, definite cause

when should Pts with OA get surgery

decline of treatment benefit further disease progression radiograph of joint space

restrictions of hip OA

decreased IR, AB, and flexion slowed gait decreased proprioception

everything that the left brain controls

detail oriented compare/contrast the environment process info sequentially right visual field right hemiparesis right sensory loss aphasia apraxia of motor planning left/right confusion compulsiveness slowness language understanding anxiety depression brocas werneckes conduction aphasia global aphasia

what is DDH

developmental dysplasia of the hip Abnormal development of the hip, can result in subluxation or dislocation

dysphagia

difficulty swallowing compromised nutrition and hydration possible aspiration

managing tone

dynasplint or casting for moderate to severe splint for mild low load and prolonged stretch

what joint is often involved in high ankle sprains

distal tibiofibular joint

right sided stroke decreases arousal and attention due to decreased ____

dopamine

Lasegue's sign

dorsiflexing the foot during straight leg raise neural test

closed packed position of the talocrural joint

dorsiflexion

test for tarsal tunnel syndrome

dorsiflexion-eversion test

factors effecting the recovery from coma

duration length of post-traumatic amnesia age

At Rancho level ___ (purposeful, appropriate, modified independent), they are socially appropriate in all settings, can use metacognition, can handle multiple tasks simultaneously in all environments, tho may need breaks; periods of depression may occur, irritability & low frustration tolerance when sick, fatigued &/or under emotional stress. a. VI b. VII c. VIII d. IX e. X

e. X

Which of the following is NOT a cause of shoulder pain post-SCI? a. neurtitic pain from a nerve root injury b. radicular pain w/ parasthesias c. phantom sensations d. reflex sympathetic dystrophy e. all are causes

e. all are causes

Which of the following is NOT a risk factor for stroke? a. age b. gender c. race d. heredity e. all are risk factors

e. all are risk factors

Which of the following is NOT a non-verbal pain indicator? a. agitation/aggression b. resistance to care c. facial grimacing/wincing d. bracing or rocking e. all of the above are indicators

e. all of the above are indicators

Which of the following is NOT a likely drug a pt w/ a brain injury might be taking? a. anti-seizure b. diuretic c. hyperosmotic d. anti-hypertensives e. all of the above are likely to be taken

e. all of the above are likely to be taken

Loss of compressive muscular force that keeps the HOH in the glenoid is the fault of weakness in which mm post-stroke? a. supraspinatus b. infraspinatus c. teres minor d. subscapularis e. entire rotator cuff

e. entire rotator cuff

Using the ASIA scale, at level T1, you test ___. a. elbow flexion b. wrist extension c. elbow extension d. finger flexion e. finger abductors

e. finger abductors

Tx for ___ strokes includes lowering arterial BP & surgery to remove the clot & decrease intracranial pressure. a. thrombotic b. TIA c. lacunar d. embolic e. hemorrhagic

e. hemorrhagic

___ strokes are caused by AV malformations, weakness of arterial walls, aneurysms, or head injuries. a. thrombotic b. TIA c. lacunar d. embolic e. hemorrhagic

e. hemorrhagic

OASIS is a stroke assessment that would be administered at the time of admission into which rehab setting? a. acute care b. LTAC c. SNF d. IP rehab e. home health f. OP

e. home health

This setting is the next step toward community reintegration & progress is expected to continue w/ the support/assistance of the family/caregiver. Assessments should center around the tasks that are important to the person & environment. a. acute care b. LTAC c. SNF d. IP rehab e. home health f. OP

e. home health

A lesion at the R striate cortex would cause what type of loss? a. R eye blindness b. bitemporal hemianopsia c. L homonymous hemianopsia d. upper quadrant hemianopsia e. homonymous hemianopsia w/ macular sparing

e. homonymous hemianopsia w/ macular sparing

Using the ASIA scale, at level L3, you test ___. a. plantar flexors b. ankle dorsiflexors c. hip flexors d. great toe extensors e. knee extensors

e. knee extensors

Pts w/ SCI often have problems w/ all of the following EXCEPT: a. heart rate b. blood pressure c. sweating d. temperature regulation e. they have prob w/ all of the above

e. they have prob w/ all of the above

___ brain herniation occurs when the cerebellum ascends thru the tentorial opening. a. uncal b. central c. cingulate d. transcalvarial e. upward f. tonsillar

e. upward

The clinical prognosis of Brown Sequard syndrome is: a. very poor b. poor c. fair d. good e. very good

e. very good

The clinical prognosis of cauda equina lesions is: a. very poor b. poor c. fair d. good e. very good

e. very good

signs/symptoms of achilles rupture

early 40s happened at push off, sudden DF with FWB during fall, or violent DF when jumping immediate pain, trouble walking, snap, swelling

muscles needed terminal swing

eccentric hamstrings concentric DF

symptoms of grade I West Point sprain

edema ecchymosis ambulate with stability ATFL tender 60-70% involved ATFL + anterior drawer

symptoms of grade II West Point sprain

edema ecchymosis may not be able to ambulate normally ATFL and CFL tender to palpation 20% of all ankle sprains involve ATFL & CFL

symptoms of grade III West Point sprain

edema ecchymosis unable to WB joint unstable foot drops and supinates ATFL, CFL, and PTFL tender to palpation

how to support muscles with tape

elastic properties replicate and enhance muscle fxn improves contraction and relaxation influence Golgi tendon organs

ASIA C7

elbow extensors

ASIA C5

elbow flexors

edema management

elevate pillow to 45 degrees or greater A/PROM resting hand splint

for better obversation of gait, do what four things

focus on slow moving segments proximal to distal stance phase then swing sagittal plane and then frontal

what can put valgus stress on the knee, injuring the ACL

foot pronation body and trunk leaning laterally IR of femur

tibialis anterior compensation

foot slap foot flat at initial contact

People at L1-3 may ambulate for exercise and in their household and will need what devices

forearm crutches

People at T10-12 may ambulate for exercise and in their household and will need what devices

forearm crutches or a walker

People at L4-5 may ambulate for exercise, in their household, and limited in the community and will need what devices

forearm crutches, axillary crutches, or 2 canes

Forefoot varus

forefoot in SUPINATED in respect to rearfoot in subtalar neutral

Forefoot valgus

forefoot is PRONATED in respect to rearfoot in subtalar neutral

turf toe

hyperextension sprain with compressive load

non-compensation for glute max and hamstrings

forward trunk lean hip is open, knee hyperextension, don't get CPG of hip ext

compensation for quads

forward trunk lean, knee hyperextension without forward trunk line, avoiding heel strike

People who ambulate at L1-3 will use what kind of gait pattern

four point (sometimes 2 point)

what is a basilar skull fracture

fracture is located at the base of the skull; raccoon eyes and Battle's sign, may leak CSF through eyes and nose

osteochondral fracture

fracture of bone near joining with articular cartilage

ogden and Murphy group 2

fracture separation of the whole tuberosity with displacement of the fractured segment; whole tubercle breaks off

concrete red flag screenings

fracture- Ottawa knee rules cancer infection DVT compartment syndrome vascular claudication

Ogden and Murphy Group 1

fractures are minor; distal and undisplaced

inflammatory mediators compartment syndrome

from ischemic skeletal muscles increase capillary permeability activate coagulation cascade

Ia afferents

from proprioceptors in hip extensor muscles regulate transition from stance to swing phase.

Ib afferents

from the Golgi Tendon Organ in plantarflexors inhibit motorneurons at rest • When PF are stretched, they are inhibited in order to allow DF to work to take a step

cognitive deficits are associated with lesions of the ___ lobe

frontal

global aphasia

frontal and temporal areas loss of ability to comprehend language and formulate speech some automatic speech expressions

activation of glute med frontal and transverse

frontal- eccentric AB transverse- isometric IR

Ranchos level II

generalized Response- reflex response to painful stimuli, response to auditory stimuli is increased or decreased, response to external stimuli with generalized physiological changes, delayed responses

myositis ossificans progressive

genetic disease

growth and resultant ship of the skeleton are affected by

genetics nutrition mechanical forces

cranial nerve IX and X tests

glossopharyngeal and vagus listen to voice swallow "ah" gag reflex

Treatment of Legg-Calve-Perthes Disease

goal is to maintain femoral head shape and containment of femoral head in acetabulum

why is standing good in ICU

good for briefly normalizing muscle tone and stretching joints

symptoms of plantar fasciatis

heel pain medial side pain worse when first wake up and at night

problems with rear foot varus

heel strike more lateral STJ pronates more

what is a bakers cyst

herniation of posterior joint capsule through defect in the semimembranosis fascia - associated with medial meniscus tear self-contained, popliteal fluid-filled cyst

Pes cavus

higharched foot, supinated foot

one of the few joints in the body that the closed packed position is not associated with maximal joint congruency

hip

non-compensation for gluteus medius

hip drop (on strong side) during SLS

ROM needed terminal stance

hip extension 30 knee 0 ankle ?

anterior tilt muscles short

hip flexors and back extensors

muscles active in terminal stance

hip flexors eccentric knee none ankle PF isometric hip AB isometric ankle supinators concentric

muscles active in midstance

hip flexors eccentric no knee ankle PF eccentric hip AB isometric ankle supinators concentric

muscles need preswing

hip flexors iso, then concentric knee none ankle PF concentric NO FRONTAL PLANE

ROM needed preswing

hip neutral knee 40 flex ankle 20 PF

transient synovitis

hip pain w/ limp following URI or other illness; can lead to Leg-Calve resolves in 7 days

devices for internal fixation

hip pin compression plate IM nail compression screw-plate

yellow flag screening

hip referral neuro psychosocial factors

red flags for osteonecrosis of femoral head

history of long-term corticosteroid use history of avascular necrosis of contralateral hip trauma

tests for vagus and glossopharyngeal nerves

hoarse or nasally voice swallow say "ah" gag reflex

type 1 salter harris fracture

horizontal fracture through the growth plate

what is an equivalent diagnosis to hip OA

hypo mobility with superior glide

cranial nerve XII tests

hypoglossal articulation of words stick tongue out

Treatment of Osgood-Schlatter

ice rest decrease activity avoid squat/jump knee brace stretch hams/quads

three types of scoliosis

idiopathic congenital neuromuscular

extra capsular ligaments

iliofemoral, ischiofemoral, and pubofemoral ligaments

largest bursa in the body

iliopsoas

sully stabilizer

immobilization multi-directional instabilities rotator cuff strains AC separations

sesamoiditis

impact, overpronation, or great toe injury

Quadriplegia (tetraplegia)

impairment or loss of motor and / or sensory function in the upper and lower extremities, trunk, and pelvic organs. result of lesion in cervical cord

paraplegia

impairment or loss of motor and/or sensory function due to damage in the thoracic, lumbar, or sacral segments of the spinal cord. Function may be impaired in the trunk and/or lower extremities.

femoral anterior glide syndrome at the hip is often associated with ____ and mis-diagnosed as _____

impingement iliopsoas tendonitis

in ____ deficient knee, MCL provides the most anterior stability

in ACL deficient knee

when does DDH occur

in utero or after birth

apraxia of speech

inability to program the sequence of movements for speech despite absence of motor deficits o Difficulty with initiating the movement necessary to speak

somatagnosia

inability to recognize body parts

vertical fracture of patella

indirect blow to patella (kick or force that hits knee and then skips off)

what is bursitis

inflammation of a bursa; the term is often misused to describe pain experienced at the hip. If true inflammation of the bursa exists, it is often related to repetitive motions and associated with muscles that are too short/inflexible. Direct trauma to the bursa may also cause inflammation, however this is fairly rare at the hip and pelvis, with one exception (ischial bursitis).

x-ray changes for OA

joint space narrowing subchondral sclerosis osteophytes cysts

tibial tubercle fractures occur more commonly in ____

kids

ASIA L3

knee extensors

AFO relative contraindications

knee flexion contracture significant ankle motion is required fixed deformity of ankle open wound

compensations with limited dorsi

knee hyperextension forward trunk shortening of opposite step

how to tell tibial vs femoral antetorsion

knee point straight = tibial knees point in = femoral

topographical orientation

knowing ones location in a larger space

DD tests for ACL

lachman test active lachman anterior drawer

symptoms of cuboid syndrome

lateral mid foot pain unable to run/jump "walking on pebbles"

DD for anterolateral knee instability test

lateral pivot shift test

compensation of weak hip AB

lateral trunk lean cane in opposite hand

compensation for gluteus medius

lateral trunk lean in loading and midstance

higher ICP leads to less ____ or a lower ____

less cerebral perfusion lower CPP (cerebral perfusion pressure)

to be in in-patient rehab, what RLA level must you be at

level 3-4 with quick recovery

where is the iliopsoas bursa

lies between the iliopsoas tendon and the lesser trochanter, extending upward into the iliac fossa beneath the iliacus muscle

intracapsular ligaments

ligamentum teres transverse acetabular

sequence of motor recovery following stroke

limb or trunk is flaccid initial tone marked spasticity recruit flexion or extension spasticity evident with rapid or difficult movement coordination and patterns of movement nearly normal normal

job of the PCL

limit posterior translation of tibia on femur

KAFO precautions/contraindications

limited energy reserves limited strength of trunk and UE

KAFO indications

limited or no control at knee and foot misalignment of knee requires control at hip and trunk

two primary functions of the menisci

load-bearing stability in absence of other primary structures

what is a contusion

localized bleeding on the brain (macroscopic)

signs of stress fractures

localized tenderness WB painful percussion sign

motor 5 glasgow

localizes stimulus

transverse acetabular ligament

located inferiorly and medially along the rim of the acetabulum, this ligament spans the acetabular notch, which reinforces the labrum in this region

ischiofemoral ligament

located posteriorly, it attaches proximally to the ischium and to the acetabulum. It resists extension and internal rotation can resist posterior movement of femur with hip flexion

long plantar ligaments

long cacaneocuboid superficial long plantar

percutaneous endoscopic gastrostomy

long-term nutrition

features of mild pes cavus

longitudinal arch high NWB, near normal WB, toes claw NWB, may have hind foot varus

features of moderate pes cavus

longitudinal arch high, calluses under prominent metatarsal heads, limited dorsi

intracondylar fractures

longitudinal stress (fall landing on feet) CPM may be utilized if articular surface is involved

metatarsal disarticulation deficits

loss of anterior lever arm and balance

ray resection

loss of balance and weight-bearing surface

lisfranc amputation deficits

loss of balance and weight-bearing surface disruption of anterior compartment insertions equinas deformity

osteochondritis dissecans

loss of blood flow to subchondral bone leading to separation/instability of segment of cartilage and free movement within joint space

great toe amputation

loss of push-off and anterior level arm steel shank/carbon fiber insert

verbal 2 glasgow

make unintelligible sounds

hallmarks of PNF

maximal resistance moving in diagonal patterns quick prolonged stretch timing traction approximation verbal cues visual cues

MOI for muscle strains

maximum contraction before the muscle is ready force generated exceeds the muscles ability to withstand force

disability rating scale

measure of change over the course of recovery not for mild TBI

clinical signs of rear foot varus

medial bunion hammer toes heel callus

GRFV for the gluteus medius

medial to hip joint AD moment

what is compression plate fracture fixation

metal plates and screws. At times the metal plate is removed following fracture healing, because the plate alters the normal weight bearing pattern of the bone and increases the potential for future fracture. If the plate is removed, there is a temporary weakening of the bone where the screws were removed, and weight bearing must be limited for a few weeks.

Hallux valgus

metatarsophalangeal joint enlarged and permanently laterally displaced

what artery: UE involvement

middle

what artery: hemianopia

middle

what artery: limb kinetic apraxia

middle

what artery: broca's or wernecke's

middle (left superior or left inferior)

Broca's or Wernicke's

middle cerebral

hemianopia

middle cerebral

limb-kinetic apraxia

middle cerebral

why is the medial meniscus more likely to be injured

more heavily bonded to other structures

major pelvis fracture

more than one break in the pelvic ring and include displaced sacral injuries, and may involve internal fixation. Major fractures signify potential pelvic instability or serious associated complications. Typically require surgical fixation

synaptogenesis is enhanced through

motor learning

Thrombosis (50%)

narrowing, occur where arteries branch Exacerbated by HBP Most occur during sleep Usually due to HTN, diabetes, vascular disease Around the area of anoxia Need t-PA within 4.5 hours (need to make sure it is not hemorrhagic) May need intra arterial thrombolysis, stent retriever

what is hypertonicity

nerve input to muscle is increased, greater resting tone

motor 1 glasgow

no response

vegetative state

no signs of awareness, may have some arousal regulate their own HR and breathing no meaningful response, emotions, or cognitive function

historical clues: ACL tear

non contact with pop knee gave out acute swelling

fluids of the brain are ____ so once pressure begins to build ____ increases rapidly

non-compressible ICP

MOI for patellar subluxation or dislocation

non-contact (pivot, cut, twist) valgus stress with strong quad contraction bony anomalies biomechanics predisposition

What is arthrogryposis multiplex congenita?

non-progressive NM syndrome characterized by severe joint contractors, muscle weakness, and fibrosis

Arthrogryposis Multiplex Congenita

non-progressive neuromuscular syndrome severe joint contracture, muscle weakness, fibrosis

gastroc compensatoins

o Lack of forward propulsion (push-off) o Overuse of hip flexors (if available) (pull-off) o Short step length with swing on the stronger leg and subsequent landing on the stronger leg o Inadequate knee flexion in swing phase (from lack of push-off)

outcomes of right side stroke

o Left hemiparesis o Left sensory loss o Visuospatial impairment o Poor body scheme o Poor attention o Neglect Syndrome o Memory problems for procedures o Time disorientation o Problem solving o Poor Awareness o Impulsiveness/safety o Concrete thinking

jobs of the temporal lobe

o Left- language, interpretation, understanding o Right- sounds, rhythm, visual performance, affective expression

jobs of the frontal lobe

o Motor- primary motor (hemiparesis), pre-motor (motor planning), motor association area o Executive cognitive skills- judgment, reasoning o Personality- emotions, motivation, inhibition/social skills o Expressive speech (Broca's)

phases of spinal shock

o Phase One: Complete loss or weakening of all reflexes below the level of injury. Usually lasts 24-48 hours. No motor or sensation at all. o Phase Two: Occurs over days 2 - 3. There is beginning return of reflexic activity. o Phase Three: Hyperreflexia - due to axon-supported synapse growth - from days to four months. Motor or sensory tracts that are viable may begin to show recovery. o Phase Four: Full spasticity is due to soma-supported synapse growth - from days to four months. Motor or sensory tracts that are viable may begin to show recovery.

jobs of the parietal lobe

o Primary sensory areas- receive/process sensation of touch o Left- R/L discrimination, praxis (putting together the motor plan) o Right- visuospatial orientation

outcomes of left sided stroke

o Right hemiparesis o Right sensory loss o Aphasia with decreased reading & writing o Apraxia/motor planning o Left/right confusion o Compulsiveness o Slowness

jobs of the occipital lobe

o Synthesis and integration of visual information o Visual memory o Formation of visuospatial relationships o Visual reception L- only attends to right field R- attends to both fields

motor 6 glasgow

obey command

cortical blindness is from the ____ lobe

occipital

lacunar stroke

occur in smaller penetrating end arteries

what is a subdural hematoma

occurs under the due but OUTSIDE the brain and arachnoid membrane **most common type of hematoma

Segond fracture

occurs with severe rotary stress and concurrent ACL disruption potential avulsion of Anterior Lateral Ligament

red flags for femoral neck fracture

older women with hip, groin, thigh pain history of falls from standing pain worse with movement shortened and ER LE

cranial nerves

olfactory, optic, oculomotor, trochlear, trigeminal, abducens, facial, vestibulocochlear, glossopharyngeal, vagus, accessory, hypoglossal

metatarsalgia

pain over met heads pes cavus, tight achilles, hyper mobile foot

signs/symptoms for plantar fasciitis

pain with WB pain at medial calcaneal tubercle pain with DF great toe DF limited

signs and symptoms of a quad strain

pain with resisted knee extension and passive stretch swelling location palpable defect muscle spasm

symptoms of arthritis

pain, stiffness, swelling crepitus, deformity, instability, loss of function

People at T1-9 may ambulate for exercise only and will need what devices

parallel bars, forearm crutches, or a walker

explain second degree strain

partial tear, significant early functional loss Decelerating limb, insufficient warm-up, lack of flexibility

transverse fracture of patella

patella receives a relatively minor blow while the quadriceps is strongly contracting a separation of the superior and inferior fragments.

what is PEAK

pelvis in neutral equal weight on both sides angles knees facing forward

hip during pre-swing

pelvis rotated back hips extended STJ maximal supination CONCENTRIC

what is considered a vulnerable foot that is at risk for amputation

peripheral vascular disease mechanical stresses causing structural breakdown neuropathy (sensory, motor, autonomic) wound care issues

three types of achilles tendonitis

peritendinitis- possible crepitus with long standing injury, above insertion tendinosis- focal area of degeneration; weekend warrior both together

possible associated injuries with proximal fibula fracture

peroneal nerve palsy, leading to drop foot

persistent and permanent vegetative state

persistent- vegetative for 4 weeks permanent- 6 months for non-traumatic or 12 months for traumatic

mild brain injury criteria

person is confused brief loss of consciousness tests/scans normal **concussion

memory deficit, ataxia, contralateral sensory loss

posterior cerebral

GFRV quads- loading

posterior to knee, use eccentric extension

visual cognitive components

posterior visual pathway thalamus, primary visual cortex, occipital, parietal, temporal, and frontal lobes

popliteus tendon attaches to ______

posterolateral corner of lateral meniscus

semimembranosus fascia attaches to the ____

posterolateral corner of the medial meniscus

key findings to indicate femoral anterior glide syndrome with medial rotation

postural deviations SLS- pelvis drop on stance side supine hip flex and ER- stiff Supine SLR- femur head goes naterior sidelying AB- weak medium sidelying ER- difficult prone knee flexion prone hip extension- hams dominant sitting hip rotation quadruped rock-back swayback pelvis torque to affected side (ASIS forward)

key findings to indicate multidirectional accessory hypermobility

postural deviations supine SLR- + anterior glide of femur sidelying ER prone knee flexion- causes femoral ER prone hip ext- painful, hip ER sitting hip rotation- less deep inguinal crease quadruped rock-back- affected hip goes lower

key findings to indicate femoral posterior glide syndrome with IR

postural deviations- deeper inguinal crease on affected side supine hip flex and ER- hypermobile hip supine SLR- negative anterior glide sidelying ER prone knee flexion prone hip ext- hamstring dominance sitting hip rotation- weak ER muscles quadruped rock-back- affected hip low, head of opposite femur prominent deeper inguinal crease

TIA

precursors of stroke 35% will progress to stroke in 5 years Brief focal loss of function Full recovery in 24 hours Probably due to ischemia Plaques cleared by endarterectomy or angioplasty with stinting

sequential compression devices

prevent DVT prevent blood stasis ankle 45-50mmHg mid thigh 30mmHg

LCL: primary restraint to ____ secondary restraint to ___ significant restraint to ___ when combined with other lateral structures

primary: adduction secondary: anterior/posterior tibial displacement (when other ligaments damaged) significant: restraint to ER of tibia

ACL: primary restraint to ____ contributes the most at ___ degrees secondary restraint to ____

primary: anterior tibial translation 30 degrees flexion secondary: internal tibial rotation prevent knee hyperextension resists AB and AD and full extension

PCL: primary restraint is ____ secondary restraint is ___

primary: posterior translation of tibia secondary: ER of tibial at 90 flex, reduces upon knee extension

subtalar joint movement during gait cycle

pronates during loading supinates throughout mid/terminal stance reaches neutral by heel off

Ely's test

prone knee bend AKA femoral nerve stretch

treatment of acute injuries

protect healing tissue pain control minimize disuse sequelae education address psychosocial factors

how to activate endogenous analgesic system with tape

provide sensory stimulation to skin receptors during movement

single strap hemi-sling

provide upward vertical support

rancho los amigos level of recovery

provides a mean of determining various levels of recovery following head injury useful to gauge cognitive and psychosocial recovery

congenital deficiency: PFFD

proximal femoral focal dysplasia lose knee and foot fxn with complicated hip fxn

ogden and Murphy group 3

proximal fractures with major displacements, comminution (splintering) involving upper tibial region

CN injury observations and what CN they indicate

ptosis (III) facial droop or asymmetry (VII) hoarse voice (X) articulation of words (V, VII, X, XII) abnormal eye position (III, IV, VI) abnormal or asymmetrical pupils (II, III)

roylan humeral cuff sling

pull humerus vertically with force enveloping the humerus pull superior no increase horizontal asymmetry

acute compartment syndrome

pulselessness, numbness, paleness, paralysis, worsening pain

ranchos level IX

purposeful appropriate SBA on request

rancho level IX

purposeful appropriate SBA on request I with tasks up to 2 hours, can think about consequences of actions, accurately estimates abilities, can self monitor social appropriateness testing phase

ranchos level X

purposeful appropriate modified independent

rancho level VIII

purposeful, appropriate SBA consistently oriented x3, familiar tasks I for 1 hour, recall past and present, assistive memory device, aware of impairment but needs SBA to correct, acknowledges others needs/feelings, low frustration tolerance

rancho level X

purposeful, appropriate: mod I can multitask, consistently appropriate social behavior, may require more time uneasy acceptance

treatment for posterior tibial tendonitis

reduce stress flexibility control inflammation PRICED strengthen- progressing to closed chain and eccentric proprioception

positioning in ICU

reposition every 2 hours between supine and side lying

what is reciprocal inhibition

reflex action occurs when overactive muscle causes activity of its antagonist to be suppressed

acute care and inpatient

rehab goals secondary to medical stability basic ADLs

eyes 1 glasgow

remain closed

IT band friction syndrome MOI

repetitie knee bending (30 degrees) tightness of TFL/ITB running on uneven surfaces faulty biomechanics

FAI results in ____ which can lead to ____

repetitive contact between femoral neck and acetabular rim labral tears and damage to acetabular articular cartilage

HKFO issues

restrictive and significantly higher energy costs to use needs assistive device and swing-to or swing-through gait high rejection rate

post-op rehab for FAI (phase 2)

resume full-weight bearing more-normal walking bridges, squats, side-step, step-up

treatment of chronic

return to function CV endurance muscle power movement re-ed (RNT) activity tolerance

what can lateral flexion cause

ribcage shift downward rotation shoulder asymmetry loss of scapular stability

unilateral body neglect often occurs with ____

right parietal lobe lesions (couple with issues of unilateral spatial neglect)

with plantar flexion, roll and glide

roll posterior glide anterior in open packed

SCOI shoulder brace used for and degrees

rotator cuff repairs Bankhart procedures prosthetic shoulder replacements 30-150 degrees of abduction

multiple ligament injured knee

rupture of 2+ ligaments increased rotary instability can have neuromuscular involvement and bleed out more likely to develop arthrofibrosis and have prolonged dysfunction **can cause full knee dislocation

mallet finger splinting

ruptured extensor digitorum Typically splinted for 6 weeks if no avulsion fx. Often referred to a Hand Specialist with avulsion or unable to achieve full Passive extension or greater than 30% of joint

neural structures of the knee

sciatic (with common fibular branch) common fibular splits into peroneals tibial (extension of sciatic) sural (branch of tibial) femoral (with saphenous branch)

tests for acoustic nerve

screen hearing test for lateralization compare air and bone conduction

signs and symptoms of a closed head injury

secondary swelling CNs controlling eye muscles are affected CN III compression leading to dilation

MCL: secondary role _____

secondary: provide anterior knee stability (enhanced by ER tibia)

sensory -> _______ --> ________ --> ________

sensory --> perception --> cognition --> behavior

ASIA T2 through L1

sensory only

ASIS C1-4

sensory only

FAI correlation and conjoint problems

slipped capital femoral epiphysis and acetabulofemoral impingement conjointly with movement impairment syndromes (anterior femoral glide)

compensation for weak plantar

solid AFO

three types of sensory problems

somatosensory visual vestibular

management of acetabula-femoral dislocation

some immobilization active isolated motions **don't want patient to perform combines motions that produce dislocation until 6-7 weeks

where is the subgluteus medium bursa

somewhat posterior and superior to the proximal edge of the greater trochanter.

what about subluxation causes shoulder pain

spastic IRs stretching and muscle imbalance limited ROM impingement tendonitis bursitis shoulder-hand syndrome

triple flexion

spinal reflex consisting of flexion of hips and knees due to painful stimulation

what is recovery of diaschisis

state of low reactivity that occurs after injury to brain

four subtypes of CMT

sternomastoid tumor muscular torticollis postural torticollis postnatal muscular torticollis

diagnosing FAI

stiff and non-painful hip exacerbated by activity and long-term sitting positive Fabere's restricted flex and AD rotation produces bony end-feels

treatment for scoliosis

strengthen convex stretch concave respiratory surgery >45 degrees orthosis 30-40degrees 20-29 with progressive

OA is a disparity between ____ and ____

stress applied to articular cartilage strength of articular cartilage

____ are modulated in the GS muscle groups and lead to improved push-off and surface recognition

stretch reflexes

PT intervention for AMC

stretching positioning strengthening bracing functional mobility assistive tech

most common hip bursitis (two bursa)

sub gluteus medius bursa subgluteus maximus bursa

signs and symptoms of meniscal injury

sudden locking or catching localized joint line pain giving way delayed effusion pain with WB pain on hyperflexion/squat + mcmurrays test or thessalys test

MOI for hamstring strain

sudden maximal contraction (concentric) rapid eccentric contraction to decelerate limb

What is prepatellar bursitis?

swelling outside of the knee joint

signs and symptoms of tibial plateau fracture

swelling/effusion unable to weight-bear stiffness

People who ambulate at T10-12 will use what kind of gait pattern

swing through

People who ambulate at T1-9 will use what kind of gait pattern

swing to

where does the pain of a meniscal injury come from

synovitis

After a stroke, thrombolytic therapy ___ can help control the damage, but it must be given w/in ___ hours of the onset of the stroke to be effective.

t-PA; 3

what bone is mechanically involved with all three major joints of the ankle

talus talocrural, subtaler, and transverse ligaments

test for trigeminal nerve

temporal and masseter muscle strength three division for pain sensation corneal reflex

cranial nerve V test

temporal and masseter muscles facial pain sensation corneal reflex

Morton's neuroma

tenderness in web spaces lateral compression of forefoot improper shoes

what is tensegrity

tension integrity;quality of structures that maintain their integrity due primarily to balance b/w tension and compression

common sites of avulsion fracture

the ASIS, AIIS, greater trochanter, and lesser trochanter.

what is a depressed skull fracture

the broken piece of skull bone moves in towards the brain; can be open or closed

lower trap and serratus weakness =

trouble with upward rotation

principle: specificity

the nature of the training experience dictates the nature of the plasticity

what is a compound skull fracture

the scalp is cut and the skull is fractured

synovial plica

thickening of synovial membrane problem only if it becomes irritated over a bony area or through repetitive use

when there is no PEAK

thoracic flexes elongation with weakness in back extensor shortened RA/obliques

Type 3 Salter-Harris Fracture

through growth plate and epiphysis

above knee amputations

through-the-knee: allows distal weight bearing trans/supracondylar: no distal weight-bearing transfemoral- shorter lever arm, muscle imbalance myoplasty- suture muscles over distal femur myodesis- sewn directly to distal femur hip disarticulation- lose all LE biomechanics hemipelvectomy- loss of WB surface for sitting

muscles that plantar flex and invert

tibialis posterior FDL FHL achilles tendon

deep posterior LE compartment

tibialis posterior FDL FHL posterior tibial and peroneal artery posterior tibial n

primary function of ligament teres

to carry a small artery (a branch of the obturator artery) to the head of the femur in early life, but little to none in adults. Also contains mechanoreceptors

when is serial casting used

to gain range of motion and/or correct equinus deformity

patients with excessive extensor tone throughout should be laid on the SIDE due to

tonic labyrinthine reflex

complete SCI injury

total absence of sensory and motor function in the lowest sacral level complete severing of the cord

chopart amputation deficits

total loss of anterior level arm and weight-bearing surface equinas deformity

principle: salience matters

training experience must be sufficiently salient to induce plasticity

principle: use it and improve it

training that drives a specific brain function can lead to an enhancement of that function

Neurogenic Heterotrophic Ossification

traumatic SCI

what is a closed head injury

traumatic blow to the head without a fracture to the skull

People who ambulate at S1-3 will use what kind of gait pattern

two or 3 point

People who ambulate at L4-5 will use what kind of gait pattern

two or four point

severe BI criteria

unconscious for days-months four subcategories: locked in, vegetative, persistent/permanent vegetative state

the mid tarsal joint is _____

uniaxial

the lack of visual spatial attention in those with R hemi lesions indicates ____

unilateral neglect syndrome

flaccid stage of recovery results in

unopposed gravitational pull on the arm and inferior sublux of the humeral head

pronator drift is indicative of

upper motor neuron disease

immobilization AFO

used in distal tibia/fibula fracture foot bone fracture tendocalcaneus rupture Charcot foot

HKFO indications

used to allow upright positioning and for encouraging limited ambulation Benefits of being upright: improved: circulation, bowel & bladder motility, reduced spasticity, reduced osteoporosis, prevention of joint contractures, improved kidney function, psychological benefits of being more "able" and part of the "regular" population, limited ambulation, and many other benefits

floor rxn AFO

uses floor reaction force through toe aspect of foot plate to prevent forward tibial progression and subsequent knee collapse

what mechanism often fails after brain injury

vessel constriction and dilation in relation to SBP there is some compensation through blood and CSF moving from brain into spina columns

cranial nerve VIII test

vestibulocochlear hearing lateralization of hearing compare air to bone

tests for optic nerve

visual acuity screen visual fields by confrontation pupillary rxn to light pupillary rxn to accomodation

what does the posterior visual pathway do

visual attention visuomemory spatial relations depth perception

remedial intervention approach VVMAC

visual demonstration verbal instructions manual guidance appropriate feedback consistent and repetitive practice

unilateral neglect syndrome includes hemineglect along with losses in ____

visual field somatosensation body awareness spatial awareness hemiparesis

early signs of heterotrophic ossifications

warmth, swelling, significant decreased ROM, pain MOST common: shoulder, elbow, hip, knee

superior sublux caused by

weak infra, subscap, and teres minor activation of delts without downward glide

what is OA

weakening of cartilage chronic joint disorder progressive disintegration of articular cartilage growth of osteophytes

hypoxic brain injury

when brain receives some but not enough oxygen

causes of central cord syndrome

• Anterior and/or posterior cord compression • Caused by an acute hyperextension injury. • Caused by chronic or congenital condition that results in progressive stenosis • Spondylosis • Osteophytes can create a "pincer" effect • Damage is from microvascular compromise of the center of the cord

anterior cord syndrome causes

• Aortic insufficiency • Atherosclerosis • Disc herniation • Bone, cartilage, or other trauma that compromises the integrity of the anterior spinal artery. • Frequently caused by teardrop or "burst" fractures of the vertebral body.

how to reduce edema

• Ask patient to sleep with arm in 30 degrees elevation • Use compression wraps to control edema • Use splints • Use activity and exercise • Gentle retrograde massage

clinical picture of central cord syndrome

• Bilateral flaccid paralysis and sensation loss due to loss of the grey matter (LMN!!) AT the level of the injury • BELOW the level of the injury the client will have spastic paralysis in the order listed in the previous slide. • Clients can re-gain motor function and sensation and will do so first in the sacral, then lumbar, then thoracic and finally cervical tracts. • Progress may stop at any time • 77% regain ambulation, 53% BBS, 42% hand fxn

clinical picture anterior cord syndrome

• Bilateral flaccid paralysis and sensation loss due to loss of the grey matter (LMN!!) AT the level of the injury • BELOW the level of the injury the client will have spastic paralysis with voluntary motor and (pain and temperature) sensory loss. • Posterior column function remains intact on both sides. • They have touch, proprioception, kinesthesia, sense of pressure • They LOSE pain and motor • Prognosis is poor for ANY motor return

what happens with central cord syndrome

• Central gray matter is compromised first because its metabolic and and perfusion needs are greater, thus is more at risk during periods of compromised circulation • Central white matter is also compromised • Any hemorrhage or edema begin in the center of the cord and spread to the periphery. • Resolution occurs in the opposite manner

decerebrate posture cause

• Damage to upper midbrain and lower pons; can also be a sign of bleeding in the brain or brain herniation; far more serious than decorticate posturing • It is possible to have alternating decorticate and decerebrate posturing on one side of the body or the other.

PNF cons of remediation

• Delay of fxnl independence • Significant hands-on approach • Labor-intensive and prolonged • Poor evidence

prognosis for cauda equina lesion

• EXCELLENT!! Peripheral injuries regenerate! • Pain can be a limiting factor to return of strength and function. • Average clients will NOT re-gain calf and foot intrinsic strength (due to degeneration of the myoneural junction over the time the nerve needs to regenerate), and may require orthotics. • Nerves regrow about an inch a month

causes of edema

• Lack of muscle contraction acting as a vascular pump • Entrapment/impingement due to a postural change after stroke • Sympathetic nerve response to hemiplegia (RSD) • Blood clot/DVT

incomplete lesion

• One in which there is partial preservation of sensory and or motor function below the neurological level and in the lowest sacral segment. (S2-5) • Is there bowel, bladder, sexual function? If so, it's incomplete **named by the lowest normal level**

PNF pros of compensation

• Quick compensation • Environmental adaptation • Practice new skills

intervention for RLA level IV and V

• Reduce agitation through motor activity and structured schedule • May need 24-hour supervision

PNF pros of remediation

• Reduce sensorimotor deficits and promote motor recovery/improve function of impaired segments • Requires voluntary movement • Training focuses on remediation of impairments

what do you want to do immediately following a stroke

• Stop progression of lesion • Reduce cerebral edema • Decrease risk of hydrocephalus • Treat seizures • Reduce secondary complications

PNF cons of compensation

• Suppress some parts of recovery • Learned non-use • Develop splinter skills: cannot be easily generalized to other environments or variations of the same task

how to name SCI

• The level named is the last level in which motor and sensory function are completely normal

cauda equina lesions clinical picture

• These injuries occur at L1 vertebrae or below. • Damage thus occurs to the cauda equina, NOT the spinal cord! • Because these injuries injure peripheral nerves, there is a flaccid paralysis with no spasticity. • Patients DO have severe amounts of pain, parasthesia, burning, and tingling occurs. • Peripheral nerves grow back

brown squared syndrome

• When something bisects the spinal cord (stab, gun, etc.) • Can be causes by unilateral facet lock injuries, burst fractures at the lateral body of the vertebrae What happens- One half of the spinal cord in damaged • Prognosis is very good • Nearly all patients are able to walk, although some with orthotics on one leg and with a cane... • 80% regain hand function, 100% regain bladder function, 80% regain bowel function.

three strategies for DD

• pathophysiologic reasoning- signs and symptoms • pattern recognition- identify a problem via characteristic groupings of findings, signs, and symptoms. (aunt Minnie) • probabilistic reasoning- use the information at hand along with knowledge of the incidence of certain pathologies to express the likelihood of a specific problem (ex: heard pop + playing football + can't walk = ACL)

what is shoulder/hand syndrome

• sympathetic nervous system overflow • hypersensitivity to pain • lose active (those with motor control) and passive (without motor control) ROM, stand weight bearing

considering age of child with Thera ex

≤7 y/o = little or no weight and low volume 8-10 y/o = gradual increase in load and low weight (e.g., 15 reps with good form, 1-3 lbs) 11-13 y/o = progressive load, but advance exercise should have little or no load 14-15 y/o = resistance with sport specific exercise ≥ 16 y/o = entry-level adult programs


Conjuntos de estudio relacionados

Chapter 28: Developing the Role of the Leader

View Set

Management quiz for chapter 9,10,11 and 12

View Set

7 Habits of Highly Effective People

View Set

Chapter 2 Testing in the Context of the Software Development Lifecycle

View Set

Spanish Verbs Irregular in the Present Tense Yo Form

View Set